NCLEX: Assessment of Cardiovascular Function

Pataasin ang iyong marka sa homework at exams ngayon gamit ang Quizwiz!

The nurse is caring for a patient prescribed warfarin (Coumadin) orally. The nurse reviews the patient's prothrombin time (PT) level to evaluate the effectiveness of the medication. The nurse should also evaluate which of the following laboratory values?

International normalized ratio (INR)

A nurse is performing a cardiac assessment on an elderly client. Which finding warrants further investigation?

Irregularly irregular heart rate

High LDL Levels

Jack Donohue, a 62-year-old stock broker, attends his annual physical appointment and indicates physical changes since his last examination. He reports chest pain and palpitation during and after his morning jogs. Jack's family history reveals includes coronary artery disease. His lipid profile reveals his LDL level to be 122 mg/dl. Which of the following correctly states the Jack's condition?

The nurse is reviewing the morning laboratory test results for a client with cardiac problems. Which of the following would the nurse regard as a priority to report to the physician?

K+ 3.1 mEq/L

Within the physiology of the heart, each chamber has a particular role in maintaining cellular oxygenation. Which chamber of the heart is responsible for receiving oxygenated blood from the lungs?

Left atrium

Within the physiology of the heart, each chamber has a particular role in maintaining cellular oxygenation. Which chamber of the heart is responsible for pumping blood to all the cells and tissues of the body?

Left ventricle

What is a common urinary problem in people with heart failure?

Nocturia

Admission lab values on a patient admitted with congestive heart failure are as follows: potassium 3.4 mEq/L; sodium 148 mEq/L; calcium 9.8 mg/dL; and magnesium 1.5 mEq/L. Which lab value is abnormal?

Sodium

The nurse correctly identifies which of the following data as an example of BP and HR measurements in a patient with postural hypotension?

Supine: BP 120/70 mm Hg, HR 70 bpm; sitting: BP 100/55 mm Hg, HR 90 bpm; standing: BP 98/52 mm Hg, HR 94 bpm

Before a transesophageal echocardiogram, a nurse gives a client an oral topical anesthetic spray. When the client returns from the procedure, the nurse observes that he has no active gag reflex. In response, the nurse should:

withhold food and fluids

before a transesophageal echocardiogram, a nurse gives a client an oral topical anesthetic spray. When the client returns from the procedure, the nurse observes that he has no active gag reflex. In response, the nurse should:

withhold food and fluids

After receiving nitroglycerin (Nitrostat), a client verbalizes relief of chest pain. The physician prescribes transdermal nitroglycerin (Nitro-Dur), 5-mg patch daily, as prophylaxis for angina pectoris. When teaching the client how to apply the transdermal system, the nurse should provide which instruction? 1. "Use the same clean, hairless application site each day." 2. "You may touch the medication pad after washing your hands." 3. "Be sure to report skin irritation or other adverse reactions." 4. "Store your supply of transdermal pads in the refrigerator."

Correct Answer: 3 RATIONALES: This is the only correct instruction because transdermal nitroglycerin may cause skin irritation. The other options reflect incorrect teaching regarding transdermal nitroglycerin. The client should rotate the patch application site daily to prevent sensitization and tolerance; should avoid touching the medication-impregnated pad because this may cause drug absorption; and should store pads away from temperature and humidity extremes, which may inactivate the drug.

During a routine checkup, the nurse performs a physical examination on a client with aortic insufficiency. The client's history reveals an Austin Flint murmur. To best hear this murmur, the nurse should place the stethoscope: 1. over the carotid artery. 2. at the base of the heart. 3. at the apex of the heart. 4. at the left fifth intercostal space.

Correct Answer: 3 RATIONALES: To best hear an Austin Flint murmur, the nurse should place the stethoscope at the apex of the heart. An Austin Flint murmur produces a soft, low-pitched, rumbling, middiastolic or presystolic bruit. Placing the stethoscope over the carotid artery, at the base of the heart, or at the left fifth intercostal space would make this murmur more difficult to hear.

A client with severe angina and ST-segment elevation on the electrocardiogram is being seen in the emergency department. In terms of diagnostic laboratory testing, it's most important for the nurse to advocate ordering a: 1. creatine kinase level. 2. hemoglobin (Hb) level. 3. troponin level. 4. liver panel.

Correct Answer: 3 RATIONALES: Troponin is a myocardial cell protein that is elevated in the serum when myocardial damage has occurred during a myocardial infarction (MI). It's the best serum indicator of MI and is more indicative of cardiac damage than creatine kinase. Hb values and liver panel components aren't as useful in the diagnosis of MI as a troponin level.

A client with no known history of peripheral vascular disease comes to the emergency department complaining of sudden onset of lower leg pain. Inspection and palpation reveal absent pulses; paresthesia; and a mottled, cyanotic, cold, and cadaverous left calf. While the physician determines the appropriate therapy, the nurse should: 1. place a heating pad around the affected calf. 2. elevate the affected leg as high as possible. 3. keep the affected leg level or slightly dependent. 4. shave the affected leg in anticipation of surgery.

Correct Answer: 3 RATIONALES: While the physician makes treatment decisions, the nurse should maintain the client on bed rest, keeping the affected leg level or slightly dependent (to aid circulation) and protecting it from pressure and other trauma. Warming the leg with a heating pad (or chilling it with an ice pack) would further compromise tissue perfusion and increase injury to the leg. Elevating the leg would worsen tissue ischemia. Shaving an ischemic leg may cause accidental trauma from cuts or nicks.

When assessing a client with left-sided heart failure, the nurse expects to note: 1. ascites. 2. jugular vein distention. 3. air hunger. 4. pitting edema of the legs.

Correct Answer: 3 RATIONALES: With left-sided heart failure, the client typically has air hunger and other signs of pulmonary congestion. Ascites, jugular vein distention, and pitting edema of the legs are signs of right-sided heart failure.

A client is admitted to the emergency department after complaining of acute chest pain radiating down his left arm. Which laboratory studies would be indicated? 1. Hemoglobin and hematocrit 2. Serum glucose 3. Creatinine phosphokinase (CPK) 4. Troponin T and troponin I 5. Myoglobin 6. Blood urea nitrogen (BUN)

Correct Answer: 3,4,5 RATIONALES: Levels of CPK, troponin T, and troponin I elevate because of cellular damage. Myoglobin elevation is an early indicator of myocardial damage. Hemoglobin, hematocrit, serum glucose, and BUN levels don't provide information related to myocardial ischemia.

The nurse is awaiting the arrival of a client from the emergency department. The client has a left ventricular myocardial infarction and is being admitted. In caring for this client, the nurse should be alert for which signs and symptoms of leftsided heart failure? 1. Jugular vein distention 2. Hepatomegaly 3. Dyspnea 4. Crackles 5. Tachycardia 6. Right upper quadrant pain

Correct Answer: 3,4,5 RATIONALES: Signs and symptoms of left-sided heart failure include dyspnea, orthopnea, and paroxysmal nocturnal dyspnea; fatigue; nonproductive cough and crackles; hemoptysis; point of maximal impulse displaced toward the left anterior axillary line; tachycardia and S3 and S4 heart sounds; and cool, pale skin. Jugular vein distention, hepatomegaly, and right upper quadrant pain are all signs of right-sided heart failure.

The nurse is evaluating the 12-lead electrocardiogram (ECG) of a client experiencing an inferior wall myocardial infarction (MI). While conferring with the team, she correctly identifies which ECG changes associated with an evolving MI? 1. Notched T-wave 2. Presence of a U-wave 3. T-wave inversion 4. Prolonged PR-interval 5. ST-segment elevation 6. Pathologic Q-wave

Correct Answer: 3,4,5 RATIONALES: T-wave inversion, ST-segment elevation, and a pathologic Q-wave are all signs of tissue hypoxia which occur during an MI. Ischemia results from inadequate blood supply to the myocardial tissue and is reflected by T-wave inversion. Injury results from prolonged ischemia and is reflected by ST-segment elevation. Q-waves may become evident when the injury progresses to infarction. A notched T-wave may indicate pericarditis in an adult client. The presence of a U-wave may or may not be apparent on a normal ECG; it represents repolarization of the Purkinje fibers. A prolonged PR-interval is associated with first-degree atrioventricular block.

When measuring the radial pulse of a client with known aortic insufficiency, the nurse isn't surprised to find a "waterhammer" or Corrigan's pulse. What are the characteristics of this pulse? 1. Weak and feeble, with a slow upstroke and prolonged peak 2. Alternating strong and weak beats 3. Rapid upstroke with two systolic peaks 4. Bounding, with a rapid rise and fall

Correct Answer: 4 RATIONALES: A "water-hammer" pulse is bounding, with a rapid rise and fall. A weak, feeble pulse with a slow upstroke and prolonged peak is called pulsus tardus. A pulse with alternating weak and strong beats and a regular rhythm is termed pulsus alternans. A pulse with a rapid upstroke and two systolic peaks is called pulsus bisferiens.

After having several Stokes-Adams attacks over 4 months, a client reluctantly agrees to implantation of a permanent pacemaker. Before discharge, the nurse reviews pacemaker care and safety guidelines with the client and spouse. Which safety precaution is appropriate for a client with a pacemaker? 1. Stay at least 2′ away from microwave ovens. 2. Never engage in activities that require vigorous arm and shoulder movement. 3. Avoid going through airport metal detectors. 4. Avoid having magnetic resonance imaging (MRI).

Correct Answer: 4 RATIONALES: A client with a pacemaker should avoid having an MRI because the magnet may disrupt pacemaker function and cause injury to the client. Disruption is less likely to occur with newer microwave ovens; nonetheless, the client should stay at least 5′ away from microwaves, not 2′. The client must avoid vigorous arm and shoulder movement only for the first 6 weeks after pacemaker implantation. Airport metal detectors don't harm pacemakers; however, the client should notify airport security guards of the pacemaker because its metal casing and programming magnet may trigger the metal detector.

A white male, age 43, with a tentative diagnosis of infective endocarditis is admitted to an acute care facility. His medical history reveals diabetes mellitus, hypertension, and pernicious anemia; he underwent an appendectomy 20 years ago and an aortic valve replacement 2 years ago. Which history finding is a major risk factor for infective endocarditis? 1. Race 2. Age 3. History of diabetes mellitus 4. History of aortic valve replacement

Correct Answer: 4 RATIONALES: A heart valve prosthesis, such as an aortic valve replacement, is a major risk factor for infective endocarditis. Other risk factors include a history of heart disease (especially mitral valve prolapse), chronic debilitating disease, I.V. drug abuse, and immunosuppression. Although race, age, and a history of diabetes mellitus may predispose a person to cardiovascular disease, they aren't major risk factors for infective endocarditis.

A client is recovering from an acute myocardial infarction (MI). During the first week of recovery, the nurse should stay alert for which abnormal heart sound? 1. Opening snap 2. Graham Steell's murmur 3. Ejection click 4. Pericardial friction rub

Correct Answer: 4 RATIONALES: A pericardial friction rub, which sounds like squeaky leather, may occur during the first week after an MI. Resulting from inflammation of the pericardial sac, this abnormal heart sound arises as the roughened parietal and visceral layers of the pericardium rub against each other. Certain stenosed valves may cause a brief, high-pitched opening snap heard early in diastole. Graham Steell's murmur is a high-pitched, blowing murmur with a decrescendo pattern; heard during diastole, it indicates pulmonary insufficiency, such as from pulmonary hypertension or a congenital pulmonary valve defect. An ejection click, associated with mitral valve prolapse or a rigid, calcified aortic valve, causes a high-pitched sound during systole.

An elderly client who underwent total hip replacement exhibits a red, painful area on the calf of the affected leg. What test validates presence of thromboembolism? 1. Romberg's 2. Phalen's 3. Rinne 4. Homans'

Correct Answer: 4 RATIONALES: A positive Homans' sign, or pain in the calf elicited upon flexion of the ankle with the leg straight, indicates the presence of a thrombus. Romberg's test assesses cerebellar function. Phalen's test assesses carpal tunnel syndrome. The Rinne test compares air and bone conduction in both ears to screen for or confirm hearing loss.

A client is taking spironolactone (Aldactone) to control her hypertension. Her serum potassium level is 6 mEq/L. For this client, the nurse's priority would be to assess her: 1. neuromuscular function. 2. bowel sounds. 3. respiratory rate. 4. electrocardiogram (ECG) results.

Correct Answer: 4 RATIONALES: Although changes in all these findings are seen in hyperkalemia, ECG changes can indicate potentially lethal arrhythmias such as ventricular fibrillation. It wouldn't be appropriate to assess the client's neuromuscular function, bowel sounds, or respiratory rate for effects of hyperkalemia.

When assessing a client who reports recent chest pain, the nurse obtains a thorough history. Which statement by the client most strongly suggests angina pectoris? 1. "The pain lasted about 45 minutes." 2. "The pain resolved after I ate a sandwich." 3. "The pain got worse when I took a deep breath." 4. "The pain occurred while I was mowing the lawn."

Correct Answer: 4 RATIONALES: Angina pectoris is chest pain caused by a decreased oxygen supply to the myocardium. Lawn mowing increases the cardiac workload; this, in turn, increases the heart's need for oxygen and may precipitate angina. Anginal pain typically is self-limiting and lasts 5 to 15 minutes. Food consumption doesn't reduce this pain, although it may ease pain caused by a GI ulcer. Deep breathing has no effect on anginal pain.

The nurse is instructing a client about the use of antiembolism stockings. Antiembolism stockings help prevent deep vein thrombosis (DVT) by: 1. encouraging ambulation to prevent pooling of blood. 2. providing warmth to the extremity. 3. elevating the extremity to prevent pooling of blood. 4. forcing blood into the deep venous system.

Correct Answer: 4 RATIONALES: Antiembolism stockings prevent DVT by forcing blood into the deep venous system, instead of allowing blood to pool. Ambulation prevents blood from pooling and prevents DVT, but encouraging ambulation isn't a function of the stockings. Antiembolism stockings could possibly provide warmth, but this isn't how they prevent DVT. Elevating the extremity will decrease edema but won't prevent DVT.

The nurse is teaching a client who will be discharged soon with a prescription for warfarin (Coumadin). The nurse should include which statement in discharge teaching? 1. "Increase your intake of yogurt and broccoli." 2. "This drug will dissolve any clots you may still have." 3. "If you miss a dose, double the next dose." 4. "Avoid aspirin while taking warfarin."

Correct Answer: 4 RATIONALES: Because aspirin decreases platelet agglutination and interferes with clotting, concomitant use of aspirin with warfarin, an anticoagulant, may lead to excessive anticoagulant effects — and bleeding. Warfarin therapy doesn't necessitate dietary changes. Although warfarin interrupts the normal clotting cycle, it doesn't dissolve clots that have already formed. The client should take warfarin exactly as prescribed to maintain the desired level of anticoagulation. Doubling a dose could lead to bleeding.

The physician is treating a client in the cardiac care unit for atrial arrhythmia and prescribes propranolol (Inderal), 10 mg P.O. three times a day. Propranolol inhibits the action of sympathomimetics at beta1-receptor sites. Where these sites are mainly located? 1. Uterus 2. Blood vessels 3. Bronchi 4. Heart

Correct Answer: 4 RATIONALES: Beta1-receptor sites are mainly located in the heart. Beta2-receptor sites are located in the uterus, blood vessels, and bronchi.

The monitor technician on the telemetry unit asks the charge nurse why every client whose monitor shows atrial fibrillation is receiving warfarin (Coumadin). Which response by the charge nurse is best? 1. "It's just a coincidence; most clients with atrial fibrillation don't receive warfarin." 2. "Warfarin controls heart rate in the client with atrial fibrillation." 3. "Warfarin prevents atrial fibrillation from progressing to a lethal arrhythmia." 4. "Warfarin prevents clot formation in the atria of clients with atrial fibrillation."

Correct Answer: 4 RATIONALES: Blood pools in the atria of clients with atrial fibrillation. As the blood pools, clots form. These clots can be forced from the atria as the heart beats, placing the client at risk for stroke. Warfarin is prescribed in most clients with atrial fibrillation to prevent clot formation and decrease the risk of stroke, not to control heart rate. Digoxin is typically prescribed to control heart rate in atrial fibrillation. Atrial fibrillation doesn't typically progress to a lethal arrhythmia such as ventricular fibrillation.

A client in the emergency department complains of squeezing substernal pain that radiates to the left shoulder and jaw. He also complains of nausea, diaphoresis, and shortness of breath. What should the nurse do? 1. Complete the client's registration information, perform an electrocardiogram, gain I.V. access, and take vital signs. 2. Alert the cardiac catheterization team, administer oxygen, attach a cardiac monitor, and notify the physician. 3. Gain I.V. access, give sublingual nitroglycerin, and alert the cardiac catheterization team. 4. Administer oxygen, attach a cardiac monitor, take vital signs, and administer sublingual nitroglycerin.

Correct Answer: 4 RATIONALES: Cardiac chest pain is caused by myocardial ischemia. Administering supplemental oxygen increases the myocardial oxygen supply. Cardiac monitoring helps detect life-threatening arrhythmias. Ensure that the client isn't hypotensive before giving sublingual nitroglycerin for chest pain. Registration information may be delayed until the client is stabilized. Alerting the cardiac catheterization team before completing the initial assessment is premature.

A client with refractory angina is scheduled for a percutaneous transluminal coronary angioplasty (PTCA). The cardiologist orders an infusion of abciximab (ReoPro). Before beginning the infusion, the nurse should ensure the client has: 1. negative history of tonic-clonic seizures. 2. ampule of naloxone (Narcan) at the bedside. 3. continuous electrocardiogram (ECG) monitoring. 4. up-to-date partial thromboplastin time (PTT) result in his record.

Correct Answer: 4 RATIONALES: Clients undergoing PTCA receive abciximab because it inhibits platelet aggregation and, thereby, reduces cardiac ischemic complications. Before abciximab is administered, the client should have an up-to-date PTT result available. The drug isn't contraindicated in clients with a seizure history. Abciximab isn't an opioid; therefore, an opioid antagonist doesn't need to be at the bedside. Any client with refractory angina should be on continuous ECG monitoring; however, monitoring isn't a requirement for administering abciximab.

A client with venous insufficiency develops varicose veins in both legs. Which statement about varicose veins is accurate? 1. Varicose veins are more common in men than in women. 2. Primary varicose veins are caused by deep vein thrombosis and inflammation. 3. Sclerotherapy is used to cure varicose veins. 4. The severity of discomfort isn't related to the size of varicosities.

Correct Answer: 4 RATIONALES: Clients with varicose veins commonly complain of aching, heaviness, itching, moderate swelling, and unsightly appearance of the legs. However, the severity of discomfort is hard to assess and seems unrelated to the size of varicosities. Varicose veins are more common in women than in men. Primary varicose veins typically result from a congenital or familial predisposition that makes the vein wall less elastic; secondary varicosities occur when trauma, obstruction, deep vein thrombosis, or inflammation damages valves. Sclerotherapy, in which a sclerosing agent is injected into a vein, is used to treat varicose veins; it doesn't cure them.

The nurse records a client's history and discovers several risk factors for coronary artery disease. Which cardiac risk factors are considered controllable? 1. Diabetes, hypercholesterolemia, and heredity 2. Diabetes, age, and gender 3. Age, gender, and heredity 4. Diabetes, hypercholesterolemia, and hypertension

Correct Answer: 4 RATIONALES: Controllable risk factors include hypertension, hypercholesterolemia, obesity, lack of exercise, smoking, diabetes, stress, alcohol abuse, and use of contraceptives. Uncontrollable risk factors for coronary artery disease include gender, age, and heredity.

What mechanical device increases coronary perfusion and cardiac output and decreases myocardial workload and oxygen consumption in a client with cardiogenic shock? 1. Cardiac pacemaker 2. Hypothermia-hyperthermia machine 3. Defibrillator 4. Intra-aortic balloon pump

Correct Answer: 4 RATIONALES: Counterpulsation with an intra-aortic balloon pump may be indicated for temporary circulatory assistance in clients with cardiogenic shock. Cardiac pacemakers are used to maintain the heartbeat at a predetermined rate. Hypothermia-hyperthermia machines are used to cool or warm clients with abnormalities in temperature regulation. The defibrillator is commonly used for termination of life-threatening ventricular rhythms.

The physician prescribes digoxin (Lanoxin) for a client with heart failure. During digoxin therapy, which electrolyte imbalance may predispose the client to digitalis toxicity? 1. Hypermagnesemia 2. Hypercalcemia 3. Hypernatremia 4. Hypokalemia

Correct Answer: 4 RATIONALES: During digoxin therapy, conditions that may predispose a client to digitalis toxicity include hypokalemia, hypomagnesemia, hypothyroidism, hypoxemia, advanced myocardial disease, active myocardial ischemia, and altered autonomic tone. Hypermagnesemia, hypercalcemia, and hypernatremia aren't associated with a risk for digitalis toxicity.

The nurse is caring for a client with end-stage heart failure who is awaiting a heart transplant. The client tells the nurse that he thinks he's going to die before a donor heart is found. He also tells the nurse that he hasn't been attending a church but wants to talk to a priest. What action should the nurse take? 1. Contact her priest to see if he will see the client. 2. Reassure the client that he has nothing to worry about because donors are usually found in time. 3. Tell the client that it doesn't matter if he attends a church or not. 4. Contact the clergy member who is assigned to the transplant team.

Correct Answer: 4 RATIONALES: Each multidisciplinary transplant team has a clergy person assigned. The nurse should contact that person and request that he visit the client. It isn't appropriate for the nurse to ask her priest to see the client. Telling the client that he has nothing to worry about because donors are typically found offers false reassurance. Telling the client that it doesn't matter if he attends a church invalidates the client's concern.

Following coronary artery bypass grafting, a client begins having chest "fullness" and anxiety. The nurse suspects cardiac tamponade and prints a lead II electrocardiograph (ECG) strip for interpretation. In looking at the strip, the change in the QRS complex that would most support her suspicion is: 1. narrowing complex. 2. widening complex. 3. amplitude increase. 4. amplitude decrease.

Correct Answer: 4 RATIONALES: Fluid surrounding the heart such as in cardiac tamponade, suppresses the amplitude of the QRS complexes on an ECG. Narrowing and widening complexes as well as an amplitude increase aren't what is expected on the ECG of an individual with cardiac tamponade

A client comes to the physician's office for a follow-up visit 4 weeks after suffering a myocardial infarction (MI). The nurse takes this opportunity to evaluate the client's knowledge of the prescribed cardiac rehabilitation program. Which evaluation statement suggests that the client needs more instruction? 1. "Client performs relaxation exercises three times per day to reduce stress." 2. "Client's 24-hour dietary recall reveals low intake of fat and cholesterol." 3. "Client verbalizes an understanding of the need to seek emergency help if the heart rate increases markedly while at rest." 4. "Client walks 4 miles in 1 hour every day."

Correct Answer: 4 RATIONALES: Four weeks after an MI, a client's walking program should aim for a goal of 2 miles in less than 1 hour. Walking 4 miles in 1 hour is excessive and may induce another MI by increasing the heart's oxygen demands. Therefore, this client requires appropriate exercise guidelines and precautions. The other options indicate understanding of the cardiac rehabilitation program. For example, the client should reduce stress, which speeds the heart rate and thus increases myocardial oxygen demands. Reducing dietary fat and cholesterol intake helps lower the risk of atherosclerosis. A sudden rise in the heart rate while at rest warrants emergency medical attention because it may signal a life-threatening arrhythmia and increase myocardial oxygen demands.

A client with a history of an anterior wall myocardial infarction is being transferred from the coronary care unit (CCU) to the cardiac step-down unit (CSU). While giving a report to the CSU nurse, the CCU nurse says, "His pulmonary artery wedge pressures have been in the high normal range." The CSU nurse should be especially observant for: 1. hypertension. 2. high urine output. 3. dry mucous membranes. 4. pulmonary crackles.

Correct Answer: 4 RATIONALES: High pulmonary artery wedge pressures are diagnostic for left-sided heart failure. With left-sided heart failure, pulmonary edema can develop causing pulmonary crackles. In left-sided heart failure, hypotension may result and urine output will decline. Dry mucous membranes aren't directly associated with elevated pulmonary artery wedge pressures

A 43-year-old man was transferring a load of fire wood from his front driveway to his backyard woodpile at 10 a.m. when he experienced a heaviness in his chest and dyspnea. He stopped working and rested, and the pain subsided. At noon, the pain returned. At 1:30 p.m., his wife took him to the emergency department. Around 2 p.m., the emergency department physician diagnoses an anterior myocardial infarction (MI). The nurse should anticipate which orders by the physician? 1. Streptokinase, aspirin, and morphine sulfate administration 2. Morphine administration, stress testing, and admission to the cardiac care unit 3. Serial liver enzyme testing, telemetry, and a lidocaine infusion 4. Sublingual nitroglycerin, tissue plasminogen activator (tPA), and telemetry

Correct Answer: 4 RATIONALES: If 12 hours or fewer have passed since the onset of symptoms related to MI, thrombolytic therapy is indicated. (The client's chest pain began 4 hours before diagnosis.) The preferred choice is tPA, which is more specific for cardiac tissue than streptokinase. Stress testing shouldn't be performed during an MI. The client doesn't exhibit symptoms that indicate the use of lidocaine.

A client with a history of I.V. drug abuse is admitted to the medical-surgical unit for evaluation for infective endocarditis. Nursing assessment is most likely to reveal that this client has: 1. retrosternal pain that worsens during supine positioning. 2. pulsus paradoxus. 3. a scratchy pericardial friction rub. 4. Osler's nodes and splinter hemorrhages.

Correct Answer: 4 RATIONALES: Infective endocarditis occurs when an infectious agent enters the bloodstream, such as from I.V. drug abuse or during an invasive procedure or dental work. Typical assessment findings in clients with this disease include Osler's nodes (red, painful nodules on the fingers and toes), splinter hemorrhages, fever, diaphoresis, joint pain, weakness, abdominal pain, a new or altered heart murmur, and Janeway's lesions (small, hemorrhagic areas on the fingers, toes, ears, and nose). The other options are common findings in clients with pericarditis, not infective endocarditis.

An electrocardiogram (ECG) taken during a routine checkup reveals that a client has had a silent myocardial infarction. On a 12-lead ECG, which leads record electrical events in the septal region of the left ventricle? 1. Leads I, aVL, V5, and V6 2. Leads II, III, and aVF 3. Leads V1 and V2 4. Leads V3 and V4

Correct Answer: 4 RATIONALES: Leads V3 and V4 record electrical events in the septal region of the left ventricle. Leads I, aVL, V5, and V6 record electrical events on the lateral surface of the left ventricle. Leads II, III, and aVF record electrical events on the inferior surface of the left ventricle. Leads V1 and V2 record electrical events on the anterior surface of the right ventricle and the anterior surface of the left ventricle.

A client with mitral stenosis comes to the physician's office for a routine checkup. When listening to the client's heart, the nurse expects to hear which type of murmur? 1. Pansystolic, blowing, high-pitched 2. Systolic, harsh, crescendo-decrescendo 3. Diastolic, blowing, decrescendo 4. Diastolic, rumbling, low-pitched

Correct Answer: 4 RATIONALES: Mitral stenosis causes a diastolic, rumbling, low-pitched murmur heard at the apex. A pansystolic, blowing, high-pitched murmur characterizes mitral insufficiency. A systolic, harsh, crescendo-decrescendo murmur occurs with aortic insufficiency. A diastolic, blowing, decrescendo murmur accompanies aortic insufficiency.

The physician prescribes pentoxifylline (Trental), 400 mg three times daily with meals, for a client with intermittent claudication and a history of adult-onset diabetes mellitus. The nurse knows that pentoxifylline is a: 1. hemostatic agent. 2. tissue plasminogen activator. 3. thrombolytic agent. 4. blood viscosity-reducing agent.

Correct Answer: 4 RATIONALES: Pentoxifylline is a hemorheologic agent that improves blood flow by decreasing blood viscosity and is used to treat intermittent claudication. A hemostatic agent is used to stop excessive bleeding. A tissue plasminogen activator is used in early management of acute myocardial infarction. A thrombolytic agent is prescribed to dissolve clots and other substances in thrombi and emboli.

After abdominal surgery, which factor would predispose a client to deep vein thrombosis? 1. The client is 5′ 9″ tall and weighs 128 lb. 2. The client has been pregnant four times. 3. The client usually walks 3 miles a day. 4. The client will be immobile during and shortly after surgery.

Correct Answer: 4 RATIONALES: Postoperative immobility and subsequent venous stasis predispose the client to deep vein thrombosis. Other predisposing factors for this condition include obesity and current pregnancy, which don't apply to this client. Exercise isn't a risk factor or preventive measure for deep vein thrombosis.

The most important reason for the nurse to encourage a client with peripheral vascular disease to initiate a walking program is that this form of exercise: 1. reduces stress. 2. aids in weight reduction. 3. increases high-density lipoprotein (HDL) level. 4. promotes collateral circulation.

Correct Answer: 4 RATIONALES: Regular walking is the best way to promote collateral circulation, which becomes a critical source of blood supply to limbs with compromised blood flow distal to a stenotic lesion. Regular exercise also aids in stress reduction and weight reduction and increases the formation of HDLs — all of which are helpful for a client with peripheral vascular disease. However, these changes don't have as significant an effect on the client's condition as the development of collateral circulation.

After experiencing a transient ischemic attack (TIA), a client is prescribed aspirin, 325 mg P.O. daily. The nurse should teach the client that this medication has been prescribed to: 1. control headache pain. 2. enhance the immune response. 3. prevent intracranial bleeding. 4. reduce the chance of blood clot formation.

Correct Answer: 4 RATIONALES: TIAs are considered forerunners of stroke. Because strokes may result from clots in cerebral vessels, aspirin is prescribed to prevent clot formation by reducing platelet agglutination. A 325-mg dose of aspirin is inadequate to relieve headache pain in an adult. Aspirin has no effect on the body's immune response. Intracranial bleeding isn't associated with TIAs, and the action of aspirin probably would worsen any bleeding present.

A client with second-degree atrioventricular heart block is admitted to the coronary care unit. The nurse closely monitors the heart rate and rhythm. When interpreting the client's electrocardiogram (ECG) strip, the nurse knows that the QRS complex represents: 1. atrial repolarization. 2. ventricular repolarization. 3. atrial depolarization. 4. ventricular depolarization.

Correct Answer: 4 RATIONALES: The QRS complex on the ECG strip represents ventricular depolarization. Atrial repolarization usually occurs at the same time as ventricular depolarization and can't be distinguished on the ECG. The T wave represents ventricular repolarization. The P wave represents atrial depolarization.

A client with severe angina and electrocardiogram changes is seen by a nurse practitioner in the emergency department. In terms of serum testing, it's most important for the nurse to order cardiac: 1. creatine kinase. 2. lactate dehydrogenase. 3. myoglobin. 4. troponin.

Correct Answer: 4 RATIONALES: The client exhibits signs of myocardial infarction (MI), and the most accurate serum determinant of MI is troponin level. The other tests can show evidence of muscle injury but they're a less specific indicator of myocardial damage than troponin.

A client requested a do-not-resuscitate (DNR) order upon admission to the hospital. He now tells the nurse that he wants the medical team to do everything possible to help him get better and is concerned about the DNR order. Which response by the nurse is best? 1. "It is too late to change your mind now." 2. "We will have to ask your physician if this is possible." 3. "Why do you want to do this?" 4. "It's not a problem to rescind your DNR order; I'll let your physician know your wishes right away."

Correct Answer: 4 RATIONALES: The client is allowed to rescind a DNR order at any time. The client makes the decision about a DNR order with input from the physician. Questioning a client's motives can make the client feel defensive and shut down communication with the nurse.

A client with chest pain, dyspnea, and an irregular heartbeat comes to the emergency department. An electrocardiogram shows a heart rate of 110 beats/minute (sinus tachycardia) with frequent premature ventricular contractions. Shortly after admission, the client has ventricular tachycardia and becomes unresponsive. After successful resuscitation, the client is taken to the intensive care unit (ICU). Which nursing diagnosis is appropriate at this time? 1. Deficient knowledge (disease process) related to interventions used to treat acute illness 2. Impaired physical mobility related to complete bed rest 3. Social isolation related to restricted visiting hours in the ICU 4. Ineffective tissue perfusion (cardiopulmonary) related to arrhythmia

Correct Answer: 4 RATIONALES: The client suffered a lethal arrhythmia, requiring immediate resuscitation. This arrhythmia was caused by ineffective perfusion to the heart. Therefore, the client should have the nursing diagnosis Ineffective tissue perfusion (cardiopulmonary). Client teaching should be limited to clear, concise explanations that reduce anxiety and promote cooperation. An anxious client has difficulty learning, so the knowledge deficit would continue despite attempts at teaching. Impaired physical mobility and Social isolation are necessitated by the client's critical condition; therefore, they are considered therapeutic, not problems warranting nursing diagnoses.

A 53-year-old client is about to undergo cardiac catheterization for which he signed an informed consent. As the nurse enters the room to administer sedation for the procedure, the client states, "I'm really worried about having this open heart surgery." Based on this statement, how should the nurse proceed? 1. Medicate the client and document his comment. 2. Medicate the client and notify the physician about the comment. 3. Explain that cardiac catheterization doesn't involve open heart surgery, and then medicate the client. 4. Withhold the medication and notify the physician immediately.

Correct Answer: 4 RATIONALES: The nurse should withhold the medication and notify the physician that the client doesn't understand the procedure. The physician then has the obligation to explain the procedure better to the client and determine whether or not the client understands. If the client doesn't understand, he can't give a true informed consent. If the medication is administered before the physician explains the procedure, the sedation may interfere with the client's ability to clearly understand the procedure. The nurse can't just medicate the client and document her finding; she must notify the physician.

How long after oral administration can the nurse expect to see digoxin's (Lanoxin) peak effect? 1. 2 to 5 minutes 2. 10 to 20 minutes 3. 30 minutes to 2 hours 4. 2 to 6 hours

Correct Answer: 4 RATIONALES: The peak effect of digoxin occurs 2 to 6 hours after an oral dose and 1 to 4 hours after an I.V. dose. Digoxin's onset of action ranges from 30 minutes to 2 hours after an oral dose and from 5 to 30 minutes after an I.V. dose.

To avoid a falsely elevated serum digoxin level, the nurse should wait how long after administering oral digoxin (Lanoxin) to draw a blood sample? 1. At least 1 hour 2. At least 4 hours 3. At least 6 hours 4. At least 8 hours

Correct Answer: 4 RATIONALES: To avoid a falsely elevated serum digoxin level, the nurse should wait at least 8 hours after administering oral digoxin and at least 6 hours after administering I.V. digoxin to draw a blood sample. In most cases, a serum sample is taken immediately before administering the daily maintenance dose, about 24 hours after the last dose.

A client is admitted to the acute care facility for treatment of heart failure. The nurse expects the physician to prescribe which drug? 1. prednisone (Orasone) 2. hydroxychloroquine sulfate (Plaquenil Sulfate) 3. lidocaine (Xylocaine) 4. furosemide (Lasix)

Correct Answer: 4 RATIONALES: To maintain fluid balance — crucial for a client with heart failure — the physician typically prescribes a diuretic, such as furosemide; vasodilating agents; and drugs that increase contractility, such as digitalis glycosides. Prednisone, a corticosteroid, and hydroxychloroquine, an antimalarial agent, aren't indicated for heart failure. Lidocaine would be used only if the client also had ventricular ectopy.

A client experiences orthostatic hypotension while receiving furosemide (Lasix) to treat hypertension. How should the nurse intervene? 1. Administer I.V. fluids as ordered. 2. Administer a vasodilator as prescribed. 3. Insert an indwelling urinary catheter as ordered. 4. Instruct the client to sit up for several minutes before standing.

Correct Answer: 4 RATIONALES: To minimize the effects of orthostatic hypotension, the nurse should instruct the client to rise slowly to a standing position, such as by sitting up for several minutes first. Administering I.V. fluids would be inappropriate (unless the client were dehydrated) because it would counteract the effects of furosemide, possibly leading to fluid imbalance. Administering a vasodilator would further reduce the client's blood pressure, worsening orthostatic hypotension. Inserting an indwelling urinary catheter would aid urine output monitoring but wouldn't minimize the effects of orthostatic hypotension.

A client comes to the emergency department complaining of visual changes and a severe headache. The nurse measures the client's blood pressure at 210/120 mm Hg. However, the client denies having hypertension or any other disorder. After diagnosing malignant hypertension, a life-threatening disorder, the physician initiates emergency intervention. What is the most common cause of malignant hypertension? 1. Pyelonephritis 2. Dissecting aortic aneurysm 3. Pheochromocytoma 4. Untreated hypertension

Correct Answer: 4 RATIONALES: Untreated hypertension is the most common cause of malignant hypertension. Pyelonephritis, dissecting aortic aneurysm, and excessive catecholamine release (an effect of pheochromocytoma) are less common causes. Rarely, malignant hypertension results from eclampsia, ingestion of or exposure to drugs or toxic substances, and food and drug interactions (such as monoamine oxidase inhibitors with aged cheeses).

A septic client with hypotension is being treated with dopamine hydrochloride (Inotropin). The nurse asks a colleague to double-check the dosage that the client is receiving. There are 400 mg of dopamine hydrochloride in 250 ml, the infusion pump is running at 23 ml/hour, and the client weighs 79.5 kg. How many micrograms per kilogram per minute is the client receiving?

Correct Answer: 7.71 RATIONALES: First, calculate how many micrograms per milliliter of dopamine hydrochloride are in the bag: 400 mg/250 ml = 1.6 mg/ml Next, convert milligrams to micrograms: 1.6 mg/ml × 1,000 mcg/mg = 1,600 mcg/ml Lastly, calculate the dose: 1,600 mcg/ml × 23 ml/hour/79.5 kg 79.5 kg/60 minutes/hour = 7.71 mcg/kg/minute

The nurse is discussing cardiac hemodynamics with a nursing student. The nurse explains afterload to the student and then asks the student what nursing interventions might cause decreased afterload. The student correctly answers which of the following?

Correct response: Administration of a vasodilating drug (as ordered by a physician) Explanation: Afterload is the amount of resistance to the ejection of blood from the ventricles. Anything that decreases this resistance will decrease afterload. Vasodilation will decrease systemic resistance. Antiembolytic stockings and keeping the client's legs elevated will increase resistance.

The nurse is discussing cardiac hemodynamics with a nursing student. The nurse explains preload to the student and then asks the student what nursing interventions might cause increased preload. The student correctly answers which of the following?

Correct response: Application of antiembolytic stockings Explanation: Preload is the amount of blood presented to the ventricles just before systole. Anything that assists in returning blood to the heart (eg, antiembolytic stockings) or preventing blood from pooling in the extremities will increase preload. Anything that decreases the amount of blood returning to the heart will decrease preload, such as vasodilation or blood pooling in the extremities.

The nurse is performing a respiratory assessment for a patient in left-sided heart failure. What does the nurse understand is the best determinant of the patient's ventilation and oxygenation status?

Correct response: Arterial blood gases Explanation: In left-sided heart failure, arterial blood gases may be obtained to assess ventilation and oxygenation.

(see full question) The nurse understands that a patient with which cardiac arrhythmia is most at risk for developing heart failure?

Correct response: Atrial fibrillation Explanation: Cardiac dysrhythmias such as atrial fibrillation may either cause or result from HF; in both instances, the altered electrical stimulation impairs myocardial contraction and decreases the overall efficiency of myocardial function

The nurse understands that a patient with which cardiac arrhythmia is most at risk for developing heart failure?

Correct response: Atrial fibrillation Explanation: Cardiac dysrhythmias such as atrial fibrillation may either cause or result from HF; in both instances, the altered electrical stimulation impairs myocardial contraction and decreases the overall efficiency of myocardial function.

(see full question) A client with pulmonary edema has been admitted to the ICU. What would be the standard care for this client?

Correct response: BP and pulse measurements every 15 to 30 minutes Explanation: Bedside ECG monitoring is standard, as are continuous pulse oximetry, automatic BP, and pulse measurements approximately every 15 to 30 minutes.

Jacob Teale, a 71-year-old retired firefighter, has been diagnosed with CHF. He suffered an MI 3 years ago and his cardiac function has been compromised ever since. How is heart failure classified?

Correct response: By the amount of activity restriction the failure imposes Explanation: Chronic heart failure is classified based on the amount of activity restriction it imposes. Although organizations that develop the classifications may have varying stages, they are all based on the level of activity restriction.

The nurse is caring for a client in the hospital with chronic heart failure that has marked limitations in his physical activity. The client is comfortable when resting in the bed or chair, but when ambulating in the room or hall, he becomes short of breath and fatigued easily. What type of heart failure is this considered according to the New York Heart Association (NYHA)?

Correct response: Class III (Moderate) Explanation: Class III (Moderate) is when there is marked limitation of physical activity. The client is comfortable at rest, but less than ordinary activity causes fatigue, heart palpitations, or dyspnea. Class I is ordinary physical activity does not cause undue fatigue, palpitations, or dyspnea. The client does not experience any limitation of activity. Class II (Mild) is when the client is comfortable at rest, but ordinary physical activity results in fatigue, heart palpitations, or dyspnea. Class IV (Severe), the client is unable to carry out any physical activity without discomfort. Symptoms of cardiac insufficiency occur at rest. Discomfort is increased if any physical activity is undertaken.

Which of the following is a cerebrovascular manifestation of heart failure?

Correct response: Dizziness Explanation: Cerebrovascular manifestations of heart failure include dizziness and confusion. Tachycardia is a cardiovascular manifestation. Ascites is a GI manifestation. Nocturia is a renal manifestation

Which drug is most commonly used to treat cardiogenic shock?

Correct response: Dopamine (Intropin) Explanation: Dopamine, a sympathomimetic drug, is used to treat cardiogenic shock. It increases perfusion pressure to improve myocardial contractility and blood flow through vital organs. Enalapril is an angiotensin-converting enzyme inhibitor that directly lowers blood pressure. Furosemide is a diuretic and doesn't have a direct effect on contractility or tissue perfusion. Metoprolol is a beta-adrenergic blocker that slows heart rate and lowers blood pressure, undesirable effects when treating cardiogenic shock.

Frequently, what is the earliest symptom of left-sided heart failure?

Correct response: Dyspnea on exertion

A patient with congestive heart failure is admitted to the hospital with complaints of shortness of breath. How should the nurse position the patient in order to decrease preload?

Correct response: Head of the bed elevated at 45 degrees and lower arms supported by pillows Explanation: Preload is the amount of blood presented to the ventricle just before systole. The patient is positioned or taught how to assume a position that facilitates breathing. The number of pillows may be increased, the head of the bed may be elevated, or the patient may sit in a recliner. In these positions, the venous return to the heart (preload) is reduced, pulmonary congestion is alleviated, and pressure on the diaphragm is minimized. The lower arms are supported with pillows to eliminate the fatigue caused by the pull of the patient's weight on the shoulder muscles.

Sixty six-year-old David Steiner is already being treated for hypertension. His doctor, concerned about the potential for heart failure, has him come back for check-ups regularly. What does hypertension have to do with heart failure?

Correct response: Hypertension causes the heart's chambers to enlarge and weaken. Explanation: Hypertension causes the heart's chambers to enlarge and weaken, making it impossible for the ventricles to eject all the blood they receive.

Cardiogenic shock is pump failure that primarily occurs because of which of the following?

Correct response: Inadequate tissue perfusion Explanation: The classic signs of cardiogenic shock are related to tissue hypoperfusion and an overall state of shock that is proportional to the extent of left ventricular damage. Reduced cardiac output and stroke volume reduces arterial blood pressure and tissue perfusion.

A new client has been admitted with right-sided heart failure. The nurse knows to look for which of the following assessment findings when assessing this client?

Correct response: Jugular venous distention Explanation: When the right ventricle cannot effectively pump blood from the ventricle into the pulmonary artery, the blood backs up into the venous system and causes jugular venous distention and congestion in the peripheral tissues and viscera. All the other choices are symptoms of left-sided heart failure.

Which of the following types of diuretic is the first-line treatment for those diagnosed with heart failure (HF)?

Correct response: Loop Explanation: Loop diuretics such as furosemide, bumetanide, and torsemide are the preferred first-line diuretics because of their efficacy in patients with and without renal impairment. Diuretics should never be used alone to treat HF because they don't prevent further myocardial damage.

Which of the following is the hallmark of systolic heart failure?

Correct response: Low ejection fraction (EF) Explanation: A low EF is a hallmark of systolic heart failure (HF); the severity of HF is frequently classified according to the patient's symptoms.

Which of the following groups of patients are at risk for thrombophlebitis?

Correct response: Older adults Explanation: Older adults with heart and blood vessel diseases are susceptible to thrombophlebitis because of impaired mobility, reduced activity, and compromised circulation. Patients who have a history of allergy and frequent indigestion are not susceptible to thrombophlebitis. Athletes are not an at risk group.

You are working in a long-term care facility with a group of older adults with cardiac disorders. Why would it be important for you to closely monitor an older adult receiving digitalis preparations for cardiac disorders?

Correct response: Older adults are at increased risk for toxicity. Explanation: Older adults receiving digitalis preparations are at increased risk for toxicity because of the decreased ability of the kidneys to excrete the drug due to age-related changes. The margin between a therapeutic and toxic effect of digitalis preparations is narrow. Using digitalis preparations does not increase the risk of cardiac arrests, hyperthyroidism, or asthma.

Which of the following clinical manifestations are consistent with right-sided heart failure (HF)?

Correct response: Positive hepatojugular reflux Explanation: When right-sided HF occurs, there is increased venous pressure, which leads to jugular vein distention (JVD) and increased hydrostatic pressure throughout the venous system. Left-sided HF is characterized by pulmonary congestion from impaired left ventricle (LV) function, left ventricular insufficiency, and pulmonary interstitial edema.

A patient has been diagnosed with systolic heart failure. The nurse would expect the patient's ejection fraction to be at which level?

Correct response: Severely reduced Explanation: The ejection fraction is normal in diastolic heart failure, but severely reduced in systolic heart failure.

A client with heart failure has been receiving an intravenous infusion at 150 mL/hr. Now the client is short of breath. The nurse auscultates crackles bilaterally and notes neck vein distention and tachycardia. Using critical thinking skills, what should the nurse do first?

Correct response: Slow the infusion and notify the physician. Explanation: The client has fluid overload, so the nurse should first slow the infusion to prevent additional overload, and then notify the physician to obtain further orders. Notifying the physician without slowing the infusion would increase the client's risk. Discontinuing the infusion is not appropriate, because having a vascular access will be important. Administering a diuretic without turning down the intravenous infusion rate is counterproductive.

The triage nurse in the Emergency Department (ED) is admitting a client with a history of Class III heart failure. What symptoms would the nurse expect the client to exhibit?

Correct response: The client is comfortable at rest, but less than ordinary activity causes fatigue, heart palpitation, or dyspnea. Explanation: Class III (Moderate): There is marked limitation of physical activity. The client is comfortable at rest, but less than ordinary activity causes fatigue, heart palpitation, or dyspnea

What is the main difference between Class I and Class II heart failure as defined by the New York Heart Association (NYHA)?

Correct response: The level of physical activity each allows Explanation: This would be a sign of Moderate heart failure. Both Class I and Class II levels of heart failure are considered Mild under the New York Heart Association (NYHA) guidelines. The difference is that in Class II, the client is comfortable at rest, but ordinary physical activity results in fatigue, heart palpitations, or dyspnea, whereas in Class I, the client is comfortable both at rest and during ordinary physical activity. This is a sign of Severe heart failure. Both Class I and Class II levels of heart failure are considered Mild under the New York Heart Association (NYHA) guidelines. The difference is that in Class II, the client is comfortable at rest, but ordinary physical activity results in fatigue, heart palpitations, or dyspnea, whereas in Class I, the client is comfortable both at rest and during ordinary physical activity

Which treatment is the best therapy for a stable client with digoxin toxicity?

Correct response: Time and symptomatic treatment Explanation: Time is the best treatment for stable clients with digoxin toxicity. Their kidneys excrete the metabolites and symptomatic treatment alleviates the rhythm disturbances or nausea caused by the toxicity. Activated charcoal is effective only if the client has taken an overdose of cardiac glycosides and a large amount of unabsorbed drug remains in the GI tract before the serum level increases. Physicians reserve hemodialysis for clients who are extremely unstable despite symptomatic treatment or whose renal function isn't sufficient to excrete the drug. Physicians might use atropine to treat bradycardia that results from digoxin toxicity, but wouldn't necessarily use atropine to treat the toxicity itself.

The student nurse is caring for a client with heart failure. Diuretics have been ordered. What method might be used with a debilitated patient to help the nurse evaluate the client''s response to diuretics?

Correct response: Using a urinary catheter Explanation: To evaluate response to diuretics, a urinary catheter is used. Mechanical ventilation helps maintain a normal breathing pattern. A pulmonary artery catheter helps estimate cardiac output. A biventricular pacemaker is used to sustain life.

The student nurse is caring for a client with heart failure. Diuretics have been ordered. What method would help the nurse evaluate the client's response to diuretics?

Correct response: Using a urinary catheter Explanation: To evaluate response to diuretics, a urinary catheter is used. Mechanical ventilation helps maintain a normal breathing pattern. A pulmonary artery catheter helps estimate cardiac output. A biventricular pacemaker is used to sustain life.

Which of the following nursing interventions should a nurse perform when a patient with valvular disorder of the heart has a heart rate less than 60 beats per minute before administering beta-blockers?

Correct response: Withhold the drug and inform the primary health care provider Explanation: Before administering beta-blockers, the nurse should monitor the patient's apical pulse. If the heart rate is less than 60 beats per minute, the nurse should withhold the drug and inform the primary health care provider.

While auscultating the heart sounds of a client with heart failure, the nurse hears an extra heart sound immediately after the second heart sound (S2). The nurse should document this as:

Correct response: a third heart sound (S3). Explanation: An S3 is heard following an S2, which commonly occurs in clients experiencing heart failure and results from increased filling pressures. An S1 is a normal heart sound made by the closing of the mitral and tricuspid valves. An S4 is heard before an S1 and is caused by resistance to ventricular filling. A murmur is heard when there is turbulent blood flow across the valves.

The nurse is caring for an elderly client with left-sided heart failure. When auscultating lung sounds, which adventitious sound is expected?

Crackles

A 52-year-old female patient is going through menopause and asks the nurse about estrogen replacement for its cardioprotective benefits. What is the best response by the nurse?

Current evidence indicates that estrogen is ineffective as a cardioprotectant and is actually potentially harmful and is no longer a recommended therapy

For both outpatients and inpatients scheduled for diagnostic procedures of the cardiovascular system, the nurse performs a thorough initial assessment to establish accurate baseline data. Which of the following data is necessary to collect if the patient is experiencing chest pain?

Description of the pain If the patient is experiencing chest pain, a history of its location, frequency, and duration is necessary, as is a description of the pain, if it radiates to a particular area, what precipitates its onset, and what brings relief. The nurse weighs the patient and measures vital signs. The nurse may measure BP in both arms and compare findings. The nurse assesses apical and radial pulses, noting rate, quality, and rhythm. The nurse also checks peripheral pulses in the lower extremities

The nurse caring for a client who is suspected of having cardiovascular disease has a stress test ordered. The client has a comorbidity of multiple sclerosis, so the nurse knows the stress test will be drug induced. What drug will be used to dilate the coronary arteries

Dobutrex

The nurse prepares to auscultate heart sounds. Which nursing interventions would be most effective to assist with this procedure?

Explain to the client that the nurse will be listening to different areas of the chest and may listen for a long time, but that does not mean that anything abnormal is heard.

he nurse is reviewing the results of the patient's echocardiogram and observes that the ejection fraction is 35%. The nurse anticipates that the patient will receive treatment for what condition?

Heart failure

During assessment of a 63-year-old retired mechanic, the nurse notes and documents an S3 heart sound. The nurse knows that this sound is an abnormal sound suggestive of

Heart failure The S3 heart sound is heard immediately after the S2 sound, early in diastole, as blood flows from the atrium into a noncompliant ventricle. Normal in children and young adults, it is a significant finding suggestive of heart failure in older adults

The nurse is caring for a patient in the ED who has a B-type natriuretic peptide (BNP) level of 115 pg/mL. The nurse understands that this finding is most suggestive of which of the following? a) Pulmonary edema b) Heart failure c) Myocardial infarction d) Ventricular hypertrophy

Heart failure A BNP level greater than 100 pg/mL is suggestive of HF. Because this serum laboratory test can be quickly obtained, BNP levels are useful for prompt diagnosis of HF in settings such as the ED. Elevations in BNP can occur from a number of other conditions such as pulmonary embolus, myocardial infarction (MI), and ventricular hypertrophy. Therefore, the clinician correlates BNP levels with abnormal physical assessment findings and other diagnostic tests before making a definitive diagnosis of HF.

During the auscultation of a patient's heart sounds, the nurse notes an S4. The nurse recognizes that an S4 is associated with which of the following? a) Turbulent blood flow b) Hypertensive heart disease c) Heart failure d) Diseased heart valves

Hypertensive heart disease Auscultation of the heart requires familiarization with normal and abnormal heart sounds. An extra sound just before S1 is an S4 heart sound, or atrial gallop. An S4 sound often is associated with hypertensive heart disease. A sound that follows S1 and S2 is called an S3 heart sound or a ventricular gallop. An S3 heart sound is often an indication of heart failure in an adult. In addition to heart sounds, auscultation may reveal other abnormal sounds, such as murmurs and clicks, caused by turbulent blood flow through diseased heart valves

The critical care nurse is caring for clients in an emergency department. When caring for a variety of clients, when is the presence of a third heart sound normal?

In pediatric clients When caring for a variety of clients, it is important to consider that a third heart sound is normal in children. In adults, a third heart sound may signify heart failure. There is no correlation between third heart sounds with heart valve replacement and an indwelling pacemaker

You are discharging a client after a cardiac catheterization. What would you include in your discharge teaching

Keep the extremity straight for several hours and avoid movement -report any warm, wet feeling that may indicate oozing blood, numbness, tingling, or sharp pain in the extremity -drink a large volume of fluid to relieve thirst and promote the excretion of the dye.

The nurse is caring for a client with ECG changes consistent with a myocardial infarction. Which of the following diagnostic test does the nurse anticipate to confirm heart damage?

Nuclear cardiology Nuclear cardiology uses a radionuclide to detect areas of myocardial damage. Chest radiography and fluoroscopy determine the size and position of the heart and condition of the lungs. Serum blood work notes elevations in enzymes suggesting muscle damage.

Two days after discharge following a permanent pacemaker insertion, the client returns to the clinic for a follow-up appointment. He reports tenderness and throbbing around the incision. The nurse observes mild swelling, erythema, and warmth at the site and suspects which of the following: a) Internal bleeding at pacemaker site b) Postoperative site hematoma c) Normal postoperative healing d) Pacemaker site infection

Pacemaker site infection Postoperative care for a pacemaker insertion includes observing for symptoms of infection. These include swelling, unusual tenderness, drainage, and increased warmth.

Two days after discharge following a permanent pacemaker insertion, the client returns to the clinic for a follow-up appointment. He reports tenderness and throbbing around the incision. The nurse observes mild swelling, erythema, and warmth at the site and suspects which of the following: a) Postoperative site hematoma b) Normal postoperative healing c) Internal bleeding at pacemaker site d) Pacemaker site infection

Pacemaker site infection Postoperative care for a pacemaker insertion includes observing for symptoms of infection. These include swelling, unusual tenderness, drainage, and increased warmth.

The nurse is educating a patient at risk for atherosclerosis. What nonmodifiable risk factor does the nurse identify for the patient?

Positive family history The health history provides an opportunity for the nurse to assess patients' understanding of their personal risk factors for coronary artery, peripheral vascular, and cerebrovascular diseases (see Chart 27-1 in Chapter 27) and any measures that they are taking to modify these risks. Risk factors are classified by the extent to which they can be modified by changing one's lifestyle or modifying personal behaviors. Stress, obesity, and hyperlipidemia are all risk factors that can be modified by personal behaviors. Family history is a nonmodifiable risk factor, because it cannot be changed.

The healthcare team is caring for a client with early atherosclerotic changes within the blood vessels. The physician orders diagnostic testing, and the nurse provides cardiovascular teaching. Which primary goal is the healthcare team working toward

Preserving the natural heart by avoiding further heart disease Monitoring cardiovascular status and current health needs

A nurse is caring for a client who is on a continuous cardiac monitor. When evaluating the client's rhythm strip, the nurse notes that the QRS interval has increased from 0.08 second to 0.14 second. Based on this finding, the nurse should withhold continued administration of which drug? a) Procainamide (Pronestyl) b) Metoprolol (Lopressor) c) Propafenone (Rythmol) d) Verapamil (Calan)

Procainamide (Pronestyl) Procainamide may cause an increased QRS complexes and QT intervals. If the QRS duration increases by more than 50%, then the nurse should withhold the drug and notify the physician of her finding. Metoprolol may cause increased PR interval and bradycardia. Propafenone and verapamil may cause bradycardia and atrioventricular blocks.

Which of the following arteries carries deoxygenated blood?

Pulmonary artery

A 97-year-old client with a history of atrial fibrillation is being admitted to the assisted living center where you practice nursing. In your initial assessment, you measure his apical pulse and compare it to his peripheral pulse. The difference between the two is known as what?

Pulse deficit

Which of the following nursing interventions is most appropriate when caring for a client with a nursing diagnosis of risk for injury related to side effects of medication (enoxaparin [Lovenox])?

Report any incident of bloody urine, stools, or both.

The nurse is assessing heart sounds in a patient with heart failure. An abnormal heart sound is detected early in diastole. How would the nurse document this?

S3 An S3 ("DUB") is heard early in diastole during the period of rapid ventricular filling as blood flows from the atrium into a noncompliant ventricle. It is heard immediately after S2. "Lub-dub-DUB" is used to imitate the abnormal sound of a beating heart when an S3 is present.

The electrical conduction system of the heart has several components, all of which are instrumental in maintaining polarization, depolarization, and repolarization of cardiac tissue. Which of the conductive structures is known as the pacemaker of the heart?

SA node

The nurse uses which term for the normal pacemaker of the heart?

Sinoatrial (SA) node

The client is admitted for a scheduled cardiac catheterization. On the morning of the procedure, while assessing the client's morning laboratory values, the nurse notes a blood urea nitrogen (BUN) of 34 mg/dL and a creatinine of 4.2 mg/dL. The nurse makes it a priority to notify the physician for which of the following reasons?

The client is at risk for renal failure due to the contrast agent that will be given during the procedure

D

The nurse is performing an assessment of the patient's heart. Where would the nurse locate the apical pulse if the heart is in normal position? A. left 2nd intercostal space at the mid-clavicular line B. right 2nd intercostal space at the mid-clavicular line C. right 3rd intercostal space at the mid-clavicular line D. left 5th intercostal space at the mid-clavicular line

Avoid tub baths, but shower as desired.

The nurse is providing discharge education for the client going home after a cardiac catheterization. Which of the following would be important information to give this client?

Chest pain, weight gain, fatigue.

The nurse is taking a health history from a client admitted with the medical diagnosis of cardiovascular disease (CVD). Identify which of the following symptoms indicate CVD.

Sinoatrial (SA) node, atrioventricular (AV) node, bundle of His, right and left bundle branches, and the Purkinje fibers

The nurse is teaching a beginning EKG class to staff nurses. As the nurse begins to discuss the the parts of the EKG complex, one of the students asks what the normal order of conduction through the heart is. The correct response would be which of the following?

B

The nurse observes a CNA obtaining a BP with a cuff that is too small for the patient. The nurse informs the CNA that using a cuff that is too small can affect the reading results in what way? A. the result will be falsely decreased B. the results will be falsely elevated C. it will give an accurate reading D. it will be significantly different with each reading

The nurse is caring for a patient with a diagnosis of pericarditis. Where does the nurse understand the inflammation is located?

The thin fibrous sac encasing the heart

You are caring for a client with right-sided heart failure. When assessing the respiratory rate of this client, what is an indication that the client is having difficulty breathing

The use of accessory muscles neck or abdominal during respiration is an indication that the client is having difficultly breathing

What is the primary underlying disorder of pulmonary edema?

You selected: Decreased left ventricular pumping Correct Explanation: Pulmonary edema is an acute event that results from heart failure. Myocardial scarring, resulting from ischemia, limits the distensibility of the ventricle, making it vulnerable to demands for increased workload. When the demand on the heart increases, there is resistance to left ventricular filling and blood backs up into the pulmonary circulation. Pulmonary edema quickly develops

Which of the following is the hallmark of heart failure?

You selected: Low ejection fraction (EF) Correct Explanation: Although a low EF is a hallmark of heart failure (HF), the severity of HF is frequently classified according to the patient's symptoms. Pulmonary congestion, limitation of ADLs, and basilar crackles are all symptoms of HF.

The patient with cardiac failure is taught to report which of the following symptoms to the physician or clinic immediately?

You selected: Persistent cough Correct Explanation: Persistent cough may indicate an onset of left-sided heart failure. Loss of appetite should be reported immediately. Weight gain should be reported immediately. Frequent urination, causing interruption of sleep, should be reported immediately.

A client has a myocardial infarction in the left ventricle and develops crackles bilaterally; 3-pillow orthopnea; an S3 heart sound; and a cough with pink, frothy sputum. The nurse obtains a pulse oximetry reading of 88%. What do these signs and symptoms indicate for this client?

You selected: The development of left-sided heart failure Correct Explanation: When the left ventricle fails, the heart muscle cannot contract forcefully enough to expel blood into the systemic circulation. Blood subsequently becomes congested in the left ventricle, left atrium, and finally the pulmonary vasculature. Symptoms of left-sided failure include fatigue; paroxysmal nocturnal dyspnea; orthopnea; hypoxia; crackles; cyanosis; S3 heart sound; cough with pink, frothy sputum; and elevated pulmonary capillary wedge pressure. COPD develops over many years and does not develop after a myocardial infarction. The development of right-sided heart failure would generally occur after a right ventricle myocardial infarction or after the development of left-sided heart failure. Corpulmonale is a condition in which the heart is affected secondarily by lung damage

The cardiologist has scheduled a client for drug-induced stress testing. What instructions should the nurse provide to prepare the client for this test?

You will receive medication via IV administration

Elective electrical cardioversion

Your client has been diagnosed with an atrial dysrhythmia. The client has come to the clinic for a follow-up appointment and to talk with the physician about options to stop this dysrhythmia. What would be a procedure used to treat this client?

Thallium-201

Your client is going to have a stress test. What radionuclide would most likely be used to diagnose ischemic heart disease during this test?

A patient has undergone a cardiac catheterization. He is to be discharged today. What information should the nurse emphasize during discharge teaching? a) Avoid heavy lifting for the next 24 hours. b) Take a tub bath, rather than a shower. c) Bend only at the waist. d) New bruising at the puncture site is normal.

a) Avoid heavy lifting for the next 24 hours. For the next 24 hours, the patient should not bend at the waist, strain, or life heavy objects. The patient should avoid tub baths, but shower as desired. The patient should call her the health care provider if she has any bleeding, swelling, new bruising, or pain from her procedure puncture site, or a temperature of 101.5 degrees Fahrenheit or more.

The nurse accompanies a client to an exercise stress test. The client can achieve the "target heart rate," but the ECG leads show an ST-segment elevation. The nurse recognizes this as a "positive" stress test, and will begin to prepare the client for which of the following procedures? a) Cardiac catheterization b) Transesophageal echocardiogram c) Pharmacologic stress test d) Telemetry monitoring

a) Cardiac catheterization An elevated ST-segment means an evolving myocardial infarction. A cardiac catheterization would be the logical next step.

The ability of the cardiac muscle to shorten in response to an electrical impulse is termed which of the following? a) Contractility b) Depolarization c) Diastole d) Repolarization

a) Contractility Explanation: Contractility is the ability of the cardiac muscle to shorten in response to an electrical impulse. Depolarization is the electrical activation of a cell caused by the influx of sodium into the cell while potassium exits the cell. Repolarization is the return of the cell to the resting state, caused by reentry of potassium into the cell while sodium exits the cell. Diastole is the period of ventricular relaxation resulting in ventricular filling.

The nurse is assessing the client newly prescribed Lasix 20mg daily for 3+ pitting edema. To evaluate the effectiveness of diuretic therapy, which of the following would be documented? a) Edema b) Blood pressure c) Urine output d) Weight

a) Edema The best method to evaluate the effectiveness of diuretic therapy is to note a decrease in edema. Weight, blood pressure, and urine output all are affected by diuretic therapy, but the therapeutic goal is to decrease the edema.

Identify which of the following as an age-related change associated with conduction system of the heart? a) Heart block b) Murmur c) Thrills d) Tachycardia

a) Heart block Age-related changes to the conduction system may include bradycardia and heart block. Age-related changes to the heart valves include the presence of a murmur or thrill.

The nurse is interviewing a client who is complaining of chest pain. Which of the following questions related to the client's history are most important to ask? Select all that apply. a) How would you describe your symptoms? b) Do you have any children? c) How did your mother die? d) Are you allergic to any medications or foods?

a) How would you describe your symptoms? c) How did your mother die? d) Are you allergic to any medications or foods? During initial assessment, the nurse should obtain important information about the client's history that focuses on a description of the symptoms before and during admission, family medical history, prescription and nonprescription drug use, and drug and food allergies.

The nurse is caring for a patient in the ICU who is being monitored with a central venous pressure (CVP) catheter. The nurse records the patient's CVP as 8 mm Hg. The nurse understands that this finding indicates the patient is experiencing which of the following? a) Hypervolemia b) Excessive blood loss c) Overdiuresis d) Left-sided heart failure (HF)

a) Hypervolemia The normal CVP is 2 to 6 mm Hg. A CVP greater than 6 mm Hg indicates an elevated right ventricular preload. Many problems can cause an elevated CVP, but the most common is hypervolemia (excessive fluid circulating in the body) or right-sided HF. In contrast, a low CVP (<2 mm Hg) indicates reduced right ventricular preload, which is most often from hypovolemia.

Age-related changes associated with the cardiac system include which of the following? Select all that apply. a) Increased size of the left atrium b) Myocardial thinning c) Endocardial fibrosis d) Increase in the number of SA node cells

a) Increased size of the left atrium c) Endocardial fibrosis Age-related changes associated with the cardiac system include endocardial fibrosis, increased size of the left atrium, decreased number of SA node cells, and myocardial thickening.

The nurse is caring for a client on the cardiac unit. Which change of condition may indicate potential increasing of right-side heart failure? Select all that apply. a) Increased weakness on ambulation b) Jugular vein distention c) Edema changed from a 3+ to a 1+ d) One-pound weight loss e) Increased palpitations f) Increased dyspnea

a) Increased weakness on ambulation b) Jugular vein distention e) Increased palpitations f) Increased dyspnea A change in assessment finding may indicate an increase in heart failure. Right-sided heart failure symptoms include jugular vein distention, increased dyspnea, increased palpitations, and an increased weakness on ambulation. Edema is a common sign of right-sided heart failure, but changing from a 3+ to 1+ is improvement in condition. Weight loss is also improvement in condition.

A nurse is performing a cardiac assessment on an elderly client. Which finding warrants further investigation? a) Irregularly irregular heart rate b) Increased PR interval c) Fourth heart sound (S4) d) Orthostatic hypotension

a) Irregularly irregular heart rate An irregularly irregular heart rate indicates atrial fibrillation and should be investigated further. It's normal for an elderly client to have a prolonged systole, which causes an S4 heart sound. It's also normal for an elderly client to have slowed conduction, causing an increased PR interval. As a person ages, it's normal for baroreceptors in the body to decrease their response to changes in body position, which can cause orthostatic hypotension.

A nurse is preparing to assess a patient for postural BP changes. Which of the following indicates the need for further education? a) Positioning the patient supine for 10 minutes prior to taking the initial BP and HR b) Letting 30 seconds elapse after each position change before measuring BP and heart rate (HR) c) Obtaining the supine measurements prior to the sitting and standing measurements d) Taking the patient's BP with the patient sitting on the edge of the bed with feet dangling

a) Positioning the patient supine for 10 minutes prior to taking the initial BP and HR The following steps are recommended when assessing patients for postural hypotension: Position the patient supine for 10 minutes before taking the initial BP and HR measurements; reposition the patient to a sitting position with legs in the dependent position, wait 2 minutes then reassess both BP and HR measurements; if the patient is symptom free or has no significant decreases in systolic or diastolic BP, assist the patient into a standing position, obtain measurements immediately and recheck in 2 minutes; continue measurements every 2 minutes for a total of 10 minutes to rule out postural hypotension. Return the patient to supine position if postural hypotension is detected or if the patient becomes symptomatic. Document HR and BP measured in each position (e.g., supine, sitting, and standing) and any signs or symptoms that accompany the postural changes.

The nurse is screening a patient prior to a magnetic resonance angiogram (MRA) of the heart. Which of the following actions should the nurse complete prior to the patient undergoing the procedure? Select all that apply. a) Remove the patient's jewelry. b) Offer the patient a headset to listen to music during the procedure. c) Remove the patient's Transderm Nitro patch. d) Sedate the patient prior to the procedure. e) Position the patient on his/her stomach for the procedure.

a) Remove the patient's jewelry. b) Offer the patient a headset to listen to music during the procedure. c) Remove the patient's Transderm Nitro patch. Transdermal patches that contain a heat-conducting aluminized layer (e.g., NicoDerm, Androderm, Transderm Nitro, Transderm Scop, Catapres-TTS) must be removed before MRA to prevent burning of the skin. A patient who is claustrophobic may need to receive a mild sedative before undergoing an MRA. During an MRA, the patient is positioned supine on a table that is placed into an enclosed imager or tube containing the magnetic field. Patients are instructed to remove any jewelry, watches, or other metal items (e.g., ECG leads). An intermittent clanking or thumping that can be annoying is generated by the magnetic coils, so the patient may be offered a headset to listen to music.

The nurse practitioner inspects the patient's skin during a physical examination. She is looking for any abnormalities, especially skin findings associated with cardiovascular disease. The nurse notes a bluish tinge in the buccal mucosa and the tongue. She knows this is probably due to: a) Intermittent arteriolar vasoconstriction. b) Congenital heart disease. c) Peripheral vasoconstriction. d) Blood leaking outside the blood vessels.

b) Congenital heart disease. Cyanosis is due to serious cardiac disorders. A bluish tinge in the tongue and buccal mucosa are signs of central cyanosis caused by venous blood passing through the pulmonary circulation without being oxygenated. In the absence of pulmonary edema and cardiogenic shock, this sign is indicative of congenital heart disease. Refer to Table 12-3 in the text.

A nurse is caring for a client with a central venous pressure (CVP) of 4 mm Hg. Which nursing intervention is appropriate? a) Call the physician and obtain an order for a fluid bolus. b) Continue to monitor the client as ordered. c) Call the physician and obtain an order for a diuretic. d) Rezero the equipment and take another reading.

b) Continue to monitor the client as ordered. Normal CVP ranges from 3 to 7 mm Hg. The nurse doesn't need to take any action other than to monitor the client. It isn't necessary to rezero the equipment. Calling a physician and obtaining an order for a fluid bolus would be an appropriate intervention if the client has a CVP less than 3 mm Hg. Administering a diuretic would be appropriate if the client had excess fluid, as demonstrated by a CVP greater than 7 mm Hg.

A nurse is completing a head to toe assessment on a patient diagnosed with right-sided heart failure. To assess peripheral edema, which of the following areas should be examined? a) Legs, Toes b) Fingers, hands c) Under the sacrum d) Lips, earlobes

b) Fingers, hands When right-sided heart failure occurs, blood accumulates in the vessels and backs up in peripheral veins, and the extra fluid enters the tissues. Particular areas for examination are the dependent parts of the body, such as the feet and ankles. Other prominent areas prone to edema are the fingers, hands, and over the sacrum. Cyanosis can be detected by noting color changes in the lips and earlobes.

A patient is admitted to a hospital to confirm a diagnosis of postural hypotension. The nurse assesses the patient's blood pressure every 4 hours. Which of the following is an indicator of postural hypotension that occurs when the patient moves from a lying to a sitting or standing position? a) Heart rate decreased from 85 to 75 bpm at the same time that the systolic pressure increased from 120 to 135 mm Hg. b) Heart rate increased from 85 to 110 bpm. c) Diastolic pressure went from 80 to 110 mm Hg. d)Systolic pressure did not change with the change in position

b) Heart rate increased from 85 to 110 bpm.

The nurse is providing discharge instructions to a client with unstable angina. The client is ordered Nitrostat 1/150 every 5 minutes as needed for angina. Which side effect, emphasized by the nurse, is common especially with the increased dosage? a) Rash b) Orthostatic hypotension c) Dry mouth d) Nausea

b) Orthostatic hypotension A common side effect of Nitrostat, especially at higher dosages, is orthostatic hypotension. The action of the medication is to dilate the blood vessels to improve circulation to the heart. The side effect of the medication is orthostatic hypotension. A rash, nausea, and dry mouth are not common side effects.

A 24-year-old obese woman describes her symptoms of palpitations, chronic fatigue, and dyspnea on exertion to the cardiologist. Upon completing the examination, the cardiologist schedules a procedure to confirm his suspected diagnosis. What diagnostic would you expect him to prescribe? a) Radionuclide angiography b) Transesophageal echocardiography c) Electrocardiography d) Chest radiograph

b) Transesophageal echocardiography TEE involves passing a tube with a small transducer internally from the mouth to the esophagus to obtain images of the posterior heart and its internal structures from the esophagus, which lies behind the heart. TEE provides superior views that are not possible using standard transthoracic echocardiography. Clients whose chests are rotund or who are obese are candidates for TEE. Chest radiography and fluoroscopy determine the size and position of the heart and condition of the lungs. The radionuclide technetium-99m is used to detect areas of myocardial damage. The radionuclide thallium-201 is used to diagnose ischemic heart disease during a stress test. Electrocardiography (ECG) is the graphic recording of the electrical currents generated by the heart muscle.

The nurse is caring for a geriatric client. The client is ordered Lanoxin (digoxin) tablets 0.125mg daily for a cardiac dysrhythmias. Which of the following assessment considerations is essential when caring for this age-group? a)Activity level b)Digoxin level c)Dyspnea d) Cardiac output

b)Digoxin level

A patient has undergone a cardiac catheterization. He is to be discharged today. What information should the nurse emphasize during discharge teaching? a) New bruising at the puncture site is normal. b) Avoid heavy lifting for the next 24 hours. c) Bend only at the waist. d) Take a tub bath, rather than a shower.

b. Avoid heavy lifting for the next 24 hours. Explanation: For the next 24 hours, the patient should not bend at the waist, strain, or life heavy objects. The patient should avoid tub baths, but shower as desired. The patient should call her the health care provider if she has any bleeding, swelling, new bruising, or pain from her procedure puncture site, or a temperature of 101.5 degrees Fahrenheit or more.

The nurse is caring for a client with an elevated blood pressure and no previous history of hypertension. At 0900, the blood pressure was 158/90 mm Hg. At 0930, the blood pressure is 142/82 mm Hg. The nurse is most correct when relating the fall in blood pressure to which structure? a) Sympathetic nerve fibers b) Vagus nerve c) Baroreceptors d) Chemoreceptors

c) Baroreceptors Baroreceptor sense pressure in nerve endings in the walls of the atria and major blood vessels. The baroreceptors respond accordingly to raise or lower the pressure. Chemoreceptors are sensitive to pH, CO2, and O2 in the blood. Sympathetic nerve fibers increase the heart rate. The vagus nerve slows the heart rate.

A nurse working in a cardiac step-down unit understands that the following drugs can affect the contractility of the heart. The nurse recognizes that contractility is depressed by which of the following drugs? a) Lanoxin b) Dobutrex c) Lopressor d) Intropin

c) Lopressor Contractility is depressed by beta-adrenergic blocking medications. The other choices all enhance contractility.

After a physical examination, the provider diagnosed a patient with a grade 4 heart murmur. During assessment, the nurse expects to hear a murmur that is: a) Quiet but readily heard. b) Easily heard with no palpable thrill. c) Loud and may be associated with a thrill sound similar to (a purring cat). d) Very loud; can be heard with the stethoscope half-way off the chest.

c) Loud and may be associated with a thrill sound similar to (a purring cat). Heart murmurs are characterized by location, timing, and intensity. A grading system is used to describe the intensity or loudness of a murmur. A grade 1 is very faint and difficult to describe, whereas a grade 6 is extremely loud. Refer to Box 12-3 in the text for a description of grades 1 to 6.

When preparing a patient for a cardiac catheterization, the patient states that she has allergies to seafood. Which of the following medications may give to her prior to the procedure? a) Furosemide (Lasix) b) Phenytoin (Dilantin) c) Methylprednisolone (Solu-Medrol) d) Lorazepam (Ativan)

c) Methylprednisolone (Solu-Medrol)

It is important for a nurse to understand cardiac hemodynamics. For blood to flow from the right ventricle to the pulmonary artery, the following must occur: a) The atrioventricular valves must open. b) Right ventricular pressure must decrease with systole. c) Right ventricular pressure must be higher than pulmonary arterial pressure. d) The pulmonic valve must be closed.

c) Right ventricular pressure must be higher than pulmonary arterial pressure.

A nurse is caring for a dying client following myocardial infarction. The client is experiencing apnea with a falling blood pressure of 60 per palpation. Which documentation of pulse quality does the nurse anticipate? a) A pulse deficit b) Weak pulse c) Thready pulse d) Bounding pulse

c) Thready pulse The nurse is most correct to anticipate a thready (barely palpable) pulse quality. A bounding pulse indicates a strong cardiac output. A weak pulse indicates a lower pulse quality. A pulse deficit occurs when the pulses between the apex of the heart differs from the radial pulse.

The nurse is awaiting results of cardiac biomarkers for a patient with severe chest pain. The nurse would identify which cardiac biomarker as remaining elevated the longest when myocardial damage has occurred? a) CK-MB b) Brain natriuretic peptide (BNP) c) Troponin T and I d) Myoglobin

c) Troponin T and I After myocardial injury, these biomarkers rise early (within 3 to 4 hours), peak in 4 to 24 hours, and remain elevated for 1 to 3 weeks. These early and prolonged elevations may make very early diagnosis of acute myocardial infarction (MI) possible and allow for late diagnosis in patients who have delayed seeking care for several days after the onset of acute MI symptoms. CK-MB returns to normal within 3 to 4 days. Myoglobin returns to normal within 24 hours. BNP is not considered a cardiac biomarker. It is a neurohormone that responds to volume overload in the heart by acting as a diuretic and vasodilator.

The nurse notes that the client has had a change in mental status. Why would it be important for the nurse to report extremes in the thought process of a client with cardiovascular disorder to the physician

confusion or disorientation can result from a decrease in the oxygen supply to the brain as a result of poor circulation

The health care provider documents that the patient's pulse quality is a +1 on a scale of 0 to 4. The nurse knows that this describes a pulse that is: a) Full, easy to palpate, and cannot be obliterated. b) Diminished, but cannot be obliterated. c) Strong and bounding and may be abnormal. d) Difficult to palpate and is obliterated with pressure.

d) Difficult to palpate and is obliterated with pressure. The quality of pulses is reported using descriptors and a scale of 0 to 4. The lower the number, the weaker the pulse and the easier it is to obliterate it. A +1 pulse is weak and thready and easily obliterated with pressure.

Which of the following tests used to diagnose heart disease is least invasive? a) Cardiac catheterization b) Magnetic resonance imaging c) Coronary arteriography d) Transthoracic echocardiography

d) Transthoracic echocardiography Transthoracic echocardiography uses high-frequency sound waves that pass through the chest wall (transthoracic) and are displayed on an oscilloscope. MRI uses magnetism to identify disorders that affect many different structures in the body without performing surgery. While an MRI does not expose clients to radiation, it does require intravenous infusion to instill medication and contrast medium. Cardiac catheterization requires the insertion of a long, flexible catheter from a peripheral blood vessel in the groin, arm, or neck into one of the great vessels and then into the heart. This procedure requires the instillation of a contrast medium into each coronary artery.

The nurse admits a 52-year-old woman with a medical diagnosis of "rule out MI." The client is very frightened and expresses surprise that a woman would have heart problems. Which of the following responses by the nurse would be most appropriate? a) "It takes longer for an electrical impulse to travel from the sinoatrial node to the atrioventricular node." b) "The stroke volume from a woman's heart is lower than from a man's heart." c) "A woman's resting heart rate is lower than a man's." d) "A woman's heart is smaller and has smaller arteries that become occluded more easily."

d. "A woman's heart is smaller and has smaller arteries that become occluded more easily." Explanation: Because the coronary arteries of a woman are smaller, they become occluded from atherosclerosis more easily. In addition, the resting rate, stroke volume, and ejection fraction of a woman's heart are higher than those of a man.

For both outpatients and inpatients scheduled for diagnostic procedures of the cardiovascular system, the nurse performs a thorough initial assessment to establish accurate baseline data. Which of the following data is necessary to collect if the patient is experiencing chest pain? a) Sound of the apical pulses b) Pulse rate in upper extremities c) Blood pressure in the left arm d) Description of the pain

d. Description of the pain Explanation: If the patient is experiencing chest pain, a history of its location, frequency, and duration is necessary, as is a description of the pain, if it radiates to a particular area, what precipitates its onset, and what brings relief. The nurse weighs the patient and measures vital signs. The nurse may measure BP in both arms and compare findings. The nurse assesses apical and radial pulses, noting rate, quality, and rhythm. The nurse also checks peripheral pulses in the lower extremities.

The nurse is caring for a geriatric client. The client is ordered Lanoxin (digoxin) tablets 0.125mg daily for a cardiac dysrhythmias. Which of the following assessment considerations is essential when caring for this age-group? a) Dyspnea b) Cardiac output c) Activity level d) Digoxin level

d. Digoxin level Explanation: The action of Digoxin slows and strengthens the heart rate. Assessment of the pulse rate is essential prior to administration in all clients. Due to decreased perfusion common in geriatric clients, toxicity may occur more often. The nurse must monitor Digoxin levels in the body. Monitoring symptoms reflecting cardiac output, activity level, and dyspnea are also important assessment considerations for all clients.

The nurse caring for a client who is suspected of having cardiovascular disease has a stress test ordered. The client has a co-morbidity of multiple sclerosis, so the nurse knows the stress test will be drug-induced. What drug will be used to dilate the coronary arteries? a) Thallium b) Ativan c) Diazepam d) Dobutamine

d. Dobutamine Explanation: Drugs such as adenosine (Adenocard), dipyridamole (Persantine), or dobutamine (Dobutrex) may be administered singularly or in combination by the IV route. The drugs dilate the coronary arteries, similar to the vasodilation that occurs when a person exercises to increase the heart muscle's blood supply. Options A, B, and C would not dilate the coronary arteries.

The nurse auscultates the apex beat at which of the following anatomical locations? a) 2 inch to the left of the lower end of the sternum b) Midsternum c) 1 inch to the left of the xiphoid process d) Fifth intercostal space, midclavicular line

d. Fifth intercostal space, midclavicular line Explanation: The left ventricle is responsible for the apex beat or the point of maximum impulse, which is normally palpable in the left midclavicular line of the chest wall at the fifth intercostal space. The right ventricle lies anteriorly, just beneath the sternum. Use of inches to identify the location of the apex beat is inappropriate based upon variations in human anatomy. Auscultation below and to the left of the xiphoid process will detect gastrointestinal sounds, but not the apex beat of the heart.

The nurse is reviewing the results of the patient's echocardiogram and observes that the ejection fraction is 35%. The nurse anticipates that the patient will receive treatment for what condition? a) Pericarditis b) Myocardial infarction c) Pulmonary embolism d) Heart failure

d. Heart failure Explanation: An ejection fraction of less than 40% indicates that the patient has decreased left ventricular function and likely requires treatment for heart failure.

The nurse is interviewing a client who is complaining of chest pain. Which of the following questions related to the client's history are most important to ask? Select all that apply. a) Are you allergic to any medications or foods? b) How did your mother die? c) How would you describe your symptoms? d) Do you have any children?

• How would you describe your symptoms? • Are you allergic to any medications or foods? • How did your mother die? During initial assessment, the nurse should obtain important information about the client's history that focuses on a description of the symptoms before and during admission, family medical history, prescription and nonprescription drug use, and drug and food allergies.

A 76-year-old client has a significant history of congestive heart failure. During his semiannual cardiology examination, for what should you, as his nurse, specifically assess? Select all that apply.

• Monitor the client for signs of lethargy or confusion. • Examine the client's neck for distended veins. Correct Explanation: During a head-to-toe assessment of a client with congestive heart failure, the nurse checks for dyspnea, auscultates apical heart rate and counts radial heart rate, measures BP, and documents any signs of peripheral edema, lethargy, or confusion. Excess tears are not part of the checklist.

A client in the cardiac unit is undergoing procedures to determine the extent of his left-sided heart failure. As his nurse, what adventitious lung sounds would you expect to hear during your auscultation of his lungs to support his diagnosis? Select all that apply

• Wheezes • Wet lung sounds With left-sided heart failure, auscultation reveals a crackling sound and possibly wheezes and gurgles. Wet lung sounds are accompanied by dyspnea and an effort to sit up to breath

You are doing an admission assessment on a client who is having outpatient testing done for cardiac problems. What should you ask this client during your assessment?

"Have you had any episodes of dizziness or fainting?"

The nurse is assessing the client's cardiovascular system. The client asks the nurse why she presses on his toenails. Which is the best reply by the nurse?

"I can see how quickly the blood returns when I press and release your toenail. This tells me how well your peripheral blood is flowing."

A client with a forceful, pounding heartbeat is diagnosed with mitral valve prolapse. Which client statement indicates to the nurse a need for additional teaching? a) "I can still drink coffee and tea." b) "I should eat foods rich in protein." c) "I should increase my fluid intake." d) "I'll enroll in an aerobic exercise program."

"I can still drink coffee and tea." The client requires more teaching if he states that he may drink coffee and tea. Caffeine is a stimulant, which can exacerbate palpitations, and should be avoided by a client with symptomatic mitral valve prolapse. High fluid intake helps maintain adequate preload and cardiac output. Aerobic exercise helps increase cardiac output and decrease heart rate. Protein-rich foods aren't restricted but high-calorie foods are.

A client presents to the emergency department via ambulance with a heart rate of 210 beats/minute and a sawtooth waveform pattern per cardiac monitor. The nurse is most correct to alert the medical team of the presence of a client with which disorder? a) Ventricular fibrillation b) Atrial flutter c) Premature ventricular contraction d) Asystole

Atrial flutter Atrial flutter is a disorder in which a single atrial impulse outside the SA node causes the atria to contract at an exceedingly rapid rate. The atrioventricular (AV) node conducts only some impulses to the ventricle, resulting in a ventricular rate slower than the atrial rate, thus forming a sawtooth pattern on the heart monitor. Asystole is the absence of cardiac function and can indicate death. Premature ventricular contraction indicates an early electric impulse and does not necessarily produce an exceedingly rapid heart rate. Ventricular fibrillation is the inefficient quivering of the ventricles and indicative of a dying heart.

The defibrillator won't deliver a shock if the synchronizer switch is turned on.

Before using a defibrillator to terminate ventricular fibrillation, a nurse should check the synchronizer switch. Why is this check so important?

A nurse is providing morning care for a patient in the ICU. Suddenly, the bedside monitor shows ventricular fibrillation and the patient becomes unresponsive. After calling for assistance, what action should the nurse take next? a) Administer intravenous epinephrine. b) Provide electrical cardioversion. c) Prepare for endotracheal intubation. d) Begin cardiopulmonary resuscitation.

Begin cardiopulmonary resuscitation. In the acute care setting, when ventricular fibrillation is noted, the nurse should call for assistance and defibrillate the patient as soon as possible. If defibrillation is not readily available, CPR is begun until the patient can be defibrillated, followed by advanced cardiovascular life support (ACLS) intervention, which includes endotracheal intubation and administration of epinephrine. Electrical cardioversion is not indicated for a patient in ventricular fibrillation.

Where is the epigastric area located?

Below the xiphoid process

Erectile dysfunction may be a side effect of what heart medication

Beta blockers

A nurse assessing a client who underwent cardiac catheterization finds the client lying flat on the bed. His temperature is 99.8° F (37.7° C). His blood pressure is 104/68 mm Hg. His pulse rate is 76 beats/minute. She detects weak pulses in the leg distal to the puncture site. Skin on the leg is cool to the touch. The puncture site is dry, but swollen. What is the most appropriate action for the nurse to take?

Contact the physician and report her findings

CRP is used to help predict the risk of

Coronary artery disease

The nurse administers basic cardiac life support to a client in cardiac arrest. Which action does the nurse perform? 1. Assessing the patency of the airway 2. Administering I.V. medications 3. Administering a countershock of 200 joules 4. Breathing for the client after inserting an endotracheal (ET) tube

Correct Answer: 1 RATIONALES: A nurse certified in basic cardiac life support can assess airway patency. I.V. medications given to maintain blood pressure, correct acidosis, or restore a cardiac rhythm are administered by a provider of advanced cardiac life support. Administering a countershock of 200 joules and breathing for the client after inserting an ET tube are measures carried out during advanced life support.

Which measurement can best be used to monitor the respiratory status of a client with pulmonary edema? 1. Arterial blood gas (ABG) analysis 2. Pulse oximetry 3. Skin color assessment 4. Lung sounds

Correct Answer: 1 RATIONALES: ABG analysis is the best measure for determining the extent of hypoxia caused by pulmonary edema and for monitoring the effects of therapy. Although any of the options can be used to detect pulmonary changes, assessment of skin color and assessment of lung fields often are subject to interpretation by practitioners. The use of pulse oximetry is unreliable, especially in the case of severe vasoconstriction as is present in pulmonary edema.

The most common site of aneurysm formation is in the: 1. abdominal aorta, just below the renal arteries. 2. ascending aorta, around the aortic arch. 3. descending aorta, beyond the subclavian arteries. 4. aortic arch, around the ascending and descending aorta.

Correct Answer: 1 RATIONALES: About 75% of aneurysms occur in the abdominal aorta, just below the renal arteries (Debakey type I aneurysms). Options 2 and 4 are characteristic of Debakey type II aneurysms, whereas option 3 is indicative of Debakey type III aneurysms

The physician prescribes several drugs for a client with hemorrhagic stroke. Which drug order should the nurse question? 1. heparin sodium (Heparin sodium injection) 2. dexamethasone (Decadron) 3. methyldopa (Aldomet) 4. phenytoin (Dilantin)

Correct Answer: 1 RATIONALES: Administration of heparin, an anticoagulant, could increase the bleeding associated with hemorrhagic stroke. Therefore, the nurse should question this order to prevent additional hemorrhage in the brain. In a client with hemorrhagic stroke, dexamethasone may be used to decrease cerebral edema and pressure; methyldopa, to reduce blood pressure; and phenytoin, to prevent seizures.

A client with a myocardial infarction (MI) develops pulmonary crackles and dyspnea. A chest X-ray shows evidence of pulmonary edema. The specific type of MI the client had is most probably: 1. anterior. 2. posterior. 3. lateral. 4. inferior.

Correct Answer: 1 RATIONALES: An anterior MI causes left ventricular dysfunction and can lead to manifestations of heart failure, which include pulmonary crackles and dyspnea. The other types of MI aren't usually associated with heart failure

A client is evaluated for hypertension. The physician prescribes atenolol (Tenormin), 50 mg P.O. daily. Atenolol should have which therapeutic effect on the client? 1. Decreased cardiac output and systolic and diastolic blood pressure 2. Decreased blood pressure with reflex tachycardia 3. Increased cardiac output and systolic and diastolic blood pressure 4. Decreased peripheral vascular resistance

Correct Answer: 1 RATIONALES: As a long-acting, selective beta1 blocker, atenolol decreases cardiac output and systolic and diastolic blood pressure; however, like other beta-adrenergic blocking agents, it increases peripheral vascular resistance at rest and with exercise. Atenolol may cause bradycardia, not tachycardia.

The telemetry unit nurse is caring for a client who was just transferred from the coronary care unit (CCU). The client is anxious because he fears he won't be monitored as closely as he was in the CCU. How can the nurse allay his fears? 1. Assign the same nurse to the client when possible. 2. Allow the client uninterrupted periods of rest as much as possible. 3. Limit the client's visitors to coincide with CCU visiting policy. 4. Move the client to a room far from the nurses' station to reduce his exposure to noise.

Correct Answer: 1 RATIONALES: Assigning the same nurse to the client when possible provides continuity of care and stability, thereby reducing his anxiety. The client needs uninterrupted periods of rest; however, providing as much rest as possible may leave the client feeling isolated. Feelings of isolation can increase the client's anxiety, and having visitors can help distract the client. A room close to nurses' station would provide this client with a sense of security because the nurses are close by in the event of an emergency.

When caring for a client with rheumatic fever, the nurse formulates a nursing diagnosis of Activity intolerance related to reduced cardiac reserve and enforced bed rest. Before the nurse can eliminate this nursing diagnosis, the client must meet which outcome measurement criterion? 1. "Erythrocyte sedimentation rate returns to normal." 2. "Pulse doesn't rise above 150 beats/minute with activity." 3. "Temperature remains normal with salicylate administration." 4. "Pericardial friction rub is diminishing in intensity."

Correct Answer: 1 RATIONALES: Bed rest must continue until the client's erythrocyte sedimentation rate returns to normal, indicating resolution of rheumatic fever; the resting pulse is under 100 beats/minute; the client can maintain a normal temperature without salicylates; and a pericardial friction rub disappears completely. If the client needs salicylates to maintain a normal temperature, inflammation still is present and activity may cause serious myocardial damage. Resuming activity before a pericardial friction rub disappears completely may impair cardiac function.

A client develops heart failure. The physician prescribes inamrinone lactate (Inocor), 0.75 mg/kg I.V. over 3 minutes followed by 5 mcg/kg/minute with continuous I.V. infusion. Which laboratory test results should the nurse obtain before starting inamrinone therapy? 1. Platelet count and liver enzyme levels 2. Hemoglobin levels and hematocrit 3. Creatine kinase (CK) level 4. White blood cell (WBC) count

Correct Answer: 1 RATIONALES: Before starting inamrinone therapy, the nurse should determine the client's baseline platelet count and liver enzyme levels because inamrinone may cause thrombocytopenia and liver enzyme alterations. The drug isn't known to cause anemia or affect the CK level or WBC count.

A client reports recent onset of chest pain that occurs sporadically with exertion. The client also has fatigue and mild ankle swelling, which is most pronounced at the end of the day. The nurse suspects a cardiovascular disorder. When exploring the chief complaint, the nurse should find out if the client has any other common cardiovascular symptoms, such as: 1. shortness of breath. 2. insomnia. 3. irritability. 4. lower substernal abdominal pain.

Correct Answer: 1 RATIONALES: Common signs and symptoms of cardiovascular dysfunction include shortness of breath, chest pain, dyspnea, palpitations, fainting, fatigue, and peripheral edema. Insomnia seldom indicates a cardiovascular problem. Although irritability may occur if cardiovascular dysfunction leads to cerebral oxygen deprivation, this symptom more commonly reflects a respiratory or neurologic dysfunction. Lower substernal abdominal pain occurs with some GI disorders

For a client with cardiomyopathy, the most important nursing diagnosis is: 1. Decreased cardiac output related to reduced myocardial contractility. 2. Excess fluid volume related to fluid retention and altered compensatory mechanisms. 3. Ineffective coping related to fear of debilitating illness. 4. Anxiety related to actual threat to health status.

Correct Answer: 1 RATIONALES: Decreased cardiac output related to reduced myocardial contractility is the greatest threat to the survival of a client with cardiomyopathy. Although the other options are important nursing diagnoses, they can be addressed when cardiac output and myocardial contractility have been restored.

An obese white male client, age 49, is diagnosed with hypercholesterolemia. The physician prescribes a low-fat, low cholesterol, low-calorie diet to reduce blood lipid levels and promote weight loss. This diet is crucial to the client's wellbeing because his race, sex, and age increase his risk for coronary artery disease (CAD). To determine if the client has other major risk factors for CAD, the nurse should assess for: 1. a history of diabetes mellitus. 2. elevated high-density lipoprotein (HDL) levels. 3. a history of ischemic heart disease. 4. alcoholism.

Correct Answer: 1 RATIONALES: Diabetes mellitus, smoking, and hypertension are other major risk factors for CAD. Elevated HDL levels aren't a risk factor for CAD; in fact, increased HDL levels seem to protect against CAD. Ischemic heart disease is another term for CAD, not a risk factor. Alcoholism hasn't been identified as a major risk factor for CAD.

A client with chronic heart failure is receiving digoxin (Lanoxin), 0.25 mg by mouth (P.O.) daily, and furosemide (Lasix), 20 mg P.O. twice daily. The nurse instructs the client to notify the physician if nausea, vomiting, diarrhea, or abdominal cramps occur because these signs and symptoms may signal digitalis toxicity. Digitalis toxicity also may cause: 1. visual disturbances. 2. taste and smell alterations. 3. dry mouth and urine retention. 4. nocturia and sleep disturbances.

Correct Answer: 1 RATIONALES: Digitalis toxicity may cause visual disturbances (such as flickering light flashes, colored or halo vision, photophobia, blurring, diplopia, and scotomata), central nervous system abnormalities (such as headache, fatigue, lethargy, depression, irritability and, if profound, seizures, delusions, hallucinations, and memory loss), and cardiovascular abnormalities (abnormal heart rate and arrhythmias). Taste and smell alterations aren't associated with digitalis toxicity. Dry mouth and urine retention typically occur with anticholinergic agents, not inotropic agents such as digoxin. Nocturia and sleep disturbances are adverse effects of furosemide — especially if the client takes the second daily dose in the evening, which may cause diuresis at night.

When administering dobutamine (Dobutrex), the nurse knows that its major clinical use is to: 1. increase cardiac output. 2. prevent sinus bradycardia. 3. treat hypotension. 4. treat hypertension.

Correct Answer: 1 RATIONALES: Dobutamine increases cardiac output for clients with acute heart failure and those undergoing cardiopulmonary bypass surgery. Epinephrine hydrochloride, another catecholamine agent, may be used to treat sinus bradycardia. Many of the catecholamine agents, including epinephrine, isoproterenol, and norepinephrine, may be used to treat acute hypotension. None of the catecholamine agents are used to treat hypertension because many of them raise blood pressure as part of their action.

During digoxin (Lanoxin) therapy, the nurse should closely monitor the client's: 1. serum potassium and magnesium levels. 2. urine glucose and ketones. 3. serum potassium and creatine kinase (CK) levels. 4. urine potassium and CK levels.

Correct Answer: 1 RATIONALES: During digoxin therapy, the nurse should closely monitor the client's serum potassium and magnesium levels. This is because hypokalemia or hypomagnesemia can predispose the client to digitalis toxicity. Glucose and ketones aren't usually in the urine except in a client with uncontrolled diabetes, and digoxin isn't known to affect these levels in the diabetic. CK levels may be elevated if digoxin is administered I.M., but this route of administration isn't recommended.

A client is recovering from coronary artery bypass graft (CABG) surgery. Which nursing diagnosis takes highest priority at this time? 1. Decreased cardiac output related to depressed myocardial function, fluid volume deficit, or impaired electrical conduction 2. Anxiety related to an actual threat to health status, invasive procedures, and pain 3. Disabled family coping related to knowledge deficit and a temporary change in family dynamics 4. Hypothermia related to exposure to cold temperatures and a long cardiopulmonary bypass time

Correct Answer: 1 RATIONALES: For a client recovering from CABG surgery, Decreased cardiac output is the most important nursing diagnosis because myocardial function may be depressed from anesthetics or a long cardiopulmonary bypass time, leading to decreased cardiac output. Other possible causes of decreased cardiac output in this client include fluid volume deficit and impaired electrical conduction. The other options may be relevant but take lower priority at this time because maintaining cardiac output is essential to sustaining the client's life.

A client with chest pain doesn't respond to nitroglycerin. On admission to the emergency department, the health care team obtains an electrocardiogram and begins infusing I.V. morphine. The physician also considers administering alteplase (Activase). This thrombolytic agent must be administered how soon after onset of myocardial infarction (MI) symptoms? 1. Within 12 hours 2. Within 24 hours 3. Within 24 to 48 hours 4. Within 5 to 7 days

Correct Answer: 1 RATIONALES: For the best chance of salvaging the client's myocardium, a thrombolytic agent must be administered within 12 hours after onset of chest pain or other signs or symptoms of MI. Within the first 24 hours after an MI, sudden death is most likely to occur. I.V. heparin therapy begins after administration of a thrombolytic agent and usually continues for 5 to 7 days.

A client is recovering from coronary artery bypass graft (CABG) surgery. The nurse knows that for several weeks after this procedure, the client is at risk for certain conditions. During discharge preparation, the nurse should advise the client and family to expect which common symptom that typically resolves spontaneously? 1. Depression 2. Ankle edema 3. Memory lapses 4. Dizziness

Correct Answer: 1 RATIONALES: For the first few weeks after CABG surgery, clients commonly experience depression, fatigue, incisional chest discomfort, dyspnea, and anorexia. Depression typically resolves on its own and doesn't require medical intervention; however, family members should be aware that symptoms don't always resolve on their own. They should also be instructed about worsening symptoms of depression and when to seek care. Ankle edema seldom follows CABG surgery and may indicate right-sided heart failure; because this condition is a sign of cardiac dysfunction, the client should report ankle edema at once. Memory lapses reflect neurologic rather than cardiac dysfunction. Dizziness may result from decreased cardiac output, an abnormal condition after CABG surgery that warrants immediate physician notification.

The nurse is caring for a client who's experiencing sinus bradycardia with a pulse rate of 40 beats/minute. His blood pressure is 80/50 mm Hg and he complains of dizziness. Which medication would be used to treat his bradycardia? 1. Atropine 2. Dobutamine (Dobutrex) 3. Amiodarone (Cordarone) 4. Lidocaine (Xylocaine)

Correct Answer: 1 RATIONALES: I.V. push atropine is used to treat symptomatic bradycardia. Dobutamine is used to treat heart failure and low cardiac output. Amiodarone is used to treat ventricular fibrillation and unstable ventricular tachycardia. Lidocaine is used to treat ventricular ectopy, ventricular tachycardia, and ventricular fibrillation.

A client, age 59, complains of leg pain brought on by walking several blocks — a symptom that first arose several weeks ago. The client's history includes diabetes mellitus and a two-pack-a-day cigarette habit for the past 42 years. The physician diagnoses intermittent claudication and prescribes pentoxifylline (Trental), 400 mg three times daily with meals. The nurse should provide which instruction concerning long-term care? 1. "Practice meticulous foot care." 2. "Consider cutting down on your smoking." 3. "Reduce your level of exercise." 4. "See the physician if complications occur."

Correct Answer: 1 RATIONALES: Intermittent claudication and other chronic peripheral vascular diseases reduce oxygenation to the feet, making them susceptible to injury and poor healing. Therefore, meticulous foot care is essential. The nurse should teach the client to bathe the feet in warm water and dry them thoroughly, cut the toenails straight across, wear well-fitting shoes, and avoid taking medications unless the physician approves. Because nicotine is a vasoconstrictor, this client should stop smoking, not just consider cutting down. Daily walking is beneficial to clients with intermittent claudication. The client must see the physician regularly to evaluate the effectiveness of the therapeutic regimen, not just when complications occur.

Within hours after suffering a massive myocardial infarction, a client experiences cardiogenic shock. All vital functions are being monitored closely; an intra-arterial catheter has been inserted to detect changes in arterial blood pressure. Which statement comparing intra-arterial and cuff blood pressure readings is accurate? 1. Intra-arterial readings should be at least 10 mm Hg higher than cuff readings. 2. Intra-arterial readings should be at least 10 mm Hg lower than cuff readings. 3. Cuff readings are easier to obtain than intra-arterial readings. 4. Cuff readings detect excessive peripheral vasoconstriction more accurately than intra-arterial readings.

Correct Answer: 1 RATIONALES: Intra-arterial blood pressure readings should be at least 10 mm Hg higher than cuff readings. Placement of an indwelling catheter for intra-arterial monitoring allows continuous recording of arterial pressure, eliminating the need to locate the client's brachial pulse and place a stethoscope on the arm for each reading. This makes intra-arterial readings easier, not harder, to obtain than cuff readings. Intra-arterial pressure monitoring can detect blood pressure in clients with excessive peripheral vasoconstriction, low cardiac output, and fluctuating hemodynamic status — even when cuff measurements can't.

A client has a heart rate of 170 beats/minute. The physician diagnoses ventricular tachycardia and orders lidocaine hydrochloride (Xylocaine), an initial I.V. bolus of 50 mg followed in 5 minutes by a second 50-mg bolus, then continuous I.V. infusion at 2 mg/minute. The nurse can expect the client to begin experiencing an antiarrhythmic effect within: 1. 1 to 2 minutes after I.V. bolus administration. 2. 1 to 2 minutes after continuous I.V. infusion. 3. 10 to 15 minutes after I.V. bolus administration. 4. 10 to 15 minutes after continuous I.V. infusion.

Correct Answer: 1 RATIONALES: Lidocaine exerts its antiarrhythmic effect in 1 to 2 minutes after I.V. bolus administration. A continuous I.V. infusion will maintain lidocaine's antiarrhythmic effect for as long as the drip is used. Lidocaine provides antiarrhythmic effects for only 15 minutes after the I.V. infusion is stopped.

Following a myocardial infarction, a client develops an arrhythmia and requires a continuous infusion of lidocaine. To monitor the effectiveness of the intervention, the nurse should focus primarily on the client's: 1. electrocardiogram (ECG). 2. urine output. 3. creatine kinase (CK) and troponin levels. 4. blood pressure and heart rate.

Correct Answer: 1 RATIONALES: Lidocaine is an antiarrhythmic and is given for the treatment of cardiac irritability and ventricular arrhythmias. The best indicator of its effectiveness is a reduction in or disappearance of ventricular arrhythmias as seen on an ECG. Urine output is an indicator of pump effectiveness; CK and troponin levels monitor myocardial damage. Blood pressure and heart rate measurements are too nonspecific to help determine the effectiveness of parenteral lidocaine.

After a myocardial infarction, a client develops a complication requiring a continuous infusion of lidocaine. To monitor the effectiveness of this infusion, the nurse should focus primarily on: 1. electrocardiogram (ECG). 2. urine output. 3. creatine kinase (CK) and troponin levels. 4. blood pressure and heart rate.

Correct Answer: 1 RATIONALES: Lidocaine is an antiarrhythmic given to treat cardiac irritability and ventricular arrhythmias. The best indicator of its effectiveness is the reduction or disappearance of ventricular arrhythmias as seen on the ECG. Urine output is an indicator of pump effectiveness. CK and troponin levels monitor myocardial damage. Blood pressure and heart rate are too nonspecific to be indicators of lidocaine's effectiveness.

A client with known coronary artery disease reports intermittent chest pain, usually on exertion. The physician diagnoses angina pectoris and prescribes sublingual nitroglycerin to treat acute angina episodes. When teaching the client about nitroglycerin administration, the nurse should provide which instruction? 1. "Be sure to take safety precautions because nitroglycerin may cause orthostatic hypotension." 2. "Replace leftover sublingual nitroglycerin tablets every 6 months to make sure they're fresh." 3. "A burning sensation after administration indicates that the nitroglycerin tablets are potent." 4. "You may take a sublingual nitroglycerin tablet every 30 minutes, if needed, to a maximum of four doses."

Correct Answer: 1 RATIONALES: Nitroglycerin commonly causes orthostatic hypotension and dizziness. To minimize these problems, the nurse should teach the client to take safety precautions, such as changing to an upright position slowly, climbing up and down stairs carefully, and lying down at the first sign of dizziness. To ensure the freshness of sublingual nitroglycerin, the client should replace tablets every 3 months, not every 6 months, and store them in a tightly closed container in a cool, dark place. Many brands of sublingual nitroglycerin no longer produce a burning sensation. The client should take a sublingual nitroglycerin tablet at the first sign of angina and may repeat the dose every 10 to 15 minutes for up to three doses; if this doesn't bring relief, the client should seek immediate medical attention

A client with hypertension visits the health clinic for a routine checkup. The nurse measures the client's blood pressure at 184/92 mm Hg and notes a 5-lb weight gain over the past month. Which nursing diagnosis reflects the most serious problem in managing a client with hypertension? 1. Noncompliance (nonadherence to therapeutic regimen) 2. Deficient knowledge (disease process) 3. Excess fluid volume 4. Imbalanced nutrition: More than body requirements

Correct Answer: 1 RATIONALES: Noncompliance is the most serious problem in managing a client with hypertension. One authority estimates that 40% to 60% of hypertensive clients fail to comply with prescribed treatment. Reasons for noncompliance include lack of symptoms, which makes the problem seem less serious; the difficulty of making required lifestyle changes, such as eating a low-sodium diet, stopping smoking, and losing or managing weight; adverse reactions to antihypertensive drugs; and the inconvenience and high cost of obtaining health care. The other options may promote or result from noncompliance. Deficient knowledge contributes to noncompliance; Excess fluid volume, caused by excess sodium intake, and Imbalanced nutrition: More than body requirements may result from noncompliance.

Which sign or symptom suggest that a client's abdominal aortic aneurysm is extending? 1. Increased abdominal and back pain 2. Decreased pulse rate and blood pressure 3. Retrosternal back pain radiating to the left arm 4. Elevated blood pressure and rapid respirations

Correct Answer: 1 RATIONALES: Pain in the abdomen and back signify that the aneurysm is pressing downward on the lumbar nerve root and is causing more pain. The pulse rate would increase with aneurysm extension. Chest pain radiating down the arm would indicate myocardial infarction. Blood pressure would decrease with aneurysm extension, and the respiratory rate may not be affected.

A client with mitral valve prolapse is advised to have elective mitral valve replacement. Because the client is a Jehovah's Witness, she declares in her advance directive that no blood products are to be administered. As a result, the consulting cardiac surgeon refuses to care for the client. It would be most appropriate for the nurse caring for the client to: 1. realize the surgeon has the right to refuse to care for the client. 2. advise the surgeon to arrange for an alternate cardiac surgeon. 3. tell the client that she can donate her own blood for the procedure. 4. inform the client that her decision could shorten her life.

Correct Answer: 1 RATIONALES: Physicians have an ethical and legal right to refuse to care for any client in a nonemergency situation when standard medical care isn't acceptable to the client. It isn't the responsibility of the surgeon to find an alternate. Jehovah's Witnesses don't believe in receiving blood transfusions. Informing the client that her decision can shorten her life is inappropriate in that the statement may be inaccurate and it ignores the client's right of autonomy.

A client with chronic heart failure is examined in the outpatient department to investigate recent onset of peripheral edema and increased shortness of breath. Physical findings include bilateral crackles, a third heart sound (S3), distended neck veins, elevated blood pressure, and pitting edema of the ankles. The nurse documents the severity of pitting edema as +1. What is the best description of this type of edema? 1. Barely detectable depression when the thumb is released from the swollen area; normal foot and leg contours 2. Detectable depression of less than 5 mm when the thumb is released from the swollen area; normal foot and leg contours 3. A 5- to 10-mm depression when the thumb is released from the swollen area; foot and leg swelling 4. A depression of more than 1 cm when the thumb is released from the swollen area; severe foot and leg swelling

Correct Answer: 1 RATIONALES: Pitting edema is documented as a +1 when a depression is barely detectable on release of thumb pressure and when foot and leg contours are normal. A detectable depression of less than 5 mm accompanied by normal leg and foot contours warrants a +2 rating. A deeper depression (5 to 10 mm) accompanied by foot and leg swelling is evaluated as +3. An even deeper depression (more than 1 cm) accompanied by severe foot and leg swelling rates a +4.

In presenting a workshop on parameters of cardiac function, which conditions should the nurse list as those most likely to lead to a decrease in preload? 1. Hemorrhage, sepsis, and anaphylaxis 2. Myocardial infarction, fluid overload, and diuresis 3. Fluid overload, sepsis, and vasodilation 4. Third spacing, heart failure, and diuresis

Correct Answer: 1 RATIONALES: Preload is the volume in the left ventricle at the end of diastole. It's also referred to as end-diastolic volume. Preload is reduced by any condition that reduces circulating volume, such as hemorrhage, sepsis, and anaphylaxis. Hemorrhage reduces circulating volume by loss of volume from the intravascular space. Sepsis and anaphylaxis reduce circulating volume by increased capillary permeability. Diuresis, vasodilation, and third spacing also reduce preload. Preload would increase with fluid overload and heart failure.

The nurse should be prepared to manage complications after abdominal aortic aneurysm resection. Which complication is most common postoperatively? 1. Renal failure 2. Hemorrhage and shock 3. Graft occlusion 4. Enteric fistula

Correct Answer: 1 RATIONALES: Renal failure commonly occurs if clamping time is prolonged, cutting off the blood supply to the kidneys. Hemorrhage and shock are the most common complications before abdominal aortic aneurysm resection, and they occur if the aneurysm leaks or ruptures. Graft occlusion and enteric fistula formation are rare complications of abdominal aortic aneurysm repair.

A client seeks medical attention for dyspnea, chest pain, syncope, fatigue, and palpitations. A thorough physical examination reveals an apical systolic thrill and heave, along with a fourth heart sound (S4) and a systolic murmur. Diagnostic tests reveal that the client has hypertrophic cardiomyopathy (HCM). Which nursing diagnosis may be appropriate? 1. Risk for injury 2. Risk for deficient fluid volume 3. Ineffective thermoregulation 4. Risk for peripheral neurovascular dysfunction

Correct Answer: 1 RATIONALES: Risk for injury is an appropriate nursing diagnosis for a client with HCM because physical exertion may cause syncope or sudden death. Heart failure may complicate HCM, causing fluid volume excess; therefore, the nursing diagnosis of Risk for deficient fluid volume isn't applicable. Ineffective thermoregulation and Risk for peripheral neurovascular dysfunction are inappropriate because HCM doesn't cause these problems.

The nurse is assessing a client's right lower leg, which is wrapped with an elastic (Ace) bandage. Which signs and symptoms suggest circulatory impairment? 1. Numbness, cool skin temperature, and pallor 2. Swelling, warm skin temperature, and drainage 3. Numbness, warm skin temperature, and redness 4. Redness, cool skin temperature, and swelling

Correct Answer: 1 RATIONALES: Signs and symptoms of impaired circulation include numbness and cool, pale skin. Signs of localized infection may include swelling, drainage, redness, and warm skin. Signs of adequate circulation include warm skin with normal return of skin color after blanching and normal sensation.

Two female nursing assistants approach the nurse on a cardiac step-down unit to report that a 45-year-old male client who experienced an acute myocardial infarction (MI) made sexual comments to them. How should the nurse intervene? 1. The nurse should explain that the client might have concerns about resuming sexual activity but is afraid to ask. 2. The nurse should report the incident to her supervisor immediately. 3. The nurse should instruct the nursing assistants to avoid answering his call light. 4. The nurse should explain that the client most likely wants extra attention.

Correct Answer: 1 RATIONALES: Sometimes clients are concerned about resuming sexual activity but are afraid to ask. Making inappropriate sexual comments provides a forum for asking questions. It's not necessary to report the incident to the nursing supervisor immediately without investigating the situation further. The client's call light must be answered in a timely fashion. More information is needed before assuming that the client is asking for extra attention.

On a routine visit to the physician, a client with chronic arterial occlusive disease reports stopping smoking after 34 years. To relieve symptoms of intermittent claudication, a condition associated with chronic arterial occlusive disease, the nurse should recommend which additional measure? 1. Taking daily walks 2. Engaging in anaerobic exercise 3. Reducing daily fat intake to less than 45% of total calories 4. Avoiding foods that increase levels of high-density lipoproteins (HDLs)

Correct Answer: 1 RATIONALES: Taking daily walks relieves symptoms of intermittent claudication, although the exact mechanism is unclear. Anaerobic exercise may make these symptoms worse. Clients with chronic arterial occlusive disease must reduce daily fat intake to 30% or less of total calories. The client should limit dietary cholesterol because hyperlipidemia is associated with atherosclerosis, a known cause of arterial occlusive disease. However, HDLs have the lowest cholesterol concentration, so this client should eat, not avoid, foods that raise HDL levels.

The nurse is evaluating a client who had a myocardial infarction (MI) 7 days ago. Which outcome indicates that the client is responding favorably to therapy? 1. The client demonstrates the ability to tolerate increasing activity without chest pain. 2. The client exhibits a heart rate above 100 beats/minute. 3. The client verbalizes the intention of making all necessary lifestyle changes except for stopping smoking. 4. The client states that sublingual nitroglycerin usually relieves chest pain.

Correct Answer: 1 RATIONALES: The ability to tolerate increasing activity without chest pain indicates a favorable response to therapy in a client who is recovering from an MI or who has a history of coronary artery disease. The client should have a normal electrocardiogram with no arrhythmias and a regular heart rate of 60 to 100 beats/minute. Smoking is a cardiovascular risk factor that the client must be willing to eliminate. A client who responds favorably to therapy shouldn't have chest pain.

Which signs and symptoms are present with a diagnosis of pericarditis? 1. Fever, chest discomfort, and elevated erythrocyte sedimentation rate (ESR) 2. Low urine output secondary to left ventricular dysfunction 3. Lethargy, anorexia, and heart failure 4. Pitting edema, chest discomfort, and nonspecific ST-segment elevation

Correct Answer: 1 RATIONALES: The classic signs and symptoms of pericarditis include fever, positional chest discomfort, nonspecific STsegment elevation, elevated ESR, and pericardial friction rub. All other symptoms may result from acute renal failure.

The nurse is preparing to begin one-person cardiopulmonary resuscitation. The nurse should first: 1. establish unresponsiveness. 2. call for help. 3. open the airway. 4. assess the client for a carotid pulse.

Correct Answer: 1 RATIONALES: The correct sequence begins with establishing unresponsiveness. The nurse should then call for help, assess the client for breathing while opening the airway, deliver two breaths, and check for a carotid pulse.

Before discharge, which instruction should the nurse give to a client receiving flecainide (Tambocor) to reduce the risk of heart failure? 1. "Limit your fluid intake." 2. "Take a diuretic before going to bed." 3. "Limit your potassium intake." 4. "Have your serum electrolyte levels measured weekly."

Correct Answer: 1 RATIONALES: The nurse should tell the client receiving flecainide to limit fluid intake. The client shouldn't take a diuretic unless prescribed by the physician; if prescribed, the diuretic should be taken early in the day to prevent nocturia. Sodium (not potassium) should be limited because excessive sodium intake causes water retention. The client's electrolyte levels don't need to be measured weekly.

The nurse is caring for a cardiac client who requires various cardiac medications. When the nurse helps the client out of bed for breakfast, the client becomes dizzy and asks to lie down. The nurse helps the client lie down, puts up the side rails, and obtains the client's blood pressure, which is 84/50 mm Hg. It's time for the nurse to administer the client's medications: nitroglycerin, metoprolol (Lopressor), and furosemide (Lasix). Which action is best taken by the nurse? 1. Withhold the medications and notify the physician. 2. Administer the medications immediately. 3. Encourage the client to sit up and eat breakfast. 4. Administer the nitroglycerin and metoprolol and withhold the furosemide.

Correct Answer: 1 RATIONALES: The nurse should withhold the three medications and notify the physician. Each of these medications has the potential to lower the client's blood pressure. Administering them together when the client is already hypotensive may severely lower the client's blood pressure. The client may continue to experience dizziness when sitting up so breakfast should be held until his blood pressure stabilizes.

After receiving shift report, the registered nurse in the cardiac step-down unit, must prioritize her client care assignment. She has an ancillary staff member available to help her care for her clients. Which of these clients should the registered nurse assess first? 1. The client with heart failure who is having some difficulty breathing. 2. The anxious client who was diagnosed with an acute myocardial infarction (MI) two days ago and who was transferred from the coronary care unit today. 3. The demanding client who underwent coronary bypass surgery three days ago. 4. The client admitted during the previous shift with new-onset controlled atrial fibrillation who has her call light on.

Correct Answer: 1 RATIONALES: The registered nurse should care for the client with heart failure who is experiencing difficulty breathing. Breathing takes precedence over the other client needs. The ancillary staff member can answer the call light of the client admitted with controlled atrial fibrillation. She can also attend to the demanding client who underwent coronary bypass surgery three days ago. Although anxiety can be detrimental to a client with an MI, anxiety doesn't take precedence over another client's breathing difficulty.

The staff nurse is caring for a client who is a potential heart donor. The client's family is concerned that the recipient will have access to personal donor information. Which response by the nurse demonstrates knowledge of the organ donation process? 1. "I will have the transplant coordinator speak with you to answer your questions." 2. "There is never contact between the donor's family and the recipient." 3. "The recipient is allowed to ask questions about the donor and have them answered." 4. "It is important that the recipient know where to send Thank-You cards."

Correct Answer: 1 RATIONALES: The transplant coordinator, a specially trained person with knowledge of the donation, procurement, and transplantation process, typically speaks to family members and answers their questions. Contact is permitted after the procedure with consent from the donor's family and the recipient. Typically, the transplant organization coordinates the communication. Confidentiality of the potential donor is always maintained unless the recipient and donor families both sign confidentiality waivers.

The unit council in the telemetry unit is responsible for performance improvement studies. What information should they gather to study whether client education about resuming sexual activity after an acute myocardial infarction (MI) is being taught? 1. The percentage of clients on the unit diagnosed with an acute MI who were taught about resuming sexual activity 2. The quality of teaching by the nurses who educate the acute MI clients on the telemetry unit 3. The amount of education the acute MI clients received on the telemetry unit 4. The nurses' assessment of the quality of client education about resuming sexual activity after an acute MI

Correct Answer: 1 RATIONALES: The unit council needs to assess the number of clients diagnosed with an acute MI on the telemetry unit who were actually taught about resuming sexual activity. The unit council needs to identify the number of clients who were taught, not the quality of the teaching. Only education about resuming sexual activity is pertinent to this performance improvement study. The nurses' assessment of the quality of client education is not pertinent to this study either.

The nurse determines that a hockey player hospitalized with bilateral leg fractures is hemodynamically stable and observes the following pattern on the electrocardiogram (ECG) monitor. Which nursing intervention is most appropriate at this time? 1. None; this arrhythmia is benign 2. Administering atropine sulfate, 0.5 mg, as prescribed, to increase the heart rate 3. Continuing to monitor for lengthening PR intervals 4. Evaluating the client's recent serum electrolyte studies

Correct Answer: 1 RATIONALES: This ECG shows sinus arrhythmia with a rate of 70 beats/minute. In this benign arrhythmia, the rhythm is irregular; the impulse originates in the sinoatrial node and travels down the conduction system normally. The P-P interval is irregular; a P wave precedes every QRS complex; and the R-R interval is irregular, increasing with inspiration and decreasing with expiration. Sinus arrhythmia commonly is associated with vagal inhibition caused by respiration. It seldom causes symptoms and doesn't call for atropine or other treatment. Continuing to monitor for lengthening PR intervals isn't necessary because the PR interval doesn't increase with sinus arrhythmia. Because this arrhythmia isn't caused by an electrolyte imbalance, evaluating serum electrolyte studies isn't warranted.

A client is in hemorrhagic shock. To determine the effectiveness of fluid replacement therapy, the nurse should monitor the client's: 1. blood pressure. 2. hemoglobin level. 3. temperature. 4. heart rate.

Correct Answer: 1 RATIONALES: With adequate fluid replacement, fluid volume in the intravascular space expands, raising the client's blood pressure. The hemoglobin level reflects red blood cell concentration, not overall fluid status. Temperature and heart rate aren't directly related to fluid status.

The nurse is counseling a 52-year-old client about risk factors for hypertension. Which risk factors should the nurse list for primary hypertension? 1. Obesity 2. Diabetes mellitus 3. Head injury 4. Stress 5. Hormonal contraceptives 6. High intake of sodium or saturated fat

Correct Answer: 1,4,6 RATIONALES: Obesity, stress, high intake of sodium or saturated fat, and family history are all risk factors for primary hypertension. Diabetes mellitus, head injury, and hormonal contraceptives are risk factors for secondary hypertension.

A client with supraventricular tachycardia, is prescribed esmolol (Brevibloc). During esmolol therapy, the nurse should monitor the client's: 1. body temperature. 2. heart rate and blood pressure. 3. ocular pressure. 4. cerebral perfusion pressure.

Correct Answer: 2 RATIONALES: Because class II antiarrhythmics such as esmolol inhibit sinus node stimulation, they may produce bradycardia. Hypotension with peripheral vascular insufficiency also may occur, especially with esmolol. Class II antiarrhythmics don't alter body temperature, ocular pressure, or cerebral perfusion pressure.

The nurse reviews a client's medication history before administering a cholinergic blocking agent. Which drug may have delayed absorption because of the adverse effects of a cholinergic blocking agent? 1. amantadine (Symmetrel) 2. nitroglycerin (Nitrostat) 3. digoxin (Lanoxin) 4. diphenhydramine (Benadryl)

Correct Answer: 2 RATIONALES: A cholinergic blocking agent may delay the sublingual absorption of nitroglycerin because of dry mouth. The nurse should offer the client sips of water before administering nitroglycerin. Amantadine, digoxin, and diphenhydramine can interact with a cholinergic blocking agent but not through delayed absorption. Amantadine and diphenhydramine enhance the effects of anticholinergic agents.

The nurse is caring for a client experiencing dyspnea, dependent edema, hepatomegaly, crackles, and jugular vein distention. What condition should the nurse suspect? 1. Pulmonary embolism 2. Heart failure 3. Cardiac tamponade 4. Tension pneumothorax

Correct Answer: 2 RATIONALES: A client with heart failure has decreased cardiac output caused by the heart's decreased pumping ability. A buildup of fluid occurs, causing dyspnea, dependent edema, hepatomegaly, crackles, and jugular vein distention. A client with pulmonary embolism experiences acute shortness of breath, pleuritic chest pain, hemoptysis, and fever. A client with cardiac tamponade experiences muffled heart sounds, hypotension, and elevated central venous pressure. A client with tension pneumothorax has a deviated trachea and absent breath sounds on the affected side as well as dyspnea and jugular vein distention.

A client admitted with a massive myocardial infarction rapidly develops cardiogenic shock. Ideally, the physician would use the intra-aortic balloon pump (IABP) to support the injured myocardium. However, this client has a history of unstable angina pectoris, aortic insufficiency, hypertension, and diabetes mellitus. Which condition contraindicates use of the IABP? 1. Unstable angina pectoris 2. Aortic insufficiency 3. Hypertension 4. Diabetes mellitus

Correct Answer: 2 RATIONALES: A history of aortic insufficiency contraindicates use of the IABP. Other contraindications for this therapy include aortic aneurysm, central or peripheral atherosclerosis, chronic end-stage heart disease, multisystemic failure, chronic debilitating disease, bleeding disorders, and a history of emboli. Unstable angina pectoris that doesn't respond to drug therapy is an indication for IABP, not a contraindication. Hypertension and diabetes mellitus aren't contraindications for IABP.

Before administering digoxin (Lanoxin), a nurse reviews information about the drug. She learns that after digoxin is metabolized, the body eliminates remaining digoxin as unchanged drug by way of the: 1. lungs. 2. kidneys. 3. feces. 4. skin.

Correct Answer: 2 RATIONALES: After digoxin is metabolized, the kidneys eliminate remaining digoxin as unchanged drug. Therefore, a client with renal dysfunction will require a decreased digoxin dosage. Although some drugs may be eliminated by other routes, digoxin isn't known to be eliminated by way of the lungs, feces, or skin.

The nurse is teaching a client about maintaining a healthy heart. The nurse should include which point in her teaching? 1. Smoke in moderation. 2. Use alcohol in moderation. 3. Consume a diet high in saturated fats and low in cholesterol. 4. Exercise one or two times per week.

Correct Answer: 2 RATIONALES: Alcohol may be used in moderation as long as there are no other contraindications for its use. Smoking, a diet high in cholesterol and saturated fat, and a sedentary lifestyle are all known risk factors for cardiac disease. The client should be encouraged to quit smoking, exercise three to four times per week, and consume a diet low in cholesterol and saturated fat.

While receiving a heparin infusion to treat deep vein thrombosis, a client reports that the gums bleed when brushing the teeth. What should the nurse do first? 1. Stop the heparin infusion immediately. 2. Notify the physician. 3. Administer a coumarin derivative, as prescribed, to counteract heparin. 4. Reassure the client that bleeding gums are a normal effect of heparin.

Correct Answer: 2 RATIONALES: Because bleeding gums are an adverse effect of heparin that may indicate excessive anticoagulation, the nurse should notify the physician, who will evaluate the client's condition. Laboratory tests, such as partial thromboplastin time, should be performed before concluding that the client's bleeding is significant. The prescribed heparin dose may be therapeutic rather than excessive, so the nurse shouldn't discontinue the heparin infusion, unless the physician orders this after evaluating the client. Protamine sulfate, not a coumarin derivative, is given to counteract heparin. Bleeding gums aren't a normal effect of heparin.

A client with left-sided heart failure complains of increasing shortness of breath and is agitated and coughing up pinktinged, foamy sputum. The nurse should recognize these as signs and symptoms of: 1. right-sided heart failure. 2. acute pulmonary edema. 3. pneumonia. 4. cardiogenic shock.

Correct Answer: 2 RATIONALES: Because of decreased contractility and increased fluid volume and pressure in clients with heart failure, fluid may be driven from the pulmonary capillary beds into the alveoli, causing pulmonary edema. In right-sided heart failure, the client would exhibit hepatomegaly, jugular vein distention, and peripheral edema. In pneumonia, the client would have a temperature spike and sputum that varies in color. Cardiogenic shock would show signs of hypotension and tachycardia.

The nurse is preparing a client for cardiac catheterization. The nurse knows that she must provide which nursing intervention when the client returns to his room after the procedure? 1. Withhold analgesics for at least 6 hours after the procedure. 2. Assess the puncture site frequently for hematoma formation or bleeding. 3. Inform the client that he may experience numbness or pain in his leg. 4. Restrict fluids for 6 hours after the procedure.

Correct Answer: 2 RATIONALES: Because the diameter of the catheter used for cardiac catheterization is large, the puncture site must be checked frequently for hematoma formation and bleeding. The nurse should administer analgesics as prescribed and needed. If the femoral artery was accessed during the procedure, the client should be instructed to report any leg pain or numbness, which may indicate arterial insufficiency. Fluids should be encouraged to eliminate dye from the client's system.

A client with substernal chest pain that radiates to the jaw is admitted to the coronary care unit. The client subsequently develops hypotension and suffers cardiac arrest. Which calcium preparation is injected into the ventricle during cardiac arrest? 1. calcium carbonate (BioCal) 2. calcium chloride 3. calcium glubionate (Neo-Calglucon) 4. calcium lactate

Correct Answer: 2 RATIONALES: Calcium chloride is the only calcium preparation that should be injected into the ventricle during cardiac arrest, if appropriate.

A client with chest pain receives nitroglycerin on the way to the acute care facility. Based on an electrocardiogram obtained on admission, the physician suspects a myocardial infarction (MI) and prescribes I.V. morphine to relieve continuing pain. A primary goal of nursing care for this client is to recognize life-threatening complications of an MI. The major cause of death after an MI is: 1. cardiogenic shock. 2. cardiac arrhythmia. 3. heart failure. 4. pulmonary embolism.

Correct Answer: 2 RATIONALES: Cardiac arrhythmias cause roughly 40% to 50% of deaths after MI. Heart failure, in contrast, accounts for 33% and cardiogenic shock for 9% of post-MI deaths. Pulmonary embolism, another potential complication of an MI, is less common.

The nurse is obtaining a history from a new client in the cardiovascular clinic. When investigating for childhood diseases and disorders associated with structural heart disease, the nurse should consider which finding significant? 1. Croup 2. Rheumatic fever 3. Severe staphylococcal infection 4. Medullary sponge kidney

Correct Answer: 2 RATIONALES: Childhood diseases and disorders associated with structural heart disease include rheumatic fever and severe streptococcal (not staphylococcal) infections. Croup — a severe upper airway inflammation and obstruction that typically strikes children ages 3 months to 3 years — may cause latent complications, such as ear infection and pneumonia. However, it doesn't affect heart structures. Likewise, medullary sponge kidney, characterized by dilation of the renal pyramids and formation of cavities, clefts, and cysts in the renal medulla, eventually may lead to hypertension but doesn't damage heart structures.

After extensive cardiac bypass surgery, a client returns to the intensive care unit on dobutamine (Dobutrex), 5 mcg/kg/minute I.V. Which classification best describes dobutamine? 1. Indirect-acting dual-active agent 2. Direct-acting beta-active agent 3. Indirect-acting beta-active agent 4. Direct-acting alpha-active agent

Correct Answer: 2 RATIONALES: Dobutamine acts directly on beta receptors. Thus, the drug can be described as a direct-acting beta-active agent. Adrenergic agents are classified according to their method of action and the type of receptor they act on. Directacting agents act directly on the sympathetically innervated organ or tissue, whereas indirect-acting agents trigger the release of a neurotransmitter, usually norepinephrine. Dual-acting agents combine direct and indirect actions. Adrenergic agents act on alpha, beta, and dopamine receptors.

A client has a blockage in the proximal portion of a coronary artery. After learning about treatment options, the client decides to undergo percutaneous transluminal coronary angioplasty (PTCA). During this procedure, the nurse expects to administer an: 1. antibiotic. 2. anticoagulant. 3. antihypertensive. 4. anticonvulsant.

Correct Answer: 2 RATIONALES: During PTCA, the client receives heparin, an anticoagulant, as well as calcium agonists, nitrates, or both, to reduce coronary artery spasm. An antibiotic isn't given routinely during this procedure; however, because the procedure is invasive, the client may receive prophylactic antibiotics afterward to reduce the risk of infection. An antihypertensive agent may cause hypotension, which should be avoided during the procedure. An anticonvulsant isn't indicated because this procedure doesn't increase the risk of seizures.

A client receives a pacemaker to treat a recurring arrhythmia. When monitoring the cardiac rhythm strip, the nurse observes extra pacemaker spikes that aren't followed by a beat. Which condition should the nurse suspect? 1. Failure to pace 2. Failure to capture 3. Failure to sense 4. Asystole

Correct Answer: 2 RATIONALES: Extra pacemaker spikes that aren't followed by a beat may indicate failure to capture, in which the pacemaker fires but the heart doesn't conduct the beat. In failure to pace, the pacemaker doesn't fire when it should, causing hypotension and other signs of low cardiac output, accompanied by bradycardia or a heart rate slower than the pacemaker's preset rate. In failure to sense, the pacemaker can't sense the client's intrinsic heartbeat; on the rhythm strip, spikes may fall on T waves, or they may fall regularly but at points where they shouldn't appear. Asystole is characterized by an absent heart rate or rhythm as reflected by a flat line on the rhythm strip.

The nurse is assessing a client with heart failure. The breath sounds commonly auscultated in clients with heart failure are: 1. tracheal. 2. fine crackles. 3. coarse crackles. 4. friction rubs.

Correct Answer: 2 RATIONALES: Fine crackles are caused by fluid in the alveoli and commonly occur in clients with heart failure. Tracheal breath sounds are auscultated over the trachea. Coarse crackles are typically caused by secretion accumulation in the airways. Friction rubs occur with pleural inflammation.

Following a percutaneous transluminal coronary angioplasty (PTCA), a client is monitored in the postprocedure unit. The client's heparin infusion was stopped 2 hours earlier. There is no evidence of bleeding or hematoma at the insertion site, and the pressure device is removed. The nurse should plan to remove the femoral sheath when the partial thromboplastin time (PTT) is: 1. 25 seconds or less. 2. 50 seconds or less. 3. 75 seconds or less. 4. 100 seconds or less.

Correct Answer: 2 RATIONALES: Heparin causes an elevation of the PTT and, thereby, increases the risk for bleeding. With a large cannulation such as a sheath used for angioplasty, the PTT should be 40 seconds or less before the sheath is removed. Removing the sheath prematurely can cause bleeding at the insertion site. The other PTT results are incorrect for determining when to remove the sheath.

The nurse is assessing a client with heart failure. To assess hepatojugular reflux, the nurse should: 1. elevate the client's head to 90 degrees. 2. press the right upper abdomen. 3. press the left upper abdomen. 4. lie the client flat in bed.

Correct Answer: 2 RATIONALES: Hepatojugular reflux, a sign of right-sided heart failure, is assessed with the head of the bed at a 45-degree angle. As the right upper abdomen (the area over the liver) is compressed for 30 to 40 seconds, the nurse observes the internal jugular vein. If the internal jugular vein becomes distended, a client has positive hepatojugular reflux.

A client is receiving a lidocaine (Xylocaine) I.V. infusion at 2 mg/minute to treat runs of ventricular tachycardia. The client experiences hypotension, dyspnea, and irregular heartbeats, indicating heart failure. Which action can the nurse expect the physician to take first? 1. Prescribing 100 mg of lidocaine P.O. every 6 hours 2. Decreasing the lidocaine infusion to 1 mg/minute 3. Increasing the lidocaine infusion to 3 mg/minute 4. Discontinuing the lidocaine infusion

Correct Answer: 2 RATIONALES: In a client with heart failure or hepatic disease, the maintenance infusion of lidocaine should be reduced by one-third to one-half. Because the client is currently receiving 2 mg/minute, the physician will probably decrease the rate to 1 mg/minute. Lidocaine isn't administered in oral form because most of an absorbed dose undergoes first-pass metabolism in the liver. Increasing the rate of the lidocaine infusion can worsen heart failure. Discontinuing lidocaine isn't warranted in the presence of life-threatening PVCs.

A client is hospitalized with end-stage cardiomyopathy. The physician, nurse, client and her family discuss the possibility of heart transplantation. After this discussion, the nurse and physician meet to discuss the case. The physician voices his concern that the client will not change her lifestyle to accommodate transplantation. Which response by the nurse indicates her role as a client advocate? 1. "She only smokes a few cigarettes per day." 2. "With the proper support and education, she could make the necessary changes." 3. "It's not easy to make and maintain these kinds of changes." 4. "A transplant would save her life."

Correct Answer: 2 RATIONALES: In option 2, the nurse is advocating for the client by telling the physician that with proper education and support, the client could most likely make the necessary lifestyle changes. Informing the physician about the number of cigarettes the client smokes each day does not indicate advocacy. In option 3, the nurse is indicating that she is doubtful that the client could make the necessary changes. Option 4 may be true; however, this example does not display client advocacy.

A client is in the early stage of heart failure. During this time, which compensatory mechanism occurs? 1. Decreased renal blood flow causes the renin-angiotensin-aldosterone system to lower the secretion of aldosterone and antidiuretic hormone. 2. Low blood pressure triggers the baroreceptors to increase sympathetic nervous system stimulation. 3. Decreased renal blood flow causes the renin-angiotensin-aldosterone system to raise aldosterone secretion. 4. Low blood pressure triggers the baroreceptors to decrease sympathetic nervous system stimulation.

Correct Answer: 2 RATIONALES: In the early stage of heart failure, low blood pressure triggers baroreceptors in the carotid sinus and aortic arch to increase sympathetic nervous system stimulation, causing a faster heart rate, vasoconstriction, and increased myocardial oxygen consumption. Decreased renal blood flow causes the renin-angiotensin-aldosterone system to increase, not reduce, secretion of aldosterone and antidiuretic hormone, in turn causing sodium and water retention and arterial vasoconstriction.

A client with a permanent pacemaker and a long history of cardiac disease is admitted to the coronary care unit for evaluation for heart failure. The nurse observes the following electrocardiogram (ECG) pattern. What does this pattern indicate? 1. Use of a DDD pacemaker with a rate of 78 beats/minute 2. Use of a VVI pacemaker with a rate of 72 beats/minute 3. Use of an AVI pacemaker with a rate of 76 beats/minute 4. Use of an AAI pacemaker with a rate of 80 beats/minute

Correct Answer: 2 RATIONALES: In the pacemaker identification code, the first letter stands for the heart chamber being paced (atrium, ventricle, or both [D]); the second letter stands for the chamber being sensed (atrium, ventricle, both, or none [O]); and the third letter stands for the pacemaker's response to the sensed event (inhibited, triggered, both, or none). This ECG indicates use of a VVI pacemaker, which paces and senses the ventricle and is inhibited by a sensed event (a spontaneous QRS complex). A spike precedes every QRS complex stimulated by the pacemaker. Sensing that the client's intrinsic rate is below 72 beats/minute, the pacemaker triggers a ventricular impulse. The other options give incorrect rates; also, if the atrium were being paced, a spike would precede each P wave, indicating atrial contraction.

A client with an acute myocardial infarction is receiving nitroglycerin (Tridil) by continuous I.V. infusion. Which statement by the client indicates that this drug is producing its therapeutic effect? 1. "I have a bad headache." 2. "My chest pain is decreasing." 3. "I feel a tingling sensation around my mouth." 4. "My blood pressure must be up because my vision is blurred."

Correct Answer: 2 RATIONALES: Nitroglycerin, a vasodilator, increases the arterial supply of oxygen-rich blood to the myocardium, thus producing its intended effect: relief of chest pain. Headache is an adverse effect of nitroglycerin. The drug shouldn't cause a tingling sensation around the mouth and should lower, not raise, blood pressure.

A client signed a consent form for participation in a clinical trial for implantable cardioverter-defibrillators. Which statement by the client indicates the need for further teaching before true informed consent can be obtained? 1. "This implanted defibrillator will protect me against some of those bad rhythms my heart goes into." 2. "I wonder if there is any other way to prevent these bad rhythms." 3. "The physician will make a small incision in my chest wall and place the generator there." 4. "A wire from the generator will be attached to my heart."

Correct Answer: 2 RATIONALES: Option 2 indicates that other treatment options weren't discussed with the client. Before participation in a clinical trial, the client must be informed of all other available treatment options. Options 1, 3, and 4 are all true statements about implantable cardioverter-defibrillators.

A client is receiving captopril (Capoten) for heart failure. The nurse should notify the physician that the medication therapy is ineffective if an assessment reveals: 1. skin rash. 2. peripheral edema. 3. dry cough. 4. postural hypotension.

Correct Answer: 2 RATIONALES: Peripheral edema is a sign of fluid volume excess and worsening heart failure. A skin rash, dry cough, and postural hypotension are adverse reactions to captopril, but they don't indicate that therapy is ineffective.

A postoperative client is receiving heparin (Heparin sodium injection) after developing thrombophlebitis. The nurse monitors the client carefully for adverse effects of heparin, especially bleeding. If the client starts to exhibit signs of excessive bleeding, the nurse should expect to administer an antidote that is specific to heparin. Which agent fits this description? 1. phytonadione (vitamin K) 2. protamine sulfate 3. thrombin 4. plasma protein fraction

Correct Answer: 2 RATIONALES: Protamine sulfate is the antidote specific to heparin. Phytonadione (vitamin K) is the antidote specific to oral anticoagulants such as warfarin. (Heparin isn't given orally.) Thrombin is a hemostatic agent used to control local bleeding. Plasma protein fraction, a blood derivative, supplies colloids to the blood and expands plasma volume; it's used to treat clients in shock.

A 32-year-old female with systemic lupus erythematosus (SLE) complains that her hands become pale, blue, and painful when exposed to the cold. What disease should the nurse cite as an explanation for these sign and symptoms? 1. Peripheral vascular disease 2. Raynaud's disease 3. Arterial occlusive diseases 4. Buerger's disease

Correct Answer: 2 RATIONALES: Raynaud's disease results from reduced blood flow to the extremities when exposed to cold or stress. It's commonly associated with connective tissue disorders such as SLE. Signs and symptoms include pallor, coldness, numbness, throbbing pain, and cyanosis. Peripheral vascular disease results from a reduced blood supply to the tissues. It occurs in the arterial or venous system. Build-up of plaque in the vessels or changes in the vessels results in reduced blood flow, causing pain, edema, and hair loss in the affected extremity. Arterial occlusive disease is the obstruction or narrowing of the lumen of the aorta and its major branches that interrupts blood flow to the legs and feet, causing pain and coolness. Buerger's disease is an inflammatory, nonatheromatous occlusive disease that causes segmental lesions and subsequent thrombus formation in arteries, resulting in decreased blood flow to the feet and legs.

Which treatment would be the best therapy for a stable client with digitalis toxicity? 1. Activated charcoal 2. Time and symptomatic treatment 3. Hemodialysis 4. Atropine

Correct Answer: 2 RATIONALES: Stable clients with digitalis toxicity are best treated with time while their kidneys excrete the metabolites and with the symptomatic treatment for the rhythm disturbances or nausea resulting from the toxicity. Activated charcoal is effective only if the client has taken an overdose of digitalis and a large amount of unabsorbed drug is in the GI tract, before the serum level is elevated. Hemodialysis is reserved for clients who are extremely unstable despite symptomatic treatment or who have inadequate renal function to excrete the drug. Atropine might be used to treat the bradycardia that results from digitalis toxicity, but it isn't necessarily used to treat the toxicity itself.

An elderly client asks the nurse how to treat chronic constipation. What is the best recommendation the nurse can make? 1. Take a mild laxative such as magnesium citrate when necessary. 2. Take a stool softener such as docusate sodium (Colace) daily. 3. Administer a tap-water enema weekly. 4. Administer a phospho-soda (Fleet) enema when necessary.

Correct Answer: 2 RATIONALES: Stool softeners taken daily promote absorption of liquid into the stool, creating a softer mass. They may be taken on a daily basis without developing a dependence. Dependence is an adverse effect of daily laxative use. Enemas used daily or on a frequent basis can also lead to dependence of the bowel on an external source of stimulation.

A client comes to the physician's office for a complete physical examination required for employment. The physician assesses the client's arms and legs for evidence of peripheral vascular disease. What is the most commonly used overall indicator of arm and leg circulation? 1. Exercise testing 2. Ankle-brachial index 3. Limb blood pressure 4. Allen's test

Correct Answer: 2 RATIONALES: The ankle-brachial index is the most commonly used overall indicator of arm and leg circulation. Exercise testing reveals the severity of intermittent claudication and suggests how extensively this condition affects the client's lifestyle. Limb blood pressure is the single best indicator of arm or leg perfusion, but its significance is limited to the limb being examined; limb blood pressures may vary greatly if peripheral vascular disease is present in one limb but not the other. Allen's test is used to evaluate blood flow in the arm.

When a client is started on oral or I.V. diltiazem (Cardizem), the nurse should monitor for which potential complication? 1. Flushing 2. Heart failure 3. Renal failure 4. Hypertension

Correct Answer: 2 RATIONALES: The chief adverse effects of diltiazem are hypotension, atrioventricular blocks, heart failure, and elevated liver enzyme levels. Other reactions that have been reported include flushing, nocturia, and polyuria, but not renal failure. Although flushing may occur, it's an adverse reaction, not a potential complication. Heart failure is a lifethreatening reaction.

A client with end-stage heart failure is preparing for discharge. The client and his caregiver meet with the home care nurse and voice their concern that setting up a hospital bed in the bedroom will leave him feeling isolated. Which suggestion by the home care nurse best addresses this concern? 1. Place a chair in the bedroom so guests can visit with the client. 2. Set up the hospital bed in the family room so the client can be part of household activities. 3. Set up the hospital bed in the bedroom so the client can rest in a quiet environment. 4. Set up the hospital bed in the bedroom so the client can be assessed in a quiet environment.

Correct Answer: 2 RATIONALES: The client should be kept actively involved in the household to prevent feelings of isolation. This can be accomplished by setting up the hospital bed in the family room. Placing a chair in the bedroom allows the client periods of isolation when visitors aren't present. It's important for the client to have periods of rest; however, that can be accomplished without keeping the client isolated in a bedroom. The needs of the client should be considered before the needs of the nurse who assesses the client during an occasional visit.

The nurse is assigned with an ancillary staff member to care for a group of cardiac clients. Which client should the nurse address first? 1. The client admitted with unstable angina who wants to be discharged. 2. The client who suffered an acute myocardial infarction (MI) who is complaining of constipation. 3. The client who had a pacemaker inserted yesterday and who is complaining of incisional pain. 4. The client who has his call light on.

Correct Answer: 2 RATIONALES: The client who suffered an acute MI who is complaining of constipation should be addressed first. If the client strains at stool after an MI, the vagal response may be stimulated causing bradycardia thereby provoking arrhythmias. The nurse should delegate answering the call light to the ancillary personnel. She can also delegate some of the discharge preparation, such as packing the client's belongings. After addressing the MI client with constipation, the nurse should promptly address the pain relief needs of the client who had a pacemaker inserted the previous day

The nurse just received shift report for a group of clients on the telemetry unit. Which client should the nurse assess first. 1. The client with a history of atrial fibrillation 2. The client admitted with first-degree atrioventricular (AV) block whose cardiac monitor now reveals type II second-degree AV block 3. The client with a history of heart failure who has bibasilar crackles and pitting edema in both feet 4. The client with a demand pacemaker whose monitor shows normal sinus rhythm at a rate of 90 beats/minute

Correct Answer: 2 RATIONALES: The client whose cardiac rhythm now shows type II second-degree AV block should be assessed first. The client's rhythm has deteriorated from first-degree heart block to type II second-degree AV block and may continue to deteriorate into a lethal form of AV block (known as complete heart block). The client with a history of atrial fibrillation whose monitor reveals atrial fibrillation doesn't need to be assessed first. Because his rhythm is chronic, he has most likely been given an anticoagulant and isn't at immediate risk from this rhythm. The client with a history of heart failure may have chronic bibasilar crackles and pitting edema of both feet. Therefore, assessing this client first isn't necessary. The client's demand pacemaker fires only when the client's intrinsic heart rate falls below the pacemaker's set rate. In option 4, the pacemaker isn't firing because it most likely has been set at a slower rate than the client's intrinsic heart rate of 90 beats/minute.

Before using a defibrillator to terminate ventricular fibrillation, the nurse should check the synchronizer switch. Why is this so important? 1. The delivered shock must be synchronized with the client's QRS complex. 2. The defibrillator won't deliver a shock if the synchronizer switch is turned on. 3. The defibrillator won't deliver a shock if the synchronizer switch is turned off. 4. The shock must be synchronized with the client's T wave.

Correct Answer: 2 RATIONALES: The defibrillator won't deliver a shock to the client in ventricular fibrillation if the synchronizer switch is turned on because the defibrillator needs to recognize a QRS complex when the switch is turned on. The synchronizer switch should be turned on when attempting to terminate arrhythmias that contain QRS complexes, such as rapid atrial fibrillation that is resistant to pharmacologic measures.

Considering a client's atrial fibrillation, the nurse must administer digoxin (Lanoxin) with caution because it: 1. affects the sympathetic division of the autonomic nervous system, decreasing vagal tone. 2. stimulates the parasympathetic division of the autonomic nervous system, increasing vagal tone. 3. can induce hypertensive crisis by constricting arteries. 4. can trigger proarrhythmia by increasing stroke volume.

Correct Answer: 2 RATIONALES: The nurse must administer digoxin with caution in a client with atrial fibrillation because digoxin stimulates the parasympathetic division of the autonomic nervous system, increasing vagal tone. The vagal effect slows the heart rate, increases the refractory period, and slows conduction through the atrioventricular node and junctional tissue, thus increasing the potential for new arrhythmias to develop. Digoxin doesn't constrict arteries. Although it can trigger proarrhythmias, it does so by increasing vagal tone (not stroke volume).

Before discharge, which instruction should the nurse give to a client receiving digoxin (Lanoxin)? 1. "Take an extra dose of digoxin if you miss one dose." 2. "Call the physician if your heart rate is above 90 beats/minute." 3. "Call the physician if your pulse drops below 80 beats/minute." 4. "Take digoxin with meals."

Correct Answer: 2 RATIONALES: The nurse should instruct the client to notify the physician if his heart rate is greater than 90 beats/minute because cardiac arrhythmias may occur with digitalis toxicity. To prevent toxicity, the client should be instructed never to take an extra dose of digoxin if a dose is missed. The nurse should show the client how to take her pulse and to call the physician if her pulse rate drops below 60 beats/minute — not 80 beats/minute, which is a normal pulse rate and doesn't warrant action. Digoxin shouldn't be administered with meals because this slows the absorption rate.

The visiting nurse is teaching a client with heart failure about taking his medications. The client requires six different medications that are taken at four different times per day. The client is confused about when to take each medication. How should the nurse intervene? 1. Ask the client's family to take turns coming to the house at each administration time to assist the client with his medications. 2. Teach a family member to fill a medication compliance aid once a week so the client can independently take his medications. 3. Ask the physician if the client can take fewer pills each day. 4. Come to the client's house each morning to prepare the daily allotment of medications.

Correct Answer: 2 RATIONALES: The nurse should intervene by asking a family member to fill a compliance aid each week with the client's weekly supply of medications in the appropriate time slots. Family members can't be expected to come to the client's house four times each day to administer medications. The physician shouldn't change the dosing regimen just for convenience. The home care nurse can't visit the client each morning to prepare the daily medication regimen.

The home care nurse is visiting a left-handed client who has an automated implantable cardioverter-defibrillator implanted in his left chest. The client tells the nurse how excited he is because he's planning to go rifle hunting with his grandson. How should the nurse respond? 1. "Be sure to enjoy your time with your grandson." 2. "You cannot shoot a rifle left-handed because the rifle's recoil will traumatize the AICD site." 3. "Being that close to a rifle might make your AICD fire." 4. "You will need to take an extra dose of your antiarrhythmic before you shoot."

Correct Answer: 2 RATIONALES: The recoil from the rifle can damage the AICD so the client should be warned against shooting a rifle with his left hand. Close proximity to a rifle won't cause the AICD to fire inadvertently. The client shouldn't take an extra dose of his antiarrhythmic.

The nurse correctly instructs a client with peripheral vascular disease that stress-reduction techniques: 1. are helpful only because they assist in smoking cessation. 2. are helpful because stress stimulates the release of vasoconstricting catecholamines. 3. are helpful because they distract the client from focusing on claudication pain. 4. haven't proved useful in clients with peripheral vascular disease.

Correct Answer: 2 RATIONALES: The stress-induced release of vasoactive catecholamines, such as epinephrine, causes vasoconstriction, which directly aggravates peripheral vascular disease by intensifying the ischemic burden of the affected tissues. Vasoconstriction also indirectly aggravates atherogenesis by inducing hypertension. Stress-reduction techniques make it easier for clients to avoid bad habits, such as smoking; however, this isn't the only reason why they're useful. Claudication is a signal of muscle ischemia and shouldn't be ignored

A client with a history of myocardial infarction is admitted with shortness of breath, anxiety, and slight confusion. Assessment findings include a regular heart rate of 120 beats/minute, audible third and fourth heart sounds, blood pressure of 84/64 mm Hg, bibasilar crackles on lung auscultation, and a urine output of 5 ml over the past hour. The nurse anticipates preparing the client for transfer to the intensive care unit and pulmonary artery catheter insertion because: 1. the client is experiencing heart failure. 2. the client is going into cardiogenic shock. 3. the client shows signs of aneurysm rupture. 4. the client is in the early stage of right-sided heart failure.

Correct Answer: 2 RATIONALES: This client's findings indicate cardiogenic shock, which occurs when the heart fails to pump properly, impeding blood supply and oxygen flow to vital organs. Cardiogenic shock also may cause cold, clammy skin and generalized weakness, fatigue, and muscle pain as lactic acid accumulates from poor blood flow, preventing waste removal. Left-sided and right-sided heart failure eventually cause venous congestion with jugular vein distention and edema as the heart fails to pump blood forward. A ruptured aneurysm causes severe hypotension and a quickly deteriorating clinical status from blood loss and circulatory collapse; this client has low but not severely decreased blood pressure. Also, in ruptured aneurysm, deterioration is more rapid and full cardiac arrest is common.

The nurse would advise the client with an axillofemoral bypass graft to avoid: 1. standing for prolonged periods. 2. tight belts. 3. reclining and instead sit in a chair for prolonged periods. 4. upper-extremity exercises.

Correct Answer: 2 RATIONALES: Tight belts around the waist can occlude the axillofemoral bypass; the client should use suspenders instead. Prolonged sitting can kink the femoral portion of the graft. Prolonged standing and upper-extremity exercises need not be avoided.

The nurse is caring for a client with hemiparesis caused by a stroke. The client is barely responsive. Which intervention takes highest priority? 1. Performing passive range-of-motion (ROM) exercises 2. Placing the client on the affected side 3. Using hand rolls or pillows for support 4. Applying antiembolism stockings as ordered

Correct Answer: 2 RATIONALES: To help prevent airway obstruction and reduce the risk of aspiration, the nurse should position a client with hemiparesis on the affected side. Although performing ROM exercises, providing pillows for support, and applying antiembolism stockings may be appropriate interventions for a client who has had a stroke, maintaining a patent airway is the first concern.

During surgery, a client develops sinus bradycardia. The physician orders atropine sulfate. Which dose and route is the nurse most likely to administer? 1. 0.6 mg I.M. 2. 1 mg I.V. 3. 2 mg I.M. 4. 2 mg I.V.

Correct Answer: 2 RATIONALES: To reverse arrhythmias, bradycardia, or sinus arrest, the usual adult dosage of atropine is 0.4 to 1 mg I.V. every 2 hours as needed. The drug isn't administered I.M. for the treatment of bradycardia

The nurse is caring for a client who is awaiting heart transplantation. The client and her family express their concerns about the financial cost of the procedure. Which intervention by the nurse is most appropriate? 1. Reassure the client and her family that the cost will be covered. 2. Contact the social worker and request that she speak to the client and her family about their financial concerns. 3. Tell the client that she will be responsible for all of her costs. 4. Have the physician speak to the client and family about the costs.

Correct Answer: 2 RATIONALES: Transplantation requires a multidisciplinary team approach. A social worker is always included as part of that team. The nurse should contact the social worker and request that she speak to the client and her family about their financial concerns about the transplant. Reassuring the client that the cost will be covered is false reassurance that doesn't address the client's concern. Many insurance companies pay for the expenses surrounding a transplant; the client isn't personally responsible. The physician isn't typically involved with discussions about financial responsibilities.

The nurse suspects that a 68-year-old client has digitalis toxicity. The nurse should assess for: 1. hearing loss. 2. vision changes. 3. decreased urine output. 4. gait instability.

Correct Answer: 2 RATIONALES: Vision changes, such as halos around objects, are signs of digitalis toxicity. Hearing loss can be detected through hearing assessment; however, it isn't a common sign of digitalis toxicity. Intake and output aren't affected unless there is nephrotoxicity, which is uncommon. Gait changes are also uncommon.

A client is recovering from surgical repair of a dissecting aortic aneurysm. The nurse should evaluate the client for signs of bleeding or recurring dissection. These signs include: 1. hematuria and decreased urine output. 2. hypotension and tachycardia. 3. increased urine output and bradycardia. 4. hypotension and bradycardia.

Correct Answer: 2 RATIONALES: When caring for a client recovering from surgical repair of a dissecting aortic aneurysm, the nurse must monitor for hypotension with reflex tachycardia, decreased urine output, and unequal or absent peripheral pulses — all potential signs of bleeding or recurring dissection. Hematuria, increased urine output, and bradycardia aren't signs of bleeding from aneurysm repair or recurring dissection.

A client is experiencing an acute myocardial infarction (MI) and I.V. morphine is prescribed. Morphine is given because it: 1. eliminates pain, reduces cardiac workload, and increases myocardial contractility. 2. lowers resistance, reduces cardiac workload, and decreases myocardial oxygen demand. 3. raises the blood pressure, lowers myocardial oxygen demand, and eliminates pain. 4. increases venous return, lowers resistance, and reduces cardiac workload.

Correct Answer: 2 RATIONALES: When given to treat acute MI, morphine sulfate eliminates pain, reduces venous return to the heart, reduces vascular resistance, reduces myocardial workload, and reduces the oxygen demand of the heart. Morphine sulfate doesn't increase myocardial contractility, raise blood pressure, or increase venous return.

A client who suffered blunt chest trauma in a car accident complains of chest pain, which is exacerbated by deep inspiration. On auscultation, the nurse detects a pericardial friction rub — a classic sign of acute pericarditis. The physician confirms acute pericarditis and begins appropriate medical intervention. To relieve chest pain associated with pericarditis, the nurse should encourage the client to assume which position? 1. Semi-Fowler's 2. Leaning forward while sitting 3. Supine 4. Prone

Correct Answer: 2 RATIONALES: When the client leans forward, the heart pulls away from the diaphragmatic pleurae of the lungs, helping relieve chest pain caused by pericarditis. The semi-Fowler, supine, and prone positions don't cause this pulling-away action and therefore don't relieve chest pain associated with pericarditis.

A client is prescribed lisinopril (Zestril) for treatment of hypertension. He asks the nurse about possible adverse effects. The nurse should teach him about which common adverse effects of angiotensin-converting enzyme (ACE) inhibitors? 1. Constipation 2. Dizziness 3. Headache 4. Hyperglycemia 5. Hypotension 6. Impotence

Correct Answer: 2,3,5 RATIONALES: Dizziness, headache, and hypotension are all common adverse effects of lisinopril and other ACE inhibitors. Lisinopril may cause diarrhea, not constipation. Lisinopril isn't known to cause hyperglycemia or impotence.

The nurse suspects that her client is in cardiac arrest. According to the American Heart Association, the nurse should perform the actions listed below. Order these actions in the sequence that the nurse should perform them. 1. Activate the emergency medical system. 2. Assess responsiveness. 3. Call for a defibrillator. 4. Provide two slow breaths. 5. Assess pulse. 6. Assess breathing.

Correct Answer: 213645 RATIONALES: According to the American Heart Association, the nurse should first assess responsiveness. If the client is unresponsive, the nurse should activate the emergency medical system, and then call for a defibrillator. Next, the nurse should assess breathing by opening the airway and then looking, listening, and feeling for respirations. If respirations aren't present, the nurse should administer two slow breaths, then assess the pulse. If no pulse is present, the nurse should start chest compressions.

A client with deep vein thrombosis has an I.V. infusion of heparin sodium infusing at 1,500 U/hour. The concentration in the bag is 25,000 U/500 ml. How many milliliters of solution should the nurse document as intake from this infusion for an 8-hour shift?

Correct Answer: 240 RATIONALES: First, calculate how many units are in each milliliter of the medication: 25,000 U/500 ml = 50 U/ml Next, calculate how many milliliters the client receives each hour: 1 ml/50 U × 1,500 U/hour = 30 ml/hour Lastly, multiply by 8 hours: 30 ml/hour × 8 hours = 240 ml

The nurse on the telemetry unit is faced with various situations. Which situation takes priority? 1. A client's cardiac monitor suddenly reveals sinus tachycardia with isolated premature ventricular contractions. 2. A client's cardiac rhythm suddenly changes from normal sinus rhythm to uncontrolled atrial fibrillation. 3. A client is requesting help to go to the bathroom. 4. The cardiologist is asking the nurse to make rounds with him to his clients.

Correct Answer: 2RATIONALES: The client whose cardiac rhythm suddenly changes from normal sinus rhythm to uncontrolled atrial fibrillation takes priority. This cardiac rhythm change may cause clots to shower from the atria placing the client at risk for a stroke. The client whose cardiac monitor reveals sinus tachycardia with isolated premature ventricular contractions isn't experiencing a life-threatening situation; therefore, he doesn't take priority. The nurse can ask her ancillary staff member to assist the client to the bathroom. Making rounds with the physician can wait until the nurse addresses the needs of the client in atrial fibrillation.

A client is participating in a cardiac research study in which his physician is directly involved. Which statement by the client indicates a lack of understanding about his rights as a research study participant? 1. "I can withdraw from the study at anytime." 2. "My confidentiality will not be compromised by this study." 3. "I will have to find a new physician if I do not complete this study." 4. "I understand the risks associated with this study."

Correct Answer: 3 RATIONALE: The client's participation in this study should not influence the relationship with his physician. The client has the right to withdraw from a study at any time without penalty. All information provided by the client will be kept confidential and used only by members of the study team for scientific purposes. The client must be informed of all risks associated with study participation.

A client is admitted to an acute care facility with pneumonia. When auscultating heart sounds, the nurse notes a fixed split of the second heart sound (S2) — a pathological split that doesn't vary with respirations. A fixed S2 split is the hallmark of: 1. right bundle-branch block. 2. left bundle-branch block. 3. atrial septal defect. 4. aortic stenosis.

Correct Answer: 3 RATIONALES: A fixed S2 split is the hallmark of atrial septal defect. This split, which is continuous and doesn't vary with respirations, results from prolonged emptying of the right ventricle. A right bundle-branch block causes a wide S2 split that is louder on inspiration than on expiration; this split results from delayed depolarization of the right ventricle and late pulmonic valve closure. Left bundle-branch block, aortic stenosis, and patent ductus arteriosus cause a paradoxical S2 split. Heard only on expiration, a paradoxical S2 split results from delayed aortic valve closure.

The nurse is preparing a client with Crohn's disease for a barium enema. What should the nurse do the day before the test? 1. Serve the client his usual diet. 2. Order a high-fiber diet. 3. Encourage plenty of fluids. 4. Serve dairy products.

Correct Answer: 3 RATIONALES: Adequate fluid intake is necessary to avoid dehydration that may be caused by the bowel preparation and to prevent fecal impaction after the procedure. The client may be placed on a low-residue diet 1 to 2 days before the procedure to reduce the contents in the GI tract. Fiber intake is limited in a low-residue diet. Because dairy products leave a residue, they aren't allowed the evening before the test. Clear liquids only are allowed the evening before the test.

A client with chronic arterial occlusive disease undergoes percutaneous transluminal coronary angioplasty (PTCA) for mechanical dilation of the right femoral artery. After the procedure, the client will require long-term administration of: 1. aspirin or acetaminophen (Tylenol). 2. pentoxifylline (Trental) or acetaminophen (Tylenol). 3. aspirin or dipyridamole (Persantine). 4. penicillin V potassium (Pen-Vee K) or erythromycin (E-Mycin).

Correct Answer: 3 RATIONALES: After PTCA, the client begins long-term aspirin or dipyridamole therapy to prevent thromboembolism. Heparin is given for anticoagulation during this procedure; some physicians discharge clients with a prescription for longterm warfarin (Coumadin) or low-molecular-weight heparin therapy. Pentoxifylline, a vasodilator used to treat chronic arterial occlusion, isn't required after PTCA because the procedure itself opens the vessel. The physician may prescribe short-term acetaminophen therapy to manage fever or discomfort, but prolonged therapy isn't warranted. After the procedure, the client may need an antibiotic, such as penicillin or erythromycin, for a brief period to prevent infection associated with an invasive procedure; long-term therapy isn't necessary.

A 55-year-old black male is found to have a blood pressure of 150/90 mm Hg during a work-site health screening. What should the nurse do? 1. Consider this to be a normal finding for his age and race. 2. Recommend he have his blood pressure rechecked in 1 year. 3. Recommend he have his blood pressure rechecked within 2 weeks. 4. Recommend he see his physician immediately for further evaluation.

Correct Answer: 3 RATIONALES: Although hypertension is more prevalent in the black population, a blood pressure of 150/90 mm Hg isn't considered normal. He should have his blood pressure rechecked within 2 weeks. One year is too long to wait. He need not see his physician yet.

The nurse is caring for a client with end-stage heart failure. Which statement by the client best demonstrates a good understanding of an advance directive? 1. "I will rely on my doctor to do whatever is best for me." 2. "Once I decide on an advance directive, I cannot change my mind." 3. "A living will allows my decisions for health care to be known if I can't speak for myself." 4. "A health care power of attorney will allow my daughter to use my funds to pay for my health care costs, if I can't do so"

Correct Answer: 3 RATIONALES: An advance directive is a document written in the form of a living will. It expresses the client's wishes about health care, providing direction for the physician if the client becomes terminally ill and can't express his wishes. Option 1, which requires relying on the physician to decide care, takes the decision away from the client. A client can change his mind about advance directives at any time. A health care power of attorney allows the client to designate another person to make health care decisions for the client in case that the client becomes too ill to make his own decisions.

An increase in the creatine kinase-MB isoenzyme (CK-MB) can be caused by: 1. cerebral bleeding. 2. I.M. injection. 3. myocardial necrosis. 4. skeletal muscle damage due to a recent fall.

Correct Answer: 3 RATIONALES: An increase in CK-MB is related to myocardial necrosis. An increase in total CK might occur for several reasons, including brain injury, such as cerebral bleeding; skeletal muscle damage, which can be caused by I.M. injections or falls; muscular or neuromuscular disease; vigorous exercise; trauma; or surgery.

When teaching a client with newly diagnosed hypertension about the pathophysiology of this disease, the nurse states that arterial baroreceptors, which monitor arterial pressure, are found in the carotid sinus and aorta. Which other area should the nurse mention as the site of arterial baroreceptors? 1. Brachial artery 2. Radial artery 3. Left ventricular wall 4. Right ventricular wall

Correct Answer: 3 RATIONALES: Arterial baroreceptors are found in the left ventricular wall as well as the carotid sinus and aorta. None exist in the brachial artery, radial artery, or right ventricular wall.

An 84-year-old male is returning from the operating room (OR) after inguinal hernia repair. The nurse notes that he has fluid volume excess from the operation and is at risk for left-sided heart failure. Which sign or symptom indicates leftsided heart failure? 1. Jugular vein distention 2. Right upper quadrant pain 3. Bibasilar fine crackles 4. Dependent edema

Correct Answer: 3 RATIONALES: Bibasilar fine crackles are a sign of alveolar fluid, a sequelae of left ventricular fluid, or pressure overload. Jugular vein distention, right upper quadrant pain (hepatomegaly), and dependent edema are caused by right-sided heart failure, usually a chronic condition.

A client with a forceful, pounding heartbeat is diagnosed with mitral valve prolapse. This client should avoid which of the following? 1. High volumes of fluid intake 2. Aerobic exercise programs 3. Caffeine-containing products 4. Foods rich in protein

Correct Answer: 3 RATIONALES: Caffeine is a stimulant, which can exacerbate palpitations, and should be avoided by a client with symptomatic mitral valve prolapse. High-fluid intake helps maintain adequate preload and cardiac output. Aerobic exercise helps increase cardiac output and decrease heart rate. Protein-rich foods aren't restricted but high-calorie foods are.

The nurse is caring for a client who is recovering from a myocardial infarction (MI). The cardiologist refers him to cardiac rehabilitation. Which statement by the client indicates an understanding of cardiac rehabilitation? 1. "When I finish the rehabilitation program I'll never have to worry about heart trouble again." 2. "I won't be able to jog again even with rehabilitation." 3. "Rehabilitation will help me function as well as I physically can." 4. "I'll get rest during these rehabilitation classes. All I have to do is sit and listen to the instructor."

Correct Answer: 3 RATIONALES: Cardiac rehabilitation helps the client reach his activity potential. Coronary artery disease, which typically causes an acute MI, is a chronic condition that isn't cured. Many clients who suffer an acute MI can eventually return to such activities as jogging, depending on the extent of cardiac damage. Cardiac rehabilitation involves physical activity as well as classroom education.

The home care nurse visits a client diagnosed with atrial fibrillation who is prescribed warfarin (Coumadin). The nurse teaches the client about warfarin therapy. Which statement by the client indicates the need for further teaching? 1. "I will watch my gums for bleeding when I brush my teeth." 2. "I will use an electric razor to shave." 3. "I will eat four servings of fresh, dark greens vegetables every day." 4. "I will report any unexplained or severe bruising to my doctor right away."

Correct Answer: 3 RATIONALES: Dark, green vegetables contain vitamin K, which reverses the effects of warfarin. The client should limit his intake to one to two servings per day. The client should report bleeding gums and any severe or unexplained bruising, which may indicate an excessive dose of warfarin. The client should use an electric razor to prevent cutting himself while shaving.

The home care nurse visits a client diagnosed with atrial fibrillation who is prescribed warfarin (Coumadin). The nurse teaches the client about warfarin therapy. Which statement by the client indicates the need for further teaching? 1. "I will watch my gums for bleeding when I brush my teeth." 2. "I will use an electric razor to shave." 3. "I will eat four servings of fresh, dark greens vegetables every day." 4. "I will report any unexplained or severe bruising to my doctor right away."

Correct Answer: 3 RATIONALES: Dark, green vegetables contain vitamin K, which reverses the effects of warfarin. The client should limit his intake to one to two servings per day. The client should report bleeding gums and any severe or unexplained bruising, which may indicate an excessive dose of warfarin. The client should use an electric razor to prevent cutting himself while shaving.

The nurse is caring for a client with left-sided heart failure. To reduce fluid volume excess, the nurse can anticipate using: 1. antiembolism stockings. 2. oxygen. 3. diuretics. 4. anticoagulants.

Correct Answer: 3 RATIONALES: Diuretics, such as furosemide (Lasix), reduce total blood volume and circulatory congestion. Antiembolism stockings prevent venostasis and thromboembolism formation. Oxygen administration increases oxygen delivery to the myocardium and other vital organs. Anticoagulants prevent clot formation but don't decrease fluid volume excess.

When administering low doses of dopamine (Intropin), the nurse knows that dopamine activates which receptors? 1. Alpha 2. Beta1 3. Dopaminergic 4. Beta2

Correct Answer: 3 RATIONALES: Dopamine activates dopaminergic receptor sites only at low doses. At normal or high doses, dopamine activates alpha and beta1 receptor sites. Dopamine doesn't activate beta2 receptor sites.

A client is admitted to the health care facility for treatment of an abdominal aortic aneurysm. When planning this client's care, the nurse formulates interventions with which goal in mind? 1. Decreasing blood pressure and increasing mobility 2. Increasing blood pressure and reducing mobility 3. Stabilizing the heart rate and blood pressure and easing anxiety 4. Increasing blood pressure and monitoring fluid intake and output

Correct Answer: 3 RATIONALES: For a client with an aneurysm, nursing interventions focus on stabilizing the heart rate and blood pressure, to avoid aneurysm rupture. Easing anxiety also is important because anxiety and increased stimulation may speed the heart rate and boost blood pressure, precipitating aneurysm rupture. Typically, the client with an abdominal aortic aneurysm is hypertensive, so the nurse should take measures to lower the blood pressure, such as administering antihypertensive agents, as prescribed, to prevent aneurysm rupture. To sustain major organ perfusion, a mean arterial pressure of at least 60 mm Hg should be maintained. Although mobility must be assessed individually, most clients need bed rest initially when attempting to gain stability.

The nurse administers furosemide (Lasix) to treat a client with heart failure. Which adverse effect must the nurse watch for most carefully? 1. Increase in blood pressure 2. Increase in blood volume 3. Low serum potassium level 4. High serum sodium level

Correct Answer: 3 RATIONALES: Furosemide is a potassium-wasting diuretic. The nurse must monitor the serum potassium level and assess for signs of low potassium. As water and sodium are lost in the urine, blood pressure decreases, blood volume decreases, and urine output increases.

The nurse is teaching a client how to take nitroglycerin to treat angina pectoris. The client verbalizes an understanding of the need to take up to three sublingual nitroglycerin (Nitrostat) tablets at 5-minute intervals, if necessary, and to notify the physician immediately if chest pain doesn't subside within 15 minutes. The nurse knows that nitroglycerin may cause: 1. nausea, vomiting, depression, fatigue, and impotence. 2. sedation, nausea, vomiting, constipation, and respiratory depression. 3. headache, hypotension, dizziness, and flushing. 4. flushing, dizziness, headache, and pedal edema.

Correct Answer: 3 RATIONALES: Headache, hypotension, dizziness, and flushing are classic adverse effects of nitroglycerin, a vasodilator. Vasodilators, beta-adrenergic blockers, and calcium channel blockers are three major classes of drugs used to treat angina pectoris. Nausea, vomiting, depression, fatigue, and impotence are adverse effects of propranolol, a betaadrenergic blocker. Sedation, nausea, vomiting, constipation, and respiratory depression are common adverse effects of morphine, an opioid analgesic used to relieve pain associated with acute myocardial infarction. Flushing, dizziness, headache, and pedal edema are common adverse effects of nifedipine, a calcium channel blocker.

The nurse would obtain serum levels of which electrolytes in a client with frequent episodes of ventricular tachycardia? 1. Calcium and magnesium 2. Potassium and calcium 3. Magnesium and potassium 4. Potassium and sodium

Correct Answer: 3 RATIONALES: Hypomagnesemia as well as hypokalemia and hyperkalemia are common causes of ventricular tachycardia. Calcium imbalances cause changes in the QT interval and ST segment. Alterations in sodium level don't cause rhythm disturbances.

A client is receiving nitroglycerin ointment (Nitrol) to treat angina pectoris. The nurse evaluates the therapeutic effectiveness of this drug by assessing the client's response and checking for adverse effects. Which vital sign is most likely to reflect an adverse effect of nitroglycerin? 1. Heart rate 2. Respiratory rate 3. Blood pressure 4. Temperature

Correct Answer: 3 RATIONALES: Hypotension and headache are the most common adverse effects of nitroglycerin. Therefore, blood pressure is the vital sign most likely to reflect an adverse effect of this drug. The nurse should check the client's blood pressure 1 hour after administering nitroglycerin ointment. A blood pressure decrease of 10 mm Hg is within the therapeutic range. If blood pressure falls more than 20 mm Hg below baseline, the nurse should remove the ointment and report the finding to the physician immediately. An above-normal heart rate (tachycardia) is a less common adverse effect of nitroglycerin. Respiratory rate and temperature don't change significantly after nitroglycerin administration.

A client develops atrial fibrillation after an acute myocardial infarction. The physician prescribes digoxin (Lanoxin), 0.125 mg I.M. daily. The nurse clarifies the order with the physician because I.M. administration of digoxin leads to: 1. an increased serum creatinine level. 2. a decreased serum digoxin level. 3. an increased serum creatine kinase (CK) level. 4. a decreased serum CK level.

Correct Answer: 3 RATIONALES: I.M. administration of digoxin isn't recommended because it causes severe pain at the injection site and increases serum CK, which complicates interpretation of enzyme levels. Regardless of the route of administration, digoxin doesn't increase the serum creatinine level. When digoxin is administered, the serum digoxin level will rise from zero, not decrease.

To check for arterial insufficiency when a client is in a supine position, the nurse should elevate the extremity at a 45degree angle and then have the client sit up. The nurse suspects arterial insufficiency if the assessment reveals: 1. elevational rubor. 2. no rubor for 10 seconds after the maneuver. 3. dependent pallor. 4. a 30-second filling time for the veins.

Correct Answer: 3 RATIONALES: If arterial insufficiency is present, elevation of the limb would yield a pallor from the lack of circulation. Rubor and increased venous filling time would suggest venous problems secondary to venous trapping and incompetent valves.

A client comes to the emergency department complaining of chest pain. An electrocardiogram (ECG) reveals myocardial ischemia and an anterior-wall myocardial infarction (MI). Which ECG characteristic indicates myocardial ischemia? 1. Prolonged PR interval 2. Absent Q wave 3. Elevated ST segment 4. Widened QRS complex

Correct Answer: 3 RATIONALES: Ischemic myocardial tissue changes cause elevation of the ST segment, a peaked or inverted T wave, and a pathological Q wave. A prolonged PR interval occurs with first-degree heart block, the least dangerous atrioventricular heart block; this disorder may arise in healthy people but sometimes results from drug toxicity, electrolyte or metabolic disturbances, rheumatic fever, or chronic degenerative disease of the conduction system. An absent Q wave is normal; an MI may cause a significant Q wave. A widened QRS complex indicates a conduction delay in the His-Purkinje system.

A client with high blood pressure is receiving an antihypertensive drug. The nurse knows that antihypertensive drugs commonly cause fatigue and dizziness, especially on rising. When developing a client teaching plan to minimize orthostatic hypotension, the nurse should include which instruction? 1. "Avoid drinking alcohol and straining at stool, and eat a low-protein snack at night." 2. "Wear elastic stockings, change positions quickly, and hold onto a stationary object when rising." 3. "Flex your calf muscles, avoid alcohol, and change positions slowly." 4. "Rest between demanding activities, eat plenty of fruits and vegetables, and drink 6 to 8 cups of fluid daily."

Correct Answer: 3 RATIONALES: Measures that minimize orthostatic hypotension include flexing the calf muscles to boost blood return to the heart, avoiding alcohol and straining at stool, changing positions slowly, eating a high-protein snack at night, wearing elastic stockings, and holding onto a stationary object when rising. Although the client should rest between demanding activities and consume plenty of fluids and fiber (contained in fruits and vegetables) to maintain a balanced diet, these measures don't directly relieve orthostatic hypotension.

A client with mitral stenosis is scheduled for mitral valve replacement. Which condition may arise as a complication of mitral stenosis? 1. Left-sided heart failure 2. Myocardial ischemia 3. Pulmonary hypertension 4. Left ventricular hypertrophy

Correct Answer: 3 RATIONALES: Mitral stenosis, or severe narrowing of the mitral valve, impedes blood flow through the stenotic valve, increasing pressure in the left atrium and pulmonary circulation. This may lead to low cardiac output, pulmonary hypertension, edema, and right-sided (not left-sided) heart failure. Other potential complications of mitral stenosis include mural thrombi, pulmonary hemorrhage, and embolism to vital organs. Myocardial ischemia may occur in a client with coronary artery disease. Left ventricular hypertrophy is a potential complication of aortic stenosis.

The nurse is teaching a client who receives nitrates for the relief of chest pain. Which instruction should the nurse emphasize? 1. Repeat the dose of sublingual nitroglycerin every 15 minutes for three doses. 2. Store the drug in a cool, well-lit place. 3. Lie down or sit in a chair for 5 to 10 minutes after taking the drug. 4. Restrict alcohol intake to two drinks per day.

Correct Answer: 3 RATIONALES: Nitrates act primarily to relax coronary smooth muscle and produce vasodilation. They can cause hypotension, which makes the client dizzy and weak. Nitrates are taken at the first sign of chest pain and before activities that might induce chest pain. Sublingual nitroglycerin is taken every 5 minutes for three doses. If the pain persists, the client should seek medical assistance immediately. Nitrates must be stored in a dark place in a closed container. Sunlight causes the medication to lose its effectiveness. Alcohol is prohibited because nitrates may enhance the effects of the alcohol.

The nurse is caring for a client with cholelithiasis. Which sign indicates obstructive jaundice? 1. Straw-colored urine 2. Reduced hematocrit 3. Clay-colored stools 4. Elevated urobilinogen in the urine

Correct Answer: 3 RATIONALES: Obstructive jaundice develops when a stone obstructs the flow of bile in the common bile duct. When the flow of bile to the duodenum is blocked, the lack of bile pigments results in a clay-colored stool. In obstructive jaundice, urine tends to be dark amber (not straw-colored) as a result of soluble bilirubin in the urine. Hematocrit levels aren't affected by obstructive jaundice. Because obstructive jaundice prevents bilirubin from reaching the intestine (where it's converted to urobilinogen), the urine contains no urobilinogen.

The physician orders blood coagulation tests to evaluate a client's blood-clotting ability. The nurse knows that such tests are important in assessing clients at risk for thrombi, such as those with a history of atrial fibrillation, infective endocarditis, prosthetic heart valves, or myocardial infarction. Which test is used to determine a client's response to oral anticoagulant drugs? 1. Bleeding time 2. Platelet count 3. Prothrombin time (PT) 4. Partial thromboplastin time (PTT)

Correct Answer: 3 RATIONALES: PT determines a client's response to oral anticoagulant therapy. This test measures the time required for a fibrin clot to form in a citrated plasma sample after calcium ions and tissue thromboplastin are added and compares this time with the fibrin clotting time in a control sample. Anticoagulant dosages should be adjusted, as needed, to maintain PT at 1.5 to 2.5 times the control value. PTT determines the effectiveness of heparin therapy and helps evaluate bleeding tendencies. Roughly 99% of bleeding disorders are diagnosed from PT and PTT values. Bleeding time indicates how long it takes for a small puncture wound to stop bleeding. The platelet count reveals the number of circulating platelets in venous or arterial blood.

A client is admitted for treatment of Prinzmetal's angina. When developing the care plan, the nurse keeps in mind that this type of angina is triggered by: 1. activities that increase myocardial oxygen demand. 2. an unpredictable amount of activity. 3. coronary artery spasm. 4. the same type of activity that caused previous angina episodes.

Correct Answer: 3 RATIONALES: Prinzmetal's angina results from coronary artery spasm. Activities that increase myocardial oxygen demand may trigger angina of effort. An unpredictable amount of activity may precipitate unstable angina. Worsening angina is brought on by the same type or level of activity that caused previous angina episodes; however, anginal pain is increasingly severe.

A client with a suspected diagnosis of acute myocardial infarction is admitted to the coronary care unit. To help confirm the diagnosis, the physician orders serial enzyme tests. Increased serum levels of the isoenzyme creatinine kinase of myocardial muscle (CK-MB), found only in cardiac muscle, can be detected how soon after the onset of chest pain? 1. 30 minutes to 1 hour 2. 2 to 3 hours 3. 4 to 6 hours 4. 12 to 18 hours

Correct Answer: 3 RATIONALES: Serum CK-MB levels can be detected 4 to 6 hours after the onset of chest pain. These levels peak within 12 to 18 hours and return to normal within 3 to 4 days.

An anxious client who suffered an acute myocardial infarction is transferred from the coronary care unit (CCU) to the telemetry unit. The client asks the charge nurse if he can have the same nurse care for him every day. How should the charge nurse respond? 1. "Different nurses will be assigned to you each day to avoid your becoming dependent on one nurse." 2. "It is important for you to receive care from a variety of nurses so you can evaluate your care." 3. "We will try to assign you the same nurse as often as possible." 4. "It is our policy to rotate client care assignments to ensure quality care for everyone."

Correct Answer: 3 RATIONALES: The charge nurse should try to accommodate the client's wishes by assigning him a familiar nurse whenever possible. This should help decrease the client's anxiety. Preventing dependency should not be a concern; allaying his anxiety should. The client should not be concerned with evaluating the quality of care rendered by multiple nurses. Providing continuity of care helps ensure quality care.

A client with angina pectoris must learn how to reduce risk factors that exacerbate this condition. When developing the client's care plan, the nurse should include which expected outcome? 1. "Client will verbalize an understanding of the need to call the physician if acute pain lasts more than 2 hours." 2. "Client will verbalize the intention to avoid exercise." 3. "Client will verbalize the intention to stop smoking." 4. "Client will verbalize an understanding of the need to restrict dietary fat, fiber, and cholesterol."

Correct Answer: 3 RATIONALES: The client with angina pectoris should stop smoking at once because smoking increases the blood carboxyhemoglobin level; this, in turn, reduces the heart's oxygen supply and may induce angina. The client must seek immediate medical attention if chest pain doesn't subside after three nitroglycerin doses taken 10 to 15 minutes apart; serious myocardial damage or even sudden death may occur if chest pain persists for 2 hours. To improve coronary circulation and promote weight management, the client should get regular daily exercise. The client should eat plenty of fiber, which may decrease serum cholesterol and triglyceride levels and minimize hypertension, in turn reducing the risk for atherosclerosis (which plays a role in angina).

The nurse is caring for a client with acute pulmonary edema. To immediately promote oxygenation and relieve dyspnea, the nurse should: 1. administer oxygen. 2. have the client take deep breaths and cough. 3. place the client in high Fowler's position. 4. perform chest physiotherapy.

Correct Answer: 3 RATIONALES: The high Fowler's position will initially promote oxygenation in the client and relieve shortness of breath. Additional measures include administering oxygen to increase content in the blood. Deep breathing and coughing will improve oxygenation postoperatively but may not immediately relieve shortness of breath. Chest physiotherapy results in expectoration of secretions, which isn't the primary problem in pulmonary edema.

A client hospitalized for treatment of hypertension is being prepared for discharge. The nurse should be sure to cover which teaching topic? 1. Maintaining a low-potassium diet 2. Skipping a medication dose if dizziness occurs 3. Maintaining a low-sodium diet 4. Receiving I.V. antihypertensive medications

Correct Answer: 3 RATIONALES: The nurse must teach the hypertensive client how to modify the diet to restrict sodium and saturated fats. In addition to teaching about adverse effects of prescribed antihypertensives, the nurse must discuss the actions and dosages of these drugs. Option 1 is incorrect because a client receiving antihypertensives also may take a diuretic as part of the drug regimen and thus may require dietary potassium supplements and high-potassium foods to avoid electrolyte disturbances. Instead of skipping medication if dizziness occurs (option 2), the client should notify the physician of this symptom. The client receiving antihypertensives at home takes them by mouth, not I.V., making option 4 incorrect.

The nurse should monitor a client receiving lidocaine (Xylocaine) for toxicity. Which signs or symptoms in a client would suggest lidocaine toxicity? 1. Nausea and vomiting 2. Pupillary changes 3. Confusion and restlessness 4. Hypertension

Correct Answer: 3 RATIONALES: The nurse should observe for signs of lidocaine toxicity, such as confusion and restlessness. Nausea and vomiting may occur with oral administration of mexiletine (Mexitil) or tocainide (Tonocard), other class IB drugs (lidocaine isn't administered orally). Pupillary changes and hypertension aren't signs of lidocaine toxicity, although visual changes and hypotension may occur as adverse reactions to class IB drugs.

The nurse is educating a client who's at risk for coronary artery disease (CAD). The nurse tells the client that CAD has many risk factors. Risk factors that can be controlled or modified include: 1. gender, obesity, family history, and smoking. 2. inactivity, stress, gender, and smoking. 3. obesity, inactivity, diet, and smoking. 4. stress, family history, and obesity.

Correct Answer: 3 RATIONALES: The risk factors for coronary artery disease that can be controlled or modified include obesity, inactivity, diet, stress, and smoking. Gender and family history are risk factors that can't be controlled.

When caring for a client with dysfunction in the conduction system, at which period would the nurse note that cells are resistant to stimulation?

During the refractory period

The client with a diagnosis of heart failure reports frequently awakening during the night with the need to urinate. The nurse offers what explanation?

Edema is collected in dependent extremities during the day; at night when the client lays down, it is reabsorbed into the circulation and excreted by the kidneys.

The client with a diagnosis of heart failure reports frequently awakening during the night with the need to urinate. The nurse offers which explanation?

Edema is collected in dependent extremities during the day; at night when the client lays down, it is reabsorbed into the circulation and excreted by the kidneys.

Atrial rate of 300 to 400

Electrocardiogram (ECG) characteristics of atrial fibrillation include which of the following?

Which diagnostic study best evaluates different medications ability to restore normal heart rhythm? a) Electrophysiology study b) Echocardiogram c) Electrocardiogram (ECG) d) Elective electrical cardioversion

Electrophysiology study An electrophysiology study is a procedure that enables the physician to examine the electrical activity of the heart, produce actual dysrhythmias, and determine the best method for care. Cardioversion uses synchronized electricity to change the rhythm pattern. Electrocardiogram and echocardiograms provide diagnostic information.

Within the heart, several structures and several layers all play a part in protecting the heart muscle and maintaining cardiac function. The inner layer of the heart is composed of a thin, smooth layer of cells, the folds of which form heart valves. What is the name of this layer of cardiac tissue

Endocardium

You are monitoring the results of laboratory tests performed on a client admitted to the cardiac ICU with a diagnosis of myocardial infarction. Which test would you expect to show elevated levels?

Enzymes

The nurse is assessing a patient who complains of feeling "light-headed." When obtaining orthostatic vital signs, what does the nurse determine is a significant finding?

A heart rate of more than 20 bpm above the resting rate

A nurse assessing a client who underwent cardiac catheterization finds the client lying flat on the bed. His temperature is 99.8° F (37.7° C). His blood pressure is 104/68 mm Hg. His pulse rate is 76 beats/minute. She detects weak pulses in the leg distal to the puncture site. Skin on the leg is cool to the touch. The puncture site is dry, but swollen. What is the client most likely developing?

A hematoma at the puncture site

What is BNP?

A neurohormone that responds to volume overload in the heart by acting as a diuretic and vasodilator

10 Minutes

A nurse has come upon an unresponsive, pulseless victim. She has placed a 911 call and begins CPR. The nurse understands that if the patient has not been defibrillated within which time frame, the chance of survival is close to zero?

Place the client in high Fowler's position.

A nurse is caring for a client with acute pulmonary edema. To immediately promote oxygenation and relieve dyspnea, the nurse should:

"The only difference is the rate, which will be below 60 bpm in sinus bradycardia."

A nursing student is caring for one of the nurse's assigned cardiac clients. The student asks, "How can I tell the difference between sinus rhythm and sinus bradycardia when I look at the EKG strip" The best reply by the nurse is which of the following?

A, B, C

A patient had a cardiac catheterization and is now in the recovery area. What nursing interventions should be included in the plan of care? (Select all that apply). A. assessing the peripheral pulses in the affected extremity B. checking the insertion site for hematoma formation C. evaluating temperature and color in the affected extremity D. assisting the patient to the bathroom after the procedure E. assessing vital signs every 8 hours

Decrease myocardial contractility.

A patient has a high magnesium level. Identify how hypermagnesemia affects cardiac function.

The area of the heart that is located at the third intercostal (IC) space to the left of the sternum is which of the following?

Erb's point

Decreased pulse pressure reflects which of the following? A) Reduced stroke volume B) Reduced distensibility of the arteries C) Tachycardia D) Elevated stroke volume

A) Reduced stroke volume

The nurse is reviewing discharge instructions with a patient who underwent a left groin cardiac catheterization 8 hours ago. Which of the following instructions should the nurse include?

"Do not bend at the waist, strain, or lift heavy objects for the next 24 hours.

The nurse reviews discharge instructions with a client who underwent a left groin cardiac catheterization 8 hours ago. Which instructions should the nurse include?

"Do not bend at the waist, strain, or lift heavy objects for the next 24 hours."

The client is being prepared for echocardiography when he asks the nurse why he needs to have this test. What would be the nurse's best response?

"Echocardiography is a way of determining the functioning of the left ventricle of your heart."

Your client is being prepared for echocardiography when he asks you why he needs to have this test. What would be your best response

"Echocardiography is a way of determining the functioning of the left ventricle of your heart."

Your client is being prepared for echocardiography when they ask you why they need to have this test. What would be your best response?

"Echocardiography is a way of determining the functioning of the left ventricle of your heart."

A nurse is performing discharge teaching with a client who has an implantable cardioverter defibrillator (ICD) placed. Which client statement indicates effective teaching? a) "I can't wait to get back to my football league." b) "I need to stay at least 10' away from the microwave." c) "I have an appointment for magnetic resonance imaging of my knee scheduled for next week." d) "I'll keep a log of each time my ICD discharges."

"I'll keep a log of each time my ICD discharges." The client stating that he should keep a log of all ICD discharges indicates effective teaching. This log helps the client and physician identify activities that may cause the arrhythmias that make the ICD discharge. He should also record the events right before the discharge. Clients with ICDs should avoid contact sports such as football. They must also avoid magnetic fields, which could permanently damage the ICD. Household appliances don't interfere with the ICD.

S4 sounds like "________" and is associated with ________.

"LUB-lub-dub" ("tenne-ssee") stiff ventricle, ventricular hypertrophy (seen in HTN, cardiomyopathies, aortic stenosis, etc.)

A nurse is preparing a client for a scheduled Adenocard (adenosine) stress test. Which of the following statements by the client indicates a need for further teaching?

"My wife is bringing me a cup of coffee to drink before the test."

S3 sounds like "________" and is associated with ________.

"lub-dub-DUB" ("ken-tucky") HF

A patient had a cardiac catheterization and is now in the recovery area. What nursing interventions should be included in the plan of care? (Select all that apply.)

-Checking the insertion site for hematoma formation -Evaluating temperature and color in the affected extremity -Assessing the peripheral pulses in the affected extremity

Left atrium

Within the physiology of the heart, each chamber has a particular role in maintaining cellular oxygenation. Which chamber of the heart is responsible for receiving oxygenated blood from the lungs?

The nurse understands that assessment of ejection fraction (EF) is important to determine myocardial contractility. The nurse would expect the following EF to indicate the possibility of a myocardial infarction. With a stroke volume (SV) of 70 mL, the EF would be:

28 mL

A BNP level greater than ___ pg/ml is commonly associated with mild heart failure.

51 pg/ml

What is the normal serum calcium level?

8.9-10.3

• Carry a card identifying yourself as a pacemaker recipient. • Avoid large magnetic fields.

A client has had a pacemaker inserted and is ready for discharge. The nurse is providing education about pacemaker safety. Which of the following are items that the nurse will be sure to address? Choose all that apply.

Weighing the client daily at the same time each day.

A client is at risk for excess fluid volume. Which nursing intervention ensures the most accurate monitoring of the client's fluid status?

Pulmonary Congestion

A client is being assessed for his semiannual examination and you hear crackles bilaterally in his lungs. Which of the following could be a cause of crackles in the bases of his lungs?

Bibasilar crackles

A client is returning from the operating room after inguinal hernia repair. The nurse notes that he has fluid volume excess from the operation and is at risk for left-sided heart failure. Which sign or symptom indicates left-sided heart failure?

Failure to capture

A client receives a pacemaker to treat a recurring arrhythmia. When monitoring the cardiac rhythm strip, the nurse observes extra pacemaker spikes that don't precede a beat. Which condition should the nurse suspect?

C

A 52-year-old female patient is going through menopause and asks the nurse about estrogen replacement for its cardioprotective benefits. What is the best response by the nurse? A. "That's a great idea. You don't want to have a heart attack" B. "Current research determines taht the replacement of estrogen will protect a woman after she goes into menopause" C. "Current evidence indicates that estrogen is ineffective as a cardioprotectant; estrogen is actually potentially harmful and is no longer a recommended therapy." D. "You need to research it and determine what you want to do"

The following clients are in need of exercise electrocardiography. Which client would the nurse indicate as most appropriate for a drug-induced stress test?

A 55-year-old recovering from a fall and broken femur

Gradual, unexplained weight gain.

A 63-year-old accountant was admitted to the cardiac ICU with full-blown pulmonary edema. After he was revived, the nurse discusses his symptoms with the client and his wife. What is a typical, subtle symptom that communicates right-sided heart failure?

Low blood pressure

A patient is receiving anticoagulant therapy. The nurse should be alert to potential signs and symptoms of external or internal bleeding, as evidenced by which of the following?

B

A patient tells the nurse, "I was straining to have a bowel movement and felt like I was going to faint. I took my pulse and it was so slow." What does the nurse understand occurred with this patient? A. the patient may have had a myocardial infarction B. the patient had a vagal response C. the patient was anxious about being constipated D. the patient may have an abdominal aortic aneurysm

The nurse is assigned to care for the following patients admitted to a telemetry unit. Which patient should the nurse assess first? a) A patient diagnosed with new onset of atrial fibrillation requiring scheduled IV Cardizem b) A patient who received elective cardioversion 1 hour ago with a heart rate (HR) is 115 bpm c) A patient returned from an electrophysiology (EP) procedure 2 hours ago complaining of constipation d) A patient whose implantable cardioverter defibrillator (ICD) fired twice on the prior shift requiring amiodarone IV

A patient whose implantable cardioverter defibrillator (ICD) fired twice on the prior shift requiring amiodarone IV The patient's ICD that has fired on the previous shift should be seen first. This patient is in need of antidysrhythmic medication and this is the priority intervention. The remaining patients should be seen after this patient and are in no acute distress.

A

A patient's heart rate is observed to be 140 bpm on the monitor. The nurse knows that the patient is at risk for what complication? A. myocardial ischemia B. a pulmonary embolism C. right-sided heart failure D. a stroke

The nurse admits a 52-year-old woman with a medical diagnosis of "rule out MI." The client is very frightened and expresses surprise that a woman would have heart problems. Which of the following responses by the nurse would be most appropriate?

A woman's heart is smaller and has smaller arteries that become occluded more easily.

You are doing an admission assessment on a client who is having outpatient testing done for cardiac problems. What should you ask this client during your assessment? A) "Have you had any episodes of dizziness or fainting?" B) "Have you had any episodes when you are to nauseous?" C) "Have you had any episodes of mottling in your hands?" D) "Have you had any episodes of pain radiating into your lower extremities?"

A) "Have you had any episodes of dizziness or fainting?"

The nurse is taking a health history from a client admitted with the medical diagnosis of cardiovascular disease (CVD). Identify which of the following symptoms indicate CVD: A) Chest pain, weight gain, fatigue B) Petechiae, ascites, constipation C) Fatigue, ecchymosis, confusion D) Dizziness, rash, extra-ocular eye movements

A) Chest pain, weight gain, fatigue

The nurse prepares to auscultate heart sounds. Which nursing interventions would be most effective to assist with this procedure? a) Explain to the client that the nurse will be listening to different areas of the chest and may listen for a long time, but that does not mean that anything abnormal is heard. b) Ask the client to take deep breaths through his mouth while the nurse auscultates heart sounds. c) Ask the client to sit on the edge of the bed and hold his breath while the nurse listens. d) Insist that the family members leave the room if they must speak to each other while the nurse is auscultating heart sounds.

A) Explain to the client that the nurse will be listening to different areas of the chest and may listen for a long time, but that does not mean that anything abnormal is heard. Explanation: During auscultation the client remains supine and the room should be as quiet as possible while the nurse listens to heart sounds. The client should breathe quietly during the examination.

Which of the following would be a factor that may decrease myocardial contractility? A) sympathetic activity b) Administration of digoxin (Lanoxin) c) Acidosis d)Alkalosis

Acidosis

Sodium

Admission lab values on a patient admitted with congestive heart failure are as follows: potassium 3.4 mEq/L; sodium 148 mEq/L; calcium 9.8 mg/dL; and magnesium 1.5 mEq/L. Which lab value is abnormal?

Decreased cardiac output and decreased systolic and diastolic blood pressure

After evaluating a client for hypertension, a physician orders atenolol (Tenormin), 50 mg P.O. daily. Which therapeutic effect should atenolol have?

The nurse is awaiting results of cardiac biomarkers for a patient with severe chest pain. The nurse would identify which cardiac biomarker as remaining elevated the longest when myocardial damage has occurred?

After myocardial injury, these biomarkers rise early (within 3 to 4 hours), peak in 4 to 24 hours, and remain elevated for 1 to 3 weeks. These early and prolonged elevations may make very early diagnosis of acute myocardial infarction (MI) possible and allow for late diagnosis in patients who have delayed seeking care for several days after the onset of acute MI symptoms. CK-MB returns to normal within 3 to 4 days. Myoglobin returns to normal within 24 hours. BNP is not considered a cardiac biomarker. It is a neurohormone that responds to volume overload in the heart by acting as a diuretic and vasodilator.

What category of medication is Methylprednisolone (Solu-Medrol)?

Anti-histamine

The client states, "My doctor says that because I am now taking this water pill, I need to eat more foods that contain potassium. Can you give me some ideas about what foods would be good for this?" The nurse's appropriate response is which of the following?

Apricots, dried peas and beans, dates

The client states, "My doctor says that because I am now taking this water pill, I need to eat more foods that contain potassium. Can you give me some ideas about what foods would be good for this?" The nurse's appropriate response is which of the following? a) Apricots, dried peas and beans, dates b) Asparagus, blueberries, green beans c) Cranberries, apples, popcorn d) Bok choy, cooked leeks, alfalfa sprouts

Apricots, dried peas and beans, dates Apricots, dried peas and beans, dates, and kiwi contain high amounts of potassium. The other foods listed contain minimal amounts.

The nurse is caring for a client on a monitored telemetry unit. During morning assessment, the nurse notes abnormal ECG waves on the telemetry monitor. Which action would the nurse do first?

Assess the client When a nurse notes an abnormal rhythm on a telemetry monitor, the first action is to assess the client. After client assessment, the nurse is able to make an informed decision on the next nursing action.

Where is the pulmonic area located?

At the second IC space to the left of the sternum

Where is the aortic area located?

At the second IC space to the right of the sternum

A patient has undergone a cardiac catheterization. He is to be discharged today. What information should the nurse emphasize during discharge teaching?

Avoid heavy lifting for the next 24 hours

The nurse is providing discharge education for the client going home after a cardiac catheterization. Which of the following would be important information to give this client?

Avoid tub baths, but shower as desired.

The nurse is reviewing the morning laboratory test results for a client with cardiac problems. Which of the following would the nurse regard as a priority to report to the physician? A) Ca++ 9 mg/dL B) K+ 3.1 mEq/L C) Mg++ 2 mE/L D) Na+ 140 mEq/L

B) K+ 3.1 mEq/L

A nurse is checking laboratory values on a client who has crackles in the lower lobes, 2+ pitting edema, and dyspnea with minimal exertion. Which laboratory value does the nurse expect to be abnormal?

B-type natriuretic peptide (BNP)

What neurohormone is released from the ventricles when the ventricles experience increased pressure and stretch, such as in heart failure?

BNP

The nurse is caring for a client with an elevated blood pressure and no previous history of hypertension. At 0900, the blood pressure was 158/90 mm Hg. At 0930, the blood pressure is 142/82 mm Hg. The nurse is most correct when relating the fall in blood pressure to which structure?

Baroreceptors

The nurse prepares to apply ECG electrodes to a male client who requires continuous cardiac monitoring. Which action should the nurse complete to optimize skin adherence and conduction of the heart's electrical current?

Clip the client's chest hair prior to applying the electrodes.

A nurse is preparing a client for a scheduled Adenocard (adenosine) stress test. Which of the following statements by the client indicates a need for further teaching?: A) "The effects of this medication will wear off quickly." B) "I may feel some flushing or nausea with this medication." C) "My wife is bringing me a cup of coffee to drink before the test." D) "The medication will have an effect on my heart similar to exercise."

C) "My wife is bringing me a cup of coffee to drink before the test."

A client is seen in the emergency department and reports left arm pain, fatigue, palpitations, and shortness of breath. Which of the following conditions would the nurse suspect? A) Diabetes insipidus B) Renal failure C) Acute coronary syndrome D) Diabetes mellitus

C) Acute coronary syndrome

The nurse is assessing the client newly prescribed Lasix 20mg daily for 3+ pitting edema. To evaluate the effectiveness of diuretic therapy, which of the following would be documented?: A) Weight B) Urine output C) Edema D) Blood pressure

C) Edema

The nurse is preparing to apply ECG electrodes to a male patient who requires continuous cardiac monitoring. Which of the following should the nurse complete to optimize skin adherence and conduction of the heart's electrical current?

Clip the patient's chest hair prior to applying the electrodes

The nurse accompanies a client to an exercise stress test. The client can achieve the "target heart rate," but the ECG leads show an ST-segment elevation. The nurse recognizes this as a "positive" stress test, and will begin to prepare the client for which of the following procedures?

Cardiac catheterization

A nurse is caring for a client with a history of cardiac disease and type 2 diabetes. The nurse is closely monitoring the client's blood glucose level. Which medication is the client most likely taking? a) Procainamide (Procan) b) Carvedilol (Coreg) c) Diltiazem (Cardizem) d) Amiodarone (Cordarone)

Carvedilol (Coreg) The nurse must monitor blood glucose levels closely in clients with type 2 diabetes who are taking beta-adrenergic blockers such as carvedilol, because beta-adrenergic blockers may mask the signs of hypoglycemia. The nurse should monitor QRS duration in clients taking procainamide and pulmonary function in clients taking amiodarone (because the drug may cause pulmonary fibrosis). Diltiazem may cause an increased PR interval or bradycardia.

The nurse is caring for a patient with an intra-arterial BP monitoring device. The nurse recognizes the most preventable complication associated with hemodynamic monitoring includes which of the following?

Catheter related bloodstream infections (CRBSI)

The nurse is caring for a client with a damaged tricuspid valve. The nurse knows that the tricuspid valve is held in place by which of the following?

Chordae tendineae

You are caring for a client with a damaged tricuspid valve. You know that the tricuspid valve is held in place by which of the following?

Chordae tendineae

The ability of the cardiac muscle to shorten in response to an electrical impulse is termed which of the following?

Contractility

What is the term for the ability of the cardiac muscle to shorten in response to an electrical impulse?

Contractility

A hospitalized client experiences digoxin- (Lanoxin-) induced premature ventricular contractions (PVCs). Which type of effect do such contractions represent? 1. Toxic 2. Secondary 3. Iatrogenic 4. Idiosyncratic

Correct Answer: 3 RATIONALES: Digoxin-induced PVCs are iatrogenic because the drug is mimicking a cardiac disorder. Because the client is experiencing an apparent pathological disorder, this effect isn't considered toxic, secondary, or idiosyncratic.

A client is prescribed hydralazine for blood pressure management. The nurse is teaching the client about hydralazine therapy. When should the client take his hydralazine? 1. Upon arising in the morning 2. Just before bedtime 3. On an empty stomach 4. With food

Correct Answer: 4 RATIONALES: Oral hydralazine should be taken with food to promote absorption.

In caring for a client with vasovagal syncope, the nurse should know that the associated temporary loss of consciousness is most often related to: 1. vestibular dysfunction. 2. sudden vascular fluid shifting. 3. postural hypotension. 4. bradyrhythmia.

Correct Answer: 4 RATIONALES: Parasympathetic hyperactivity leading to sudden hypotension secondary to bradyrhythmia causes vasovagal syncope. That is, bradyrhythmia leads to cerebral ischemia which, in turn, leads to syncope. Vasovagal syncope isn't caused by vestibular (inner ear) dysfunction, postural hypotension, or vascular fluid shifting.

In a client with chronic bronchitis, which sign would lead the nurse to suspect right-sided heart failure? 1. Cyanosis of the lips 2. Bilateral crackles 3. Productive cough 4. Leg edema

Correct Answer: 4 RATIONALES: Right-sided heart failure is characterized by signs of circulatory congestion, such as leg edema, neck vein distention, and hepatomegaly. Left-sided heart failure is characterized by circumoral cyanosis, crackles, and a productive cough.

Eighty six-year-old Susan Matthews has been rushed to the ED with pulmonary edema during your shift. She is going to need oxygen immediately. Which oxygen delivery system do you use first?

Correct response: *A mask Explanation: Because pulmonary edema can be fatal, lung congestion needs to be relieved as quickly as possible. Supplemental oxygen is one of the first tools used to fight pulmonary edema, in which case a mask, rather than nasal cannula, is needed to deliver the maximum percentages of oxygen. Intubation is reserved for when respiratory failure occurs. Mechanical ventilation is applied once respiratory failure occurs.

A patient has missed 2 doses of digitalis (Digoxin). What laboratory results would indicate to the nurse that the patient is within therapeutic range?

Correct response: 2.0 mg/mL Explanation: For many years, digitalis (digoxin) was considered an essential agent for the treatment of HF, but with the advent of new medications, it is not prescribed as often. Digoxin increases the force of myocardial contraction and slows conduction through the atrioventricular node. It improves contractility, increasing left ventricular output

On his return to the cardiac step-down unit after his diagnostic procedure, a client awaits the report from his cardiologist. As the client's nurse, you review the process of measuring ejection fraction and explain to the client that it measures the percentage of blood the left ventricle ejects upon contraction. What is the typical percentage of blood a healthy heart ejects?

Correct response: 55% Explanation: Normally, a healthy heart ejects 55% or more of the blood that fills the left ventricle during diastole.

The clinic nurse caring for a client with a cardiovascular disorder is performing an assessment of the client's pulse. Which of the following steps is involved in determining the pulse deficit?

Count the heart rate at the apex

A patient recently diagnosed with pericarditis asks his nurse to explain what area of his heart is involved. The nurse tells the patient that the pericardium, which is inflamed, is the:

Thin fibrous sac that encases the heart

A student nurse is to perform a cardiac assessment for a client and asks the instructor why the aortic valve closure is best heard on the right side of the sternum. The best response of the nurse is which of the following? A) "The aortic valve is located near the apex of the heart, which is on the right side." B) "The aortic valve is located on the right side of the heart." C) "The aortic valve is located near the base of the heart on the right side." D) "The aortic arch causes the closure of the aortic valve to be heard best on the right side of the sternum."

D) "The aortic arch causes the closure of the aortic valve to be heard best on the right side of the sternum."

You are caring for a client with a damaged tricuspid valve. You know that the tricuspid valve is held in place by which of the following? A) Semilunar tendineae B) Atrioventricular tendons C) Papillary tendons D) Chordae tendineae

D) Chordae tendineae

The nurse is preparing to apply ECG electrodes to a male patient who requires continuous cardiac monitoring. Which of the following should the nurse complete to optimize skin adherence and conduction of the heart's electrical current?: A)Clean the patient's chest with alcohol prior to application of the electrodes. B)Apply baby powder to the patient's chest prior to placing the electrodes. C) Once the electrodes are applied, change them every 72 hours. D) Clip the patient's chest hair prior to applying the electrodes.

D) Clip the patient's chest hair prior to applying the electrodes

The clinic nurse caring for a client with a cardiovascular disorder is performing an assessment of the client's pulse. Which of the following steps is involved in determining the pulse deficit? A) Count the radial pulse for 20 to 25 seconds. B) Calculate the palpated volume. C) Calculate the pauses between pulsations. D) Count the heart rate at the apex

D) Count the heart rate at the apex

The nurse is administering a beta blocker to a patient in order to decrease automaticity. Which medication will the nurse administer? A) Cardizem B) Cordarone C) Rythmol D) Lopressor

D) Lopressor

The client is admitted for a scheduled cardiac catheterization. On the morning of the procedure, while assessing the client's morning laboratory values, the nurse notes a blood urea nitrogen (BUN) of 34 mg/dL and a creatinine of 4.2 mg/dL. The nurse makes it a priority to notify the physician for which of the following reasons? A) The client is over-hydrated, which puts him at risk for heart failure during the procedure. B) The client is at risk for bleeding. C) These values show a risk for dysrhythmias. D)The client is at risk for renal failure due to the contrast agent that will be given during the procedure.

D)The client is at risk for renal failure due to the contrast agent that will be given during the procedure.

After evaluating a client for hypertension, a physician orders atenolol (Tenormin), 50 mg P.O. daily. Which therapeutic effect should atenolol have? a) Decreased peripheral vascular resistance b) Decreased cardiac output and decreased systolic and diastolic blood pressure c) Increased cardiac output and increased systolic and diastolic blood pressure d) Decreased blood pressure with reflex tachycardia

Decreased cardiac output and decreased systolic and diastolic blood pressure As a long-acting, selective beta1-adrenergic blocker, atenolol decreases cardiac output and systolic and diastolic blood pressure; however, like other beta-adrenergic blockers, it increases peripheral vascular resistance at rest and with exercise. Atenolol may cause bradycardia, not tachycardia.

A patient has a high magnesium level. Identify how hypermagnesemia affects cardiac function.

Decreases myocardial contractility

A cardiac patient with a magnesium lab result of 2.5 mEq/L would most likely evidence which of the following?

Depressed myocardial contractility

A patient has been diagnosed with congestive heart failure (CHF). The physician has ordered a medication to enhance contractility. The nurse would expect which medication to be ordered for the patient?

Digoxin (Lanoxin)

A patient is being scheduled for a stress test. The patient is unable to exercise during the test. The nurse would include information about which medication used for pharmacologic stress testing?

Dipyridamole If the patient is unable to exercise, a pharmacologic stress test is performed by injecting a vasodilating agent, dipyridamole (Persantine) or adenosine (Adenocard), to mimic the physiologic effects of exercise. The stress test may be combined with an echocardiogram or radionuclide imaging techniques to examine myocardial function during exercise and rest. Digoxin would not be used for stress testing. Thallium 201 and Cardiolite are radioisotopes used in myocardial perfusion scanning. (less)

The nurse is caring for a patient in the ICU diagnosed with coronary artery disease (CAD). Which of the following assessment data indicates the patient is experiencing a decrease in cardiac output?

Disorientation, 20 mL of urine over the last 2 hours

The nurse caring for a client who is suspected of having cardiovascular disease has a stress test ordered. The client has a co-morbidity of multiple sclerosis, so the nurse knows the stress test will be drug-induced. What drug will be used to dilate the coronary arteries?

Dobutamine

The higher the BNP the...

Increased the severity of heart failure

The nurse is caring for a client scheduled for a cardiac stress test at 11:00 a.m. When the nurse enters the client's room at 8:00 a.m., the client complains that he has had no breakfast and would at least like some coffee. The appropriate response by the nurse would be which of the following?

Explain that no food or drink is allowed for 4 hours before the stress test.

Which of the following is an early warning symptom of acute coronary syndrome (ACS) and heart failure (HF)?

Fatigue

The nurse auscultates the apex beat at which of the following anatomical locations?

Fifth intercostal space, midclavicular line

Identify which of the following as an age-related change associated with conduction system of the heart? a) Thrills b) Tachycardia c) Heart block d) Murmur

Heart block Age-related changes to the conduction system may include bradycardia and heart block. Age-related changes to the heart valves include the presence of a murmur or thrill.

During assessment of a 63-year-old retired mechanic, the nurse notes and documents an S3 heart sound. The nurse knows that this sound is an abnormal sound suggestive of:

Heart failure

The nurse is reviewing the results of the patient's echocardiogram and observes that the ejection fraction is 35%. The nurse anticipates that the patient will receive treatment for what condition?

Heart failure

During the auscultation of a patient's heart sounds, the nurse notes an S4. The nurse recognizes that an S4 is associated with which of the following?

Hypertensive heart disease Auscultation of the heart requires familiarization with normal and abnormal heart sounds. An extra sound just before S1 is an S4 heart sound, or atrial gallop. An S4 sound often is associated with hypertensive heart disease. A sound that follows S1 and S2 is called an S3 heart sound or a ventricular gallop. An S3 heart sound is often an indication of heart failure in an adult. In addition to heart sounds, auscultation may reveal other abnormal sounds, such as murmurs and clicks, caused by turbulent blood flow through diseased heart valves.

Heart block

Identify which of the following as an age-related change associated with conduction system of the heart?

One of your students asks what the consequences of uncorrected, left-sided heart failure would be. What would be your best response

If uncorrected, left-sided heart failure is followed by right-sided heart failure because the circulatory system is a continuous loop

The nurse reviews a patient's lab results and notes a serum calcium level of 7.9 mg/dL. The nurse knows that this reading can also be associated with which of the following?

Impaired myocardial contractility

The nurse is administering a beta blocker to a patient in order to decrease automaticity. Which medication will the nurse administer

Lopressor

The nurse is administering a beta blocker to a patient in order to decrease automaticity. Which medication will the nurse administer?

Lopressor

After a physical examination, the provider diagnosed a patient with a grade 4 heart murmur. During assessment, the nurse expects to hear a murmur that is

Loud and may be associated with a thrill sound similar to (a purring cat) Heart murmurs are characterized by location, timing, and intensity. A grading system is used to describe the intensity or loudness of a murmur. A grade 1 is very faint and difficult to describe, whereas a grade 6 is extremely loud. Refer to Box 12-3 in the text for a description of grades 1 to 6.

After a physical examination, the provider diagnosed a patient with a grade 4 heart murmur. During assessment, the nurse expects to hear a murmur that is:

Loud and may be associated with a thrill sound similar to a purring cat

When preparing a patient for a cardiac catheterization, the patient states that she has allergies to seafood. Which of the following medications may give to her prior to the procedure?

Methylprednisolone (Solu-Medrol)

The nurse is caring for a client anticipating further testing related to cardiac blood flow. Which statement, made by the client, would lead the nurse to provide additional teaching?

My niece thought that I would be ordered a magnetic resonance imaging even though I have a pacemaker. A magnetic resonance imaging (MRI) test is prohibited on clients with various metal devices within their body. External metal objects must be removed. All other options are correct statements not needing clarification.

A patient's heart rate is observed to be 140 bpm on the monitor. The nurse knows that the patient is at risk for what complication?

Myocardial ischemia

The nurse cares for a client with clubbing of the fingers and toes. The nurse should complete which action given these findings?

Obtain an oxygen saturation level

The nurse is caring for a patient with clubbing of the fingers and toes. The nurse should complete which of the following actions given these findings?

Obtain an oxygen saturation level

The nurse is providing discharge instructions to a client with unstable angina. The client is ordered Nitrostat 1/150 every 5 minutes as needed for angina. Which side effect, emphasized by the nurse, is common especially with the increased dosage?

Orthostatic hypotension

You are working on a telemetry unit. Your client was admitted with a cardiac event and is now on a cardiac monitor. You know a cardiac monitor reveals the heart's electrical but not its mechanical activity. How would you assess the mechanical activity of the client's heart?

Palpate a peripheral pulse

During assessment of a patient with chest pain, the nurse practitioner documents the following characteristics of chest pain: Sharp, substernal of intermittent duration, and radiating to the arms and back. According to the patient, the pain increases with inspiration and swallowing. The pain is alleviated when the patient sits upright. Based on these symptoms, the nurse suspects that the patient may be experiencing which of the following conditions?

Pericarditis

A nurse is caring for a client taking diltiazem (Cardizem) for arrhythmias. The nurse knows that diltiazem helps decrease arrhythmias by working during which phase of the cardiac action potential?

Phase 0 Diltiazem, a calcium channel blocker, blocks the influx of calcium into the cells during phase 0 of the cardiac action potential. This action causes the sinoatrial node and atrioventricular (AV) node to slow their response times, which results in slowed AV conduction, decreased ventricular depolarization, and arrhythmias. Diltiazem doesn't work during phase 1, 2, or 3 of the cardiac action potential. (less)

Which of the follow arteries carries deoxygenated blood?

Pulmonary artery

A client has a history of atrial fibrillation. To prevent a recurrence, the physician prescribes sustained-release procainamide hydrochloride (Procan SR), 500 mg P.O. every 6 hours. How soon after administering procainamide can the nurse expect the drug to reach its peak concentration? 1. 15 to 60 minutes 2. 45 to 60 minutes 3. 1½ to 2 hours 4. 3 to 4 hours

RATIONALES: Sustained-release procainamide preparations reach peak concentrations in 1½ to 2 hours. On the other hand, I.M. procainamide reaches peak concentrations in 15 to 60 minutes. The sustained-release form of quinidine (not procainamide) reaches a peak concentration level in 3 to 4 hours.

You are the clinic nurse doing assessments on your clients before they have outpatient diagnostic testing done. What would you document when assessing the client's pulse?

Rate, quality, and rhythm

Decreased pulse pressure reflects which of the following?

Reduced stroke volume

The nurse is caring for a client with nursing diagnosis of ineffective tissue perfusion. Which area of the heart would the nurse anticipate being compromised

Right ventricle

It is important for a nurse to understand cardiac hemodynamics. For blood to flow from the right ventricle to the pulmonary artery, what must occur?

Right ventricular pressure must be higher than pulmonary arterial pressure.

he nurse is assessing heart sounds in a patient with heart failure. An abnormal heart sound is detected early in diastole. How would the nurse document this?

S3

The nurse observes a client during an exercise stress test (bicycle). Which finding indicates a positive test and the need for further diagnostic testing?

ST-segment changes on the ECG

A patient needs additional information about her heart condition. The patient states to the nurse, "What is considered the pacemaker of the heart

The SA node

The nurse is discussing cardiac hemodynamics with a nursing student, who understands the the following formula: CO = HR X SV (cardiac output equals heart rate times stroke volume). The nursing student asks what determines heart rate. The correct response by the nurse is which of the following?

The autonomic nervous system controls the heart rate

You are evaluating the expected outcomes on a client who is recovering from a cardiac catheterization. What is an expected outcome that you would evaluate?

The client and family understands the discharge instructions

The nurse is preparing to administer adenosine (Adenocard) for a patient diagnosed with atrial flutter. Which of the following should be completed when giving this medication? a) The medication is followed by a rapid lactated Ringer's (LR) flush b) The medication is followed by a slow saline flush. c) The dose is administered rapid IV push. d) The dose is administered slow IV push.

The dose is administered by rapid IV push (1 to 2 seconds), followed with a rapid saline flush. LR is not used during administration of this medication.

D

The nurse if reviewing the results of the patient's echocardiogram and observes that the ejection fraction is 35%. The nurse anticipates that the patient will receive treatment for what condition? A. pulmonary embolism B. myocardial infarction C. pericarditis D. heart failure

B

The nurse is administering a beta blocker to a patient in order to decrease automaticity. Which medication will the nurse administer? A. cardizem B. lopressor C. cordarone D. rythmol

A

The nurse is assessing a patient who complains of feeling "light-headed". When obtaining orthostatic vital signs, what does the nurse determine is a significant finding? A. a heart rate of 20 bpm above the resting rate B. an unchanged systolic pressure C. an increase of 10 mm Hg blood pressure reading D. an increase of 5 mm Hg in diastolic pressure

A

The nurse is assessing a patient's blood pressure. What does the nurse document as the differences between the systolic and the diastolic pressure? A. pulse pressure B. auscultatory gap C. pulse deficit D. Korotkoff sound

D

The nurse is assessing a patient's electrocardiogram. What phase does the nurse determine is the resting phase before the next depolarization? A. phase 1 B. phase 2 C. phase 3 D. phase 4

C

The nurse is assessing heart sounds in a patient with heart failure. An abnormal heart sound is detected early in diastole. How would the nurse document this? A. S1 B. S2 C. S3 D. S4

Troponin T and I

The nurse is awaiting results of cardiac biomarkers for a patient with severe chest pain. The nurse would identify which cardiac biomarker as remaining elevated the longest when myocardial damage has occurred?

A

The nurse is caring for a patient with a diagnosis of pericarditis. Where does the nurse understand the inflammation is located? A. the thin fibrous sac encasing the heart B. the inner lining of the heart and valves C. the heart's muscle fibers D. the exterior layer of the heart

C

The nurse is educating a patient at risk for atherosclerosis. What nonmodifiable risk factors does the nurse identify for the patient? A. stress B. obesity C. positive family history D. hyperlipidemia

A patient tells the nurse, "I was straining to have a bowel movement and felt like I was going to faint. I took my pulse and it was so slow." What does the nurse understand occurred with this patient?

The patient had a vagal response

C

The patient has a heart rate of 72 bpm with a regular rhythm. Where does the nurse determine the impulse arises from? A. The AV node B. the Purkinje fibers C. the sinoatrial node D. the ventricles

The nurse in a cardiac clinic is taking vital signs of a 58-year-old man who is 3 months status post myocardial infarction (MI). While the physician is seeing the client, the client's spouse approaches the nurse and asks about sexual activity. "We are too afraid he will have another heart attack, so we just don't have sex anymore." The nurse's best response is which of the following?

The physiologic demands are greatest during orgasm and are equivalent to walking 3-4 mph on a treadmill

The patient has a heart rate of 72 bpm with a regular rhythm. Where does the nurse determine the impulse arises from?

The sinoatrial node

the patient has a heart rate of 72 bpm with a regular rhythm. Where does the nurse determine the impulse arises from?

The sinoatrial node

The client, an 83-year-old man, is admitted with heart faillure. The nurse is aware that education needed prior to discharge includes which of the following?

Try to avoid emotional stress and take part in mild physical stress only

A nurse is caring for a dying client following myocardial infarction. The client is experiencing apnea with a falling blood pressure of 60 per palpation. Which documentation of pulse quality does the nurse anticipate?

Weak pulse

When assessing a patient with left-sided heart failure, what would be noted on auscultation of lungs?

Wheezes with wet lung sounds

When in BNP released?

When the ventricles experience increased pressure and stretch, such as in heart failure

Coumadin

Which medication is indicated for the patient with atrial fibrillation who is at high risk for stroke?

Pulmonary artery

Which of the follow arteries carries deoxygenated blood?

Instruct the patient to restrict food and oral intake.

Which of the following nursing interventions is required to prepare a patient with cardiac dysrhythmia for an elective electrical cardioversion?

Avoid sources of electrical interference.

Which of the following postimplantation instructions must a nurse provide a patient with a permanent pacemaker?

PR interval

Which of the following tends to be prolonged on the electrocardiogram (ECG) during a first-degree atrioventricular (AV) block?

Right Ventricle

Within the physiology of the heart, each chamber has a particular role in maintaining cellular oxygenation. Which chamber of the heart is responsible for pumping blood to the lungs to be oxygenated?

Clients with recurrent life-threatening tachydysrhythmias.

You are caring for a client who has been admitted to have a cardioverter defibrillator implanted. You would know that implanted cardioverter defibrillators are used in what clients?

A nurse is caring for a client with heart failure. The nurse knows that the client has left-sided heart failure when he makes which statement?

You selected: "I sleep on three pillows each night." Correct Explanation: Orthopnea is a classic sign of left-sided heart failure. The client commonly sleeps on several pillows at night to help facilitate breathing. Swollen feet, ascites, and anorexia are signs of right-sided heart failure

Ronald is a 46-year-old who has developed congestive heart failure. He has to learn to adapt his diet and you are his initial counselor. Which of the following should you tell him to avoid?

You selected: Canned peas

Which of the following therapies are for patient who have advanced heart failure (HF) after all other therapies have failed?

You selected: Heart transplant Correct Explanation: Heart transplantation involves replacing a person's diseased heart with a donor heart. This is an option for advanced HF patients when all other therapies have failed. A ventricular access device, ICD, and cardiac resynchronization therapy would be tried prior to a heart transplant.

Your client is being prepared for echocardiography when he asks you why he needs to have this test. What would be your best response? a) "Echocardiography is a way of determining the functioning of the left ventricle of your heart." b) "This test can tell us a lot about your heart." c) "This test will find any congenital heart defects." d) "Echocardiography will tell your doctor if you have cancer of the heart."

a) "Echocardiography is a way of determining the functioning of the left ventricle of your heart." Echocardiography uses ultrasound waves to determine the functioning of the left ventricle and to detect cardiac tumors, congenital defects, and changes in the tissue layers of the heart. All answers are correct. Option C is the best answer because it addresses the client's question without making him anxious or minimizing the question.

The nurse in a cardiac clinic is taking vital signs of a 58-year-old man who is 3 months status post myocardial infarction (MI). While the physician is seeing the client, the client's spouse approaches the nurse and asks about sexual activity. "We are too afraid he will have another heart attack, so we just don't have sex anymore." The nurse's best response is which of the following? a) "The physiologic demands are greatest during orgasm and are equivalent to walking 3 to 4 miles per hour on a treadmill." b) "The medications will prevent your husband from having an erection." c) "It is usually better to just give up sex after a heart attack." d) "Having an orgasm is very strenuous and your husband must be in excellent physical shape before attempting it."

a) "The physiologic demands are greatest during orgasm and are equivalent to walking 3 to 4 miles per hour on a treadmill." The physiologic demands are greatest during orgasm. The level of activity is equivalent to walking 3 to 4 miles per hour on a treadmill. Erectile dysfunction may be a side effect of beta-blockers, but other medications may be substituted.

You are doing an admission assessment on a client who is having outpatient testing done for cardiac problems. What should you ask this client during your assessment? a) "Have you had any episodes of dizziness or fainting?" b) "Have you had any episodes when you are to nauseous?" c) "Have you had any episodes of mottling in your hands?" d) "Have you had any episodes of pain radiating into your lower extremities?"

a. "Have you had any episodes of dizziness or fainting?" Explanation: Ask if the client has episodes of dyspnea, dizziness, or fainting. Options B, C, and D are incorrect. Being nauseous, mottling of the hands, and pain radiating into the lower extremities are not indications of cardiac problems.

The nurse is taking a health history from a client admitted with the medical diagnosis of cardiovascular disease (CVD). Identify which of the following symptoms indicate CVD. a) Chest pain, weight gain, fatigue b) Petechiae, ascites, constipation c) Fatigue, ecchymosis, confusion d) Dizziness, rash, extra-ocular eye movements

a. Chest pain, weight gain, fatigue Explanation: Chest pain, weight gain, fatigue, dizziness, ascites, and confusion are all symptoms of CVD. Rash, extra-ocular eye movements, ecchymosis, and petechiae are not usually indicative of CVD

A nurse assessing a client who underwent cardiac catheterization finds the client lying flat on the bed. His temperature is 99.8° F (37.7° C). His blood pressure is 104/68 mm Hg. His pulse rate is 76 beats/minute. She detects weak pulses in the leg distal to the puncture site. Skin on the leg is cool to the touch. The puncture site is dry, but swollen. What is the most appropriate action for the nurse to take? a) Contact the physician and report her findings. b) Encourage the client to perform isometric leg exercise to improve circulation in his legs. c) Document her findings and recheck the client in 1 hour. d) Slow the I.V. fluid to prevent any more swelling at the puncture site.

a. Contact the physician and report her findings. Explanation: The client is probably developing a hematoma at the puncture site. The decreased pulses, swelling, and cool temperature in the leg are all classic signs that blood flow to that extremity is compromised. The nurse should notify the physician immediately to preserve the blood flow in the client's leg. Documenting findings and checking the client again in 1 hour, slowing the I.V. fluid, and encouraging the client to perform isometric leg exercises aren't appropriate actions for the nurse to take at this time.

The nurse is caring for an elderly client with left-sided heart failure. When auscultating lung sounds, which adventitious sound is expected? a) Crackles b) Coarseness c) Wheezes d) Rhonchi

a. Crackles Explanation: When the heart is pumping inefficiently, blood backs up into the pulmonary veins and lung tissue. Auscultation reveals a crackling sound. Possible wheezes and gurgles are also possibilities.

Within the heart, several structures and several layers all play a part in protecting the heart muscle and maintaining cardiac function. The inner layer of the heart is composed of a thin, smooth layer of cells, the folds of which form heart valves. What is the name of this layer of cardiac tissue? a) Endocardium b) Myocardium c) Pericardium d) Epicardium

a. Endocardium Explanation: The inner layer, the endocardium, is composed of a thin, smooth layer of endothelial cells. Folds of endocardium form the heart valves. The middle layer, the myocardium, consists of muscle tissue and is the force behind the heart's pumping action. The pericardium is a saclike structure that surrounds and supports the heart. The outer layer, the epicardium, is composed of fibrous and loose connective tissue.

The area of the heart that is located at the third intercostal (IC) space to the left of the sternum is which of the following? a) Erb's point b) Epigastric area c) Aortic area d) Pulmonic area

a. Erb's point Explanation: Erb's point is located at the third IC space to the left of the sternum. The aortic area is located at the second IC space to the right of the sternum. The pulmonic area is at the second IC space to the left of the sternum. The epigastric area is located below the xiphoid process.

The nurse reviews a patient's lab results and notes a serum calcium level of 7.9 mg/dL. The nurse knows that this reading can also be associated with which of the following? a) Impaired myocardial contractility b) Increased risk of heart block c) Inclination to ventricular fibrillation d) Enhanced sensitivity to digitalis

a. Impaired myocardial contractility Explanation: Normal serum calcium is 8.9 to 10.3 mg/dL. A reading of 7.9 is below normal. Hypocalcemia is associated with slow nodal functioning and impaired myocardial contractility, which can increase the risk of heart failure.

A nurse working in a cardiac step-down unit understands that the following drugs can affect the contractility of the heart. The nurse recognizes that contractility is depressed by which of the following drugs? a) Lopressor b) Intropin c) Dobutrex d) Lanoxin

a. Lopressor Contractility is depressed by beta-adrenergic blocking medications. The other choices all enhance contractility.

A patient's heart rate is observed to be 140 bpm on the monitor. The nurse knows that the patient is at risk for what complication? a) Myocardial ischemia b) A stroke c) Right-sided heart failure d) A pulmonary embolism

a. Myocardial ischemia Explanation: As heart rate increases, diastolic time is shortened, which may not allow adequate time for myocardial perfusion. As a result, patients are at risk for myocardial ischemia (inadequate oxygen supply) during tachycardias (heart rate greater than 100 bpm), especially patients with coronary artery disease.

You are the clinic nurse doing assessments on your clients before they have outpatient diagnostic testing done. What would you document when assessing the client's pulse? a) Rate, quality, and rhythm b) Rate, rhythm, and volume c) Quality, volume, and rate d) Pressure, rate, and rhythm

a. Rate, quality, and rhythm Explanation: Assess apical and radial pulses, noting rate, quality, and rhythm. Pulse quality and volume are not assessed in this instance.

Admission lab values on a patient admitted with congestive heart failure are as follows: potassium 3.4 mEq/L; sodium 148 mEq/L; calcium 9.8 mg/dL; and magnesium 1.5 mEq/L. Which lab value is abnormal? a) Sodium b) Magnesium c) Potassium d) Calcium

a. Sodium Explanation: The normal sodium level is 135 to 145 mEq/L. Potassium ranges from 3.3 to 4.9 mEq/L. The normal calcium level is 8.9 to 10.3 mg/dL. Magnesium levels range from 1.3 to 2.2 mEq/L.

A nurse is preparing a client for magnetic resonance imaging (MRI) to confirm or rule out a spinal cord lesion. During the MRI scan, which action would pose a threat to the client? a) The client wears a watch and wedding band. b) The client asks questions. c) The client hears thumping sounds. d) The client lies still.

a. The client wears a watch and wedding band. Explanation: During an MRI, the client should wear no metal objects, such as jewelry, because the strong magnetic field can pull on them, causing injury to the client and (if they fly off) to others. The client must lie still during the MRI, but he can talk to those performing the test by way of the microphone inside the scanner tunnel. The client should hear thumping sounds, which are caused by the sound waves thumping on the magnetic field.

The nursing instructor asks the student what side effects a client may experience when undergoing a magnetic resonance imaging (MRI) test. What should the student respond

adenosine is used to accelerate the heart rate

The nurse is aware that age-related changes in the heart muscle put the elderly at risk for dyspnea, angina, and syncope. Which of the following is an age-related change in the cardiovascular system that affects the sympathetic nervous system? a) An increased contractility response to exercise b) A decreased response to beta-blockers c) Decreased time for the heart rate to return to baseline d) Tachycardia

b) A decreased response to beta-blockers The sympathetic nervous system exhibits structural and functional changes that are age-related. Heart rate will decrease, and it will take longer for the heart rate to return to baseline. Refer to Table 12-1 in the text.

Which of the following would be a factor that may decrease myocardial contractility? a) Administration of digoxin (Lanoxin) b) Acidosis c) Sympathetic activity d) Alkalosis

b) Acidosis Contractility is depressed by hypoxemia, acidosis, and certain medications, such as beta-adrenergic blocking medications. Contractility is enhanced by sympathetic neuronal activity, and certain medications, such as Lanoxin.

Which of the following is a true statement regarding the role of baroreceptors? a) Increases in heart rate b) Initiates the parasympathetic response c) Initiates the sympathetic response d) Increases blood pressure

b) Initiates the parasympathetic response During elevations of blood pressure, the baroreceptors increase their rate of discharge. This initiates parasympathetic activity and inhibits sympathetic response, lowering the heart rate and blood pressure.

You are evaluating the expected outcomes on a client who is recovering from a cardiac catheterization. What is an expected outcome that you would evaluate? a) The client and family understands the need for medication. b) The client and family understands the discharge instructions. c) The client and family understands the need to restrict activity for 72 hours. d) The client and family understands the client's CV diagnosis.

b) The client and family understands the discharge instructions. The client is relaxed and feels secure. The test is performed uneventfully or the client is stabilized when complications are managed successfully. The client and family have an accurate understanding of the diagnostic testing process and discharge instructions. The scenario does not indicate that the client has a CV diagnosis, a need for medication, or a need to restrict their activity for 72 hours.

For both outpatients and inpatients scheduled for diagnostic procedures of the cardiovascular system, the nurse performs a thorough initial assessment to establish accurate baseline data. Which of the following data is necessary to collect if the patient is experiencing chest pain? a)Blood pressure in the left arm b)Description of the pain c)Pulse rate in upper extremities d)Sound of the apical pulses

b)Description of the pain

The nurse is reviewing discharge instructions with a patient who underwent a left groin cardiac catheterization 8 hours ago. Which of the following instructions should the nurse include? a) "You can take a tub bath or a shower when you get home." b) "Do not bend at the waist, strain, or lift heavy objects for the next 24 hours." c) "Contact your primary care provider if you develop a temperature above 102°F." d) "If any discharge occurs at the puncture site, call 911 immediately."

b. "Do not bend at the waist, strain, or lift heavy objects for the next 24 hours." Explanation: The nurse should instruct the patient to complete the following: If the artery of the groin was used, for the next 24 hours, do not bend at the waist, strain, or lift heavy objects; the primary provider should be contacted if any of the following occur: swelling, new bruising or pain from your procedure puncture site, temperature of 101°F or more. If bleeding occurs, lie down (groin approach) and apply firm pressure to the puncture site for 10 minutes. Notify the primary provider as soon as possible and follow instructions. If there is a large amount of bleeding, call 911. The patient should not drive to the hospital

The nurse in a cardiac clinic is taking vital signs of a 58-year-old man who is 3 months status post myocardial infarction (MI). While the physician is seeing the client, the client's spouse approaches the nurse and asks about sexual activity. "We are too afraid he will have another heart attack, so we just don't have sex anymore." The nurse's best response is which of the following? a) "Having an orgasm is very strenuous and your husband must be in excellent physical shape before attempting it." b) "The physiologic demands are greatest during orgasm and are equivalent to walking 3 to 4 miles per hour on a treadmill." c) "It is usually better to just give up sex after a heart attack." d) "The medications will prevent your husband from having an erection."

b. "The physiologic demands are greatest during orgasm and are equivalent to walking 3 to 4 miles per hour on a treadmill." Explanation: The physiologic demands are greatest during orgasm. The level of activity is equivalent to walking 3 to 4 miles per hour on a treadmill. Erectile dysfunction may be a side effect of beta-blockers, but other medications may be substituted.

The nurse is caring for a client on a monitored telemetry unit. During morning assessment, the nurse notes abnormal ECG waves on the telemetry monitor. Which action would the nurse do first? a) Assess for mechanical dysfunction. b) Assess the client. c) Call the physician with a report. d) Reposition the client.

b. Assess the client. When a nurse notes an abnormal rhythm on a telemetry monitor, the first action is to assess the client. After client assessment, the nurse is able to make an informed decision on the next nursing action.

The nurse is providing discharge education for the client going home after a cardiac catheterization. Which of the following would be important information to give this client? a) Expect bruising to appear at the site. b) Avoid tub baths, but shower as desired. c) Returning to work immediately is okay. d) Do not ambulate until the physician indicates it is appropriate.

b. Avoid tub baths, but shower as desired. Explanation: Guidelines for self-care after hospital discharge following a cardiac catheterization include shower as desired (no tub baths), avoid bending at the waist and lifting heavy objects, the physician will indicate when it is okay to return to work, and notify the physician right away if you have bleeding, new bruising, swelling, or pain at the puncture site.

The nurse is caring for a patient with an intra-arterial BP monitoring device. The nurse recognizes the most preventable complication associated with hemodynamic monitoring includes which of the following? a) Pneumothorax b) Catheter-related bloodstream infections (CRBSI) c) Hemorrhage d) Air embolism

b. Catheter-related bloodstream infections (CRBSI) Explanation: CRBSIs are the most common preventable complication associated with hemodynamic monitoring systems. Comprehensive guidelines for the prevention of these infections have been published by Centers for Disease Control and Prevention (CDC). Complications from use of hemodynamic monitoring systems are uncommon and can include pneumothorax, infection, and air embolism. A pneumothorax may occur during the insertion of catheters using a central venous approach (CVP and pulmonary artery catheters). Air emboli can be introduced into the vascular system if the stopcocks attached to the pressure transducers are mishandled during blood drawing, administration of medications, or other procedures that require opening the system to air.

You are monitoring the results of laboratory tests performed on a client admitted to the cardiac ICU with a diagnosis of myocardial infarction. Which test would you expect to show elevated levels? a) RBC b) Enzymes c) WBC d) Platelets

b. Enzymes Explanation: When tissues and cells break down, are damaged, or die, great quantities of certain enzymes are released into the bloodstream. Enzymes can be elevated in response to cardiac or other organ damage. After an MI, RBCs and platelets should not be elevated. WBCs would only be elevated if there was a bacterial infection present.

You are working on a telemetry unit. Your client was admitted with a cardiac event and is now on a cardiac monitor. You know a cardiac monitor reveals the heart's electrical but not its mechanical activity. How would you assess the mechanical activity of the client's heart? a) Take the blood pressure in both arms. b) Palpate a peripheral pulse. c) Percuss the perimeter of the heart. d) Auscultate the carotid artery.

b. Palpate a peripheral pulse. Explanation: A cardiac monitor reveals the heart's electrical but not its mechanical activity. The healthcare provider must palpate a peripheral pulse or auscultate the apical heart rate to obtain this information. You cannot obtain information on the mechanical activity of the heart by taking the client's blood pressure, auscultating the carotid artery, or attempting to percuss the perimeter of the heart.

The nurse is observing a patient during an exercise stress test (bicycle). Which of the following findings indicates a positive test and the need for further diagnostic testing? a) BP changes; 148/80 mm Hg to 166/90 mm Hg b) ST-segment changes on the ECG c) Heart rate changes; 78 bpm to 112 bpm d) Dizziness and leg cramping

b. ST-segment changes on the ECG Explanation: During the test, the following are monitored: two or more ECG leads for heart rate, rhythm, and ischemic changes; BP; skin temperature; physical appearance; perceived exertion; and symptoms, including chest pain, dyspnea, dizziness, leg cramping, and fatigue. The test is terminated when the target heart rate is achieved or if the patient experiences signs of myocardial ischemia. Further diagnostic testing, such as a cardiac catheterization, may be warranted if the patient develops chest pain, extreme fatigue, a decrease in BP or pulse rate, serious dysrhythmias or ST-segment changes on the ECG during the stress test. The other findings would not warrant the testing to be stopped.

Which of the following terms describes the amount of blood ejected per heartbeat? a) Ejection fraction b) Stroke volume c) Afterload d) Cardiac output

b. Stroke volume Explanation: Stroke volume is determined by preload, afterload, and contractility of the heart. Cardiac output is the amount of blood pumped by each ventricle during a given period and is computed by multiplying the stroke volume of the heart by the heart rate. Ejection fraction is the percentage of the end-diastolic volume that is ejected with each stroke, measured at 42% to 50% in the normal heart. Afterload is defined as the pressure that the ventricular myocardium must overcome to eject blood during systole and is one of the determinants of stroke volume.

A patient recently diagnosed with pericarditis asks his nurse to explain what area of his heart is involved. The nurse tells the patient that the pericardium, which is inflamed, is the: a) Inner lining of the heart and valves. b) Thin fibrous sac that encases the heart. c) Exterior layer of the heart. d) Heart's muscle fibers.

b. Thin fibrous sac that encases the heart. Explanation: The pericardium is a thin, fibrous sac that encases the heart. It is composed of two layers, the visceral and the parietal pericardium. The space between these two layers is filled with fluid.

The client, an 83-year-old man, is admitted with heart faillure. The nurse is aware that education needed prior to discharge includes which of the following? a) Continue to increase the amount of exercise, because cardiac output increases with age. b) Try to avoid emotional stress and take part in mild physical stress only. c) Any kind of stress is acceptable, because the aging heart has an increased ability to respond. d) Exercise tolerance should remain the same as in younger years.

b. Try to avoid emotional stress and take part in mild physical stress only. Explanation: Stressful physical and emotional conditions may have adverse effects on the aged person's heart.

Decreased pulse pressure reflects a) reduced distensibility of the arteries. b) reduced stroke volume. c) tachycardia. d) elevated stroke volume.

b. reduced stroke volume. Explanation: Decreased pulse pressure reflects reduced stroke volume and ejection velocity or obstruction to blood flow during systole. Increased pulse pressure would indicate reduced distensibility of the arteries, along with bradycardia.

In caring for a client with vasovagal syncope, the nurse should know that the associated temporary loss of consciousness is most commonly related to:

bradyrhythmia

The nurse admits a 52-year-old woman with a medical diagnosis of "rule out MI." The client is very frightened and expresses surprise that a woman would have heart problems. Which of the following responses by the nurse would be most appropriate? a) "It takes longer for an electrical impulse to travel from the sinoatrial node to the atrioventricular node." b) "A woman's resting heart rate is lower than a man's." c) "A woman's heart is smaller and has smaller arteries that become occluded more easily." d) "The stroke volume from a woman's heart is lower than from a man's heart.

c) "A woman's heart is smaller and has smaller arteries that become occluded more easily."

The nurse is caring for a patient with diabetes who is scheduled for a cardiac catheterization. Prior to the procedure, it is most important for the nurse to ask which of the following questions? a) "Are you having chest pain?" b) "What was your morning blood sugar reading?" c) "Are you allergic to shellfish?" d) "When was the last time you ate or drank?"

c. "Are you allergic to shellfish?" Explanation: Radiopaque contrast agents are used to visualize the coronary arteries. Some contrast agents contain iodine, and the patient is assessed before the procedure for previous reactions to contrast agents or allergies to iodine-containing substances (e.g., seafood). If the patient has a suspected or known allergy to the substance, antihistamines or methylprednisolone (Solu-Medrol) may be administered before the procedure. Although the other questions are important to ask the patient, it is most important to ascertain if the patient has an allergy to shellfish.

The ability of the cardiac muscle to shorten in response to an electrical impulse is termed

contractility

A nurse is checking laboratory values on a client who has crackles in the lower lobes, 2+ pitting edema, and dyspnea with minimal exertion. Which laboratory value does the nurse expect to be abnormal? a) Platelet count b) Potassium c) B-type natriuretic peptide (BNP) d) C-reactive protein (CRP)

c. B-type natriuretic peptide (BNP) Explanation: The client's symptoms suggest heart failure. BNP is a neurohormone that's released from the ventricles when the ventricles experience increased pressure and stretch, such as in heart failure. A BNP level greater than 51 pg/ml is commonly associated with mild heart failure. As the BNP level increases, the severity of heart failure increases. Potassium levels aren't affected by heart failure. CRP is an indicator of inflammation. It's used to help predict the risk of coronary artery disease. There is no indication that the client has an increased CRP. There is no indication that the client is experiencing bleeding abnormalities, such as those seen with an abnormal platelet count.

The nurse is caring for a client who has just returned from the cadiac catheterization laboratory following a coronary angioplasty. Which of the following assessments will the nurse do immediately? a) Assess for PERRLA. b) Check the client's tolerance to ambulation to the bathroom. c) Check the temperature, color, and capillary refill of the affected extremity. d) Assess when the client last had a bowel movement.

c. Check the temperature, color, and capillary refill of the affected extremity. Explanation: Nursing responsibilities after cardiac catheterization include frequent checks of the temperature, color, and capillary refill of the affected extremity. The client is also assessed for extremity pain, numbness, or tingling sensation. These could all indicate arterial insufficiency.

A patient has been diagnosed with congestive heart failure (CHF). The physician has ordered a medication to enhance contractility. The nurse would expect which medication to be ordered for the patient? a) Clopidogrel (Plavix) b) Enoxaparin (Lovenox) c) Digoxin (Lanoxin) d) Heparin

c. Digoxin (Lanoxin) Explanation: Contractility is enhanced by circulating catecholamines, sympathetic neuronal activity, and certain medications, such as Lanoxin. Increased contractility results in increased stroke volume. The other medications are classified as platelet-inhibiting medications.

The nurse cares for a client prescribed warfarin orally. The nurse reviews the client's prothrombin time (PT) level to evaluate the effectiveness of the medication. Which laboratory values should the nurse also evaluate?

international normalized ratio (INR)

The client with a diagnosis of heart failure reports frequently awakening during the night with the need to urinate. The nurse offers which explanation? a) The blood pressure is lower when the client is recumbent and this causes the kidneys to work harder; therefore, more urine is produced. b) Fluid that is held in the lungs during the day becomes part of the circulation at night and the kidneys produce an increased amount of urine. c) Edema is collected in dependent extremities during the day; at night when the client lays down, it is reabsorbed into the circulation and excreted by the kidneys. d) When the client is in the recumbent position, more pressure is put on the bladder with the result of increased need to urinate.

c. Edema is collected in dependent extremities during the day; at night when the client lays down, it is reabsorbed into the circulation and excreted by the kidneys. Explanation: Nocturia is common in patients with heart failure. Fluid collected in dependent areas during the day is reabsorbed into the circulation at night when the client is recumbent. The kidneys excrete more urine with the increased circulating volume.

Which of the following is an early warning symptom of acute coronary syndrome (ACS) and heart failure (HF)? a) Weight gain b) Change in level of consciousness c) Fatigue d) Hypotension

c. Fatigue Explanation: Fatigue is an early warning symptom of ACS, heart failure, and valvular disease. Other signs and symptoms of cardiovascular disease are hypotension, change in level of consciousness, and weight gain.

The nurse is caring for a patient prescribed warfarin (Coumadin) orally. The nurse reviews the patient's prothrombin time (PT) level to evaluate the effectiveness of the medication. The nurse should also evaluate which of the following laboratory values? a) Complete blood count (CBC) b) Sodium c) International normalized ratio (INR) d) Partial thromboplastic time (PTT)

c. International normalized ratio (INR) Explanation: The INR, reported with the PT, provides a standard method for reporting PT levels and eliminates the variation of PT results from different laboratories. The INR, rather than the PT alone, is used to monitor the effectiveness of warfarin. The therapeutic range for INR is 2 to 3.5, although specific ranges vary based on diagnosis. The other laboratory values are not used to evaluate the effectiveness of Coumadin.

Within the physiology of the heart, each chamber has a particular role in maintaining cellular oxygenation. Which chamber of the heart is responsible for receiving oxygenated blood from the lungs? a) Right ventricle b) Right atrium c) Left atrium d) Left ventricle

c. Left atrium The left atrium receives oxygenated blood from the lungs. The left ventricle pumps that blood to all the cells and tissues of the body. The right atrium receives deoxygenated blood from the venous system. The right ventricle pumps that blood to the lungs to be oxygenated.

The nurse is caring for a patient with clubbing of the fingers and toes. The nurse should complete which of the following actions given these findings? a) Assess the patient's capillary refill. b) Obtain a 12-lead ECG tracing. c) Obtain an oxygen saturation level. d) Assess the patient for pitting edema.

c. Obtain an oxygen saturation level. Explanation: Clubbing of the fingers and toes indicates chronic hemoglobin desaturation (decreased oxygen supply) and is associated with congenital heart disease. The nurse should assess the patient's O2 saturation level and intervene as directed. The other assessments are not indicated.

The nurse is providing discharge instructions to a client with unstable angina. The client is ordered Nitrostat 1/150 every 5 minutes as needed for angina. Which side effect, emphasized by the nurse, is common especially with the increased dosage? a) Nausea b) Rash c) Orthostatic hypotension d) Dry mouth

c. Orthostatic hypotension Explanation: A common side effect of Nitrostat, especially at higher dosages, is orthostatic hypotension. The action of the medication is to dilate the blood vessels to improve circulation to the heart. The side effect of the medication is orthostatic hypotension. A rash, nausea, and dry mouth are not common side effects.

Which of the following nursing interventions is most appropriate when caring for a client with a nursing diagnosis of risk for injury related to side effects of medication (enoxaparin [Lovenox])? a) Administer calcium supplements. b) Assess for hypokalemia. c) Report any incident of bloody urine, stools, or both. d) Assess for clubbing of the fingers.

c. Report any incident of bloody urine, stools, or both. Explanation: The client who takes an anticoagulant, such as a low-molecular-weight heparin, is routinely screened for bloody urine, stools, or both.

Which of the following is the term for the normal pacemaker of the heart? a) Bundle of His b) Atrioventricular (AV) node c) Sinoatrial (SA) node d) Purkinje fibers

c. Sinoatrial (SA) node Explanation: The sinoatrial node is the primary pacemaker of the heart. The AV node coordinates the incoming electrical impulses from the atria and, after a slight delay, relays the impulse to the ventricles. The Purkinje fibers rapidly conduct the impulses through the thick walls of the ventricles.

The client is admitted for a scheduled cardiac catheterization. On the morning of the procedure, while assessing the client's morning laboratory values, the nurse notes a blood urea nitrogen (BUN) of 34 mg/dL and a creatinine of 4.2 mg/dL. The nurse makes it a priority to notify the physician for which of the following reasons? a) The client is at risk for bleeding. b) The client is over-hydrated, which puts him at risk for heart failure during the procedure. c) The client is at risk for renal failure due to the contrast agent that will be given during the procedure. d) These values show a risk for dysrhythmias.

c. The client is at risk for renal failure due to the contrast agent that will be given during the procedure. Explanation: The contrast medium must be excreted by the kidneys. If there is already a degree of renal impairment (which these laboratory values indicate), the risk for contrast agent-induced nepropathy and renal failure is high.

The nurse is caring for a patient with a diagnosis of pericarditis. Where does the nurse understand the inflammation is located? a) The exterior layer of the heart b) The heart's muscle fibers c) The thin fibrous sac encasing the heart d) The inner lining of the heart and valves

c. The thin fibrous sac encasing the heart Explanation: The heart is encased in a thin, fibrous sac called the pericardium, which is composed of two layers. Inflammation of this sac is known as pericarditis.

The nurse is awaiting results of cardiac biomarkers for a patient with severe chest pain. The nurse would identify which cardiac biomarker as remaining elevated the longest when myocardial damage has occurred? a) Myoglobin b) CK-MB c) Troponin T and I d) Brain natriuretic peptide (BNP)

c. Troponin T and I Explanation: After myocardial injury, these biomarkers rise early (within 3 to 4 hours), peak in 4 to 24 hours, and remain elevated for 1 to 3 weeks. These early and prolonged elevations may make very early diagnosis of acute myocardial infarction (MI) possible and allow for late diagnosis in patients who have delayed seeking care for several days after the onset of acute MI symptoms. CK-MB returns to normal within 3 to 4 days. Myoglobin returns to normal within 24 hours. BNP is not considered a cardiac biomarker. It is a neurohormone that responds to volume overload in the heart by acting as a diuretic and vasodilator.

The nurse cares for a client with an intra-arterial blood pressure monitoring device. The nurse recognizes the most preventable complication associated with hemodynamic monitoring includes which condition?

catheter-related bloodstream infections

During auscultation of the lungs, what would a nurse note when assessing a client with left-sided heart failure?

wheezes with wet lung sounds

While auscultating the heart sounds of a client with heart failure, the nurse hears an extra heart sound immediately after the second heart sound (S2). The nurse should document this as: 1. a first heart sound (S1). 2. a third heart sound (S3). 3. a fourth heart sound (S4). 4. a murmur.

correct Answer: 2 RATIONALES: An S3 is heard following an S2, which commonly occurs in clients experiencing heart failure and results from increased filling pressures. An S1 is a normal heart sound made by the closing of the mitral and tricuspid valves. An S4 is heard before an S1 and is caused by resistance to ventricular filling. A murmur is heard when there is turbulent blood flow across the valves.

The nurse is caring for a client anticipating further testing related to cardiac blood flow. Which statement, made by the client, would lead the nurse to provide additional teaching? a) "The first test I am getting is an echocardiography. I am glad that it is not painful." b) "I had an ECG already. It provided information on my heart rhythm. c) "I am able to have a nuclide study because I do not have any allergies." d) "My niece thought that I would be ordered a magnetic resonance imaging even though I have a pacemaker."

d) "My niece thought that I would be ordered a magnetic resonance imaging even though I have a pacemaker." A magnetic resonance imaging (MRI) test is prohibited on clients with various metal devices within their body. External metal objects must be removed. All other options are correct statements not needing clarification.

The client states, "My doctor says that because I am now taking this water pill, I need to eat more foods that contain potassium. Can you give me some ideas about what foods would be good for this?" The nurse's appropriate response is which of the following? a) Bok choy, cooked leeks, alfalfa sprouts b) Cranberries, apples, popcorn c) Asparagus, blueberries, green beans d) Apricots, dried peas and beans, dates

d) Apricots, dried peas and beans, dates Apricots, dried peas and beans, dates, and kiwi contain high amounts of potassium. The other foods listed contain minimal amounts.

The clinic nurse caring for a client with a cardiovascular disorder is performing an assessment of the client's pulse. Which of the following steps is involved in determining the pulse deficit? a) Calculate the palpated volume. b) Count the radial pulse for 20 to 25 seconds. c) Calculate the pauses between pulsations. d) Count the heart rate at the apex.

d) Count the heart rate at the apex.

Which of the following is an early warning symptom of acute coronary syndrome (ACS) and heart failure (HF)? a) Hypotension b) Change in level of consciousness c) Weight gain d) Fatigue

d) Fatigue Fatigue is an early warning symptom of ACS, heart failure, and valvular disease. Other signs and symptoms of cardiovascular disease are hypotension, change in level of consciousness, and weight gain.

The nurse reviews a patient's lab results and notes a serum calcium level of 7.9 mg/dL. The nurse knows that this reading can also be associated with which of the following? a) Inclination to ventricular fibrillation b) Increased risk of heart block c) Enhanced sensitivity to digitalis d) Impaired myocardial contractility

d) Impaired myocardial contractility

A nurse working in a cardiac step-down unit understands that the following drugs can affect the contractility of the heart. The nurse recognizes that contractility is depressed by which of the following drugs? a) Dobutrex b) Lanoxin c) Intropin d) Lopressor

d) Lopressor

A patient's heart rate is observed to be 140 bpm on the monitor. The nurse knows that the patient is at risk for what complication? a) A stroke b) Right-sided heart failure c) A pulmonary embolism d) Myocardial ischemia

d) Myocardial ischemia As heart rate increases, diastolic time is shortened, which may not allow adequate time for myocardial perfusion. As a result, patients are at risk for myocardial ischemia (inadequate oxygen supply) during tachycardias (heart rate greater than 100 bpm), especially patients with coronary artery disease.

The nurse is caring for a client anticipating further testing related to cardiac blood flow. Which statement, made by the client, would lead the nurse to provide additional teaching? a) "I am able to have a nuclide study because I do not have any allergies." b) "I had an ECG already. It provided information on my heart rhythm. c) "The first test I am getting is an echocardiography. I am glad that it is not painful." d) "My niece thought that I would be ordered a magnetic resonance imaging even though I have a pacemaker."

d. "My niece thought that I would be ordered a magnetic resonance imaging even though I have a pacemaker." Explanation: A magnetic resonance imaging (MRI) test is prohibited on clients with various metal devices within their body. External metal objects must be removed. All other options are correct statements not needing clarification.

The nurse is caring for a client with an elevated blood pressure and no previous history of hypertension. At 0900, the blood pressure was 158/90 mm Hg. At 0930, the blood pressure is 142/82 mm Hg. The nurse is most correct when relating the fall in blood pressure to which structure? a) Vagus nerve b) Chemoreceptors c) Sympathetic nerve fibers d) Baroreceptors

d. Baroreceptors Explanation: Baroreceptor sense pressure in nerve endings in the walls of the atria and major blood vessels. The baroreceptors respond accordingly to raise or lower the pressure. Chemoreceptors are sensitive to pH, CO2, and O2 in the blood. Sympathetic nerve fibers increase the heart rate. The vagus nerve slows the heart rate.

The nurse is preparing to apply ECG electrodes to a male patient who requires continuous cardiac monitoring. Which of the following should the nurse complete to optimize skin adherence and conduction of the heart's electrical current? a) Apply baby powder to the patient's chest prior to placing the electrodes. b) Once the electrodes are applied, change them every 72 hours. c) Clean the patient's chest with alcohol prior to application of the electrodes. d) Clip the patient's chest hair prior to applying the electrodes.

d. Clip the patient's chest hair prior to applying the electrodes. Explanation: The nurse should complete the following actions when applying cardiac electrodes: Clip (do not shave) hair from around the electrode site, if needed; if the patient is diaphoretic (sweaty), apply a small amount of benzoin to the skin, avoiding the area under the center of the electrode; debride the skin surface of dead cells with soap and water and dry well (or as recommended by the manufacturer). Change the electrodes every 24 to 48 hours (or as recommended by the manufacturer); examine the skin for irritation and apply the electrodes to different locations.

A nurse is reviewing laboratory values for a client diagnosed with hyperlipidemia 6 months ago. Which results indicate that the client has been following his therapeutic regimen? a) Triglycerides increase from 225 mg/dl to 250 mg/dl. b) Total cholesterol level increases from 250 mg/dl to 275 mg/dl. c) Low density lipoproteins (LDL) increase from 180 mg/dl to 190 mg/dl. d) High density lipoproteins (HDL) increase from 25 mg/dl to 40 mg/dl.

d. High density lipoproteins (HDL) increase from 25 mg/dl to 40 mg/dl. Explanation: The goal of treating hyperlipidemia is to decrease total cholesterol and LDL levels while increasing HDL levels. HDL levels should be greater than 35 mg/dl. This client's increased HDL levels indicate that he's followed his therapeutic regimen. Recommended total cholesterol levels are below 200 mg/dl. LDL levels should be less than 160 mg/dl, or, in clients with known coronary artery disease (CAD) or diabetes mellitus, less than 70 mg/dl. Triglyceride levels should be between 100 and 200 mg/d.

The nurse is reviewing the morning laboratory test results for a client with cardiac problems. Which of the following would the nurse regard as a priority to report to the physician? a) Mg++ 2 mE/L b) Na+ 140 mEq/L c) Ca++ 9 mg/dL d) K+ 3.1 mEq/L

d. K+ 3.1 mEq/L Explanation: All are within normal limits except for the K+, which is low. A low K+ level can cause ventricular tachycardia or fibrillation.

The nurse is administering a beta blocker to a patient in order to decrease automaticity. Which medication will the nurse administer? a) Cordarone b) Cardizem c) Rythmol d) Lopressor

d. Lopressor Explanation: Patients may receive beta-blockers prior to the scan to control heart rate and rhythm.

When preparing a patient for a cardiac catheterization, the patient states that she has allergies to seafood. Which of the following medications may give to her prior to the procedure? a) Phenytoin (Dilantin) b) Furosemide (Lasix) c) Lorazepam (Ativan) d) Methylprednisolone (Solu-Medrol)

d. Methylprednisolone (Solu-Medrol) Explanation: Prior to cardiac catheterization, the patient is assessed for previous reactions to contrast agents or allergies to iodine-containing substances, as some contrast agents contain iodine. If allergic reactions are of concern, antihistamines or methylprednisolone (Solu-Medrol) may be administered to the patient before angiography is performed. Lasix, Ativan, and Dilantin do not counteract allergic reactions.

The nurse is caring for a client with ECG changes consistent with a myocardial infarction. Which of the following diagnostic test does the nurse anticipate to confirm heart damage? a) Chest radiography b) Serum blood work c) Fluoroscopy d) Nuclear cardiology

d. Nuclear cardiology Explanation: Nuclear cardiology uses a radionuclide to detect areas of myocardial damage. Chest radiography and fluoroscopy determine the size and position of the heart and condition of the lungs. Serum blood work notes elevations in enzymes suggesting muscle damage.

Which of the follow arteries carries deoxygenated blood? a) Left coronary artery b) Left anterior descending artery c) Right coronary artery d) Pulmonary artery

d. Pulmonary artery Explanation: The pulmonary artery is the only artery carrying deoxygenated blood. Oxygenated blood returns to the left atrium via the pulmonary veins. The left coronary artery, right coronary artery, and left anterior descending artery do not carry deoxygenated blood.

The physician orders medication to treat a client's cardiac ischemia. The nurse is aware that which of the following is causing the client's condition? a) Indigestion b) Pain on exertion c) High blood pressure d) Reduced blood supply to the heart

d. Reduced blood supply to the heart Explanation: Ischemia is reduced blood supply to body organs. Cardiac ischemia is caused by reduced blood supply to the heart muscle. It may lead to a myocardial infarction. Chest pain is a symptom of ischemia. Ischemia is reduced blood supply to body organs. Ischemia is reduced blood supply to body organs.

Central venous pressure is measured in which of the following heart chambers? a) Left ventricle b) Right ventricle c) Left atrium d) Right atrium

d. Right atrium Explanation: The pressure in the right atrium is used to assess right ventricular function and venous blood return to the heart. The left atrium receives oxygenated blood from the pulmonary circulation. The left ventricle receives oxygenated blood from the left atrium. The right ventricle is not the central collecting chamber of venous circulation.

A patient tells the nurse, "I was straining to have a bowel movement and felt like I was going to faint. I took my pulse and it was so slow." What does the nurse understand occurred with this patient? a) The patient may have had a myocardial infarction. b) The patient may have an abdominal aortic aneurysm. c) The patient was anxious about being constipated. d) The patient had a vagal response.

d. The patient had a vagal response. Explanation: When straining during defecation, the patient bears down (the Valsalva maneuver), which momentarily increases pressure on the baroreceptors. This triggers a vagal response, causing the heart rate to slow and resulting in syncope in some patients. Straining during urination can produce the same response.

Following a myocardial infarction, a client develops an arrhythmia and requires a continuous infusion of lidocaine. To monitor the effectiveness of the intervention, the nurse should focus primarily on the client's:

electrocardiogram (ECG)

Within the heart, several structures and several layers all play a part in protecting the heart muscle and maintaining cardiac function. The inner layer of the heart is composed of a thin, smooth layer of cells, the folds of which form heart valves. What is the name of this layer of cardiac tissue?

endocardium

Which symptom is an early warning sign of acute coronary syndrome (ACS) and heart failure (HF)?

fatigue

The nurse cares for a client in the emergency department who has a B-type natriuretic peptide (BNP) level of 115 pg/mL. The nurse recognizes that this finding is most indicative of which condition?

heart failure

The nurse instructor is teaching a group of nursing students about adventitious heart sounds. The instructor explains that auscultation of the heart requires familiarization with normal and abnormal heart sounds. What would the instructor tell these students a ventricular gallop indicates in an adult?

heart failure

Each chamber of the heart has a particular role in maintaining cellular oxygenation. Which chamber is responsible for receiving oxygenated blood from the lungs?

left atrium

Each chamber of the heart has a particular role in maintaining cellular oxygenation. Which chamber is responsible for pumping blood to all the cells and tissues of the body?

left ventricle

A student nurse prepares to assess a client for postural blood pressure changes. Which action indicates the student nurse needs further education?

letting 30 seconds elapse after each position change before measuring BP and HR

The nurse is assessing a patient's electrocardiogram (ECG). What phase does the nurse determine is the resting phase before the next depolarization?

phase 4

The physician orders medication to treat a client's cardiac ischemia. What is causing the client's condition?

reduced blood supply to the heart

Decreased pulse pressure reflects

reduced stroke volume

What does decreased pulse pressure reflect?

reduced stroke volume

Central venous pressure is measured in which heart chamber?

right atrium

circulation of blood in heart:

right atrium to right ventricle. The blood flows to the lungs via the pulmonary artery and returns to the heart in an oxygenated state via the pulmonary vein. The oxygenated blood then enters the left atrium then left ventricle pump through the aorta to the systemic circulation

Each chamber of the heart has a particular role in maintaining cellular oxygenation. Which chamber is responsible for pumping blood to the lungs to be oxygenated?

right ventricle

The nurse correctly identifies which data as an example of blood pressure and heart rate measurements in a client with postural hypotension?

supine: BP 120/70 mm Hg, HR 70 bpm; sitting: BP 100/55 mm Hg, HR 90 bpm; standing: BP 98/52 mm Hg, HR 94 bpm

What would you tell student a ventricular gallop indicates in an adult

the sound that fallows the S1 and S2 sounds is called S3 or ventricular gallop

You are caring for a client with a damaged tricuspid valve. You know that the tricuspid valve is held in place by which of the following

the tricuspid valve is between the right atrium and the right ventricle *Attached to the mitral and tricuspid valves are cordlike structures known as chordae tendineae, which in turn attach to papillary muscles, two major muscular projections from the ventricles.

An obese client describes symptoms of palpitations, chronic fatigue, and dyspnea on exertion to the cardiologist. Upon completing the examination, the cardiologist schedules a procedure to confirm the suspected diagnosis. What diagnostic procedure would the nurse expect to be prescribed?

transesophageal echocardiography

Which test used to diagnose heart disease is least invasive?

transthoracic echocardiography

The nurse is caring for a patient following the insertion of a permanent pacemaker. Which of the following discharge instructions are appropriate for the nurse to review with the patient? Select all that apply. a) Wear a medical alert noting the presence of a pacemaker. b) Refrain from walking through antitheft devices. c) Avoid the usage of microwave ovens and electronic tools. d) Avoid handheld screening devices in airports. e) Check pulse daily, reporting sudden slowing or increase.

• Wear a medical alert noting the presence of a pacemaker. • Check pulse daily, reporting sudden slowing or increase. • Avoid handheld screening devices in airports. Handheld screening devices used in airports may interfere with the pacemaker. Patients should be advised to ask security personnel to perform a hand search instead of using the handheld screening device. With a permanent pacemaker, the patient should be instructed initially to restrict activity on the side of implantation. Patients also should be educated to perform a pulse check daily and to wear or carry medical identification to alert personnel to the presence of the pacemaker. Patients should walk through antitheft devices quickly and avoid standing in or near these devices. Patients can safely use microwave ovens and electronic tools.


Kaugnay na mga set ng pag-aaral

CCNA 3 Scaling Networks Final Study Guide

View Set

Займенник як частина мови 4 клас

View Set

Ethics for Life Chapter 3- Conscience and Moral Development

View Set

MGMT 464 Final Exam SELU Honoree

View Set

EC 202: Inflation and the Quantity Theory of Money (Quiz)

View Set

The Great Patriotic War - Reasons and Results of Victory

View Set

corporate compliance and regulation final

View Set